Boards Part II (Mosby's)

¡Supera tus tareas y exámenes ahora con Quizwiz!

The following teeth are erupted in an 8-year-old patient. What is the space maintenance of choice? A. Band-loop space maintainer B. Lower lingual holding arch C. Nance holding arch D. Distal shoe space maintainer

A, A band-loop space maintainer would work well in this case because the maxillary first bicuspid normally erupts prior to the loss of either the second primary molar or the primary cuspid

You are evaluating a patient 5 days after extraction of tooth #17. The patient complains of a severe throbbing pain that started yesterday, 4 days after extraction. The patient most likely has which of the following conditions? A. Dry socket B. Subperiosteal abscess C. Periapical periodontitis in tooth #18 D. Neuropathic pain

A, A dry socket (alveolar osteitis) occurs on the third to fourth day after extraction and, except for pain, does not have the classic signs of infection

Even though the state-of-the-art treatment for facial fractures is with internal rigid fixation using bone plates and screws, a proper occlusal relationship must be established prior to fixation of the bony segments if the reduction is to be satisfactory. A. True B. False

A, A proper occlusal relationship is a prerequisite for satisfactory bony reduction. This is most commonly accomplished by the use if intermaxillary fixation, or wiring the jaws closed, during surgery

During a previous dental visit, you assisted a patient by generating his statement, "Even if there is some pain, it will be brief. I have ways to cope and I've done well using them." The patient will remind himself of this during future dental procedures. This patient's statement exemplifies which of the following strategies? A. Rational response B. Self-efficacy induction C. Relaxation statement D. Imagery E. Systematic desensitization

A, A rational response is a cognitive therapy technique in which the patient develops (with or without assistance) a more adaptive thought or statement as a means of coping

The term ALARA refers to _____. A. Reducing patient exposure to as low as is reasonably achievable B. As little as Roentgen allowed, an algorithm for limiting patient exposure C. A legal requirement to optimize occupational exposure in dental radiology D. Optimizing image quality E. Reducing the costs of radiographic examinations

A, ALARA (As Low As Reasonably Achievable) is a concept for minimizing patient and occupational exposure.

A patient complains of recent severe pain to percussion of a tooth. The most likely cause is _____. A. Acute periradicular periodontitis B. Chronic periradicular periodontitis C. Reversible pulpitis D. Irreversible pulpitis

A, Acute apical (periradicular) periodontitis (AAP): characterized by pain, commonly triggered by chewing or percussion. AAP alone is not indicative of irreversible pulpitis. It indicates that apical tissues are irritated, which may be associated with an otherwise vital pulp.

If your patient has a history of liver disease, which of the following would be the safest local anesthetic? A. Articaine B. Prilocaine C. Lidocaine D. Bupivacaine

A, All amide local anesthetics are biotransformed in the liver. One available local anesthetic also has an ester side chain, which means it has some degree of extrahepatic biotransformation (outside the liver). This drug is articaine and is therefore the most appropriate drug for patients with liver disease.

Inhibiting a-glucosidase and reducing glucose absorption from the gastrointestinal tract is the mechanism of action of which drug? A. Acarbose B. Acetoheximide C. Glyburide D. Metformin E. Pioglitazone

A, All of the choices are oral hypoglycemic agents. Only acarbose inhibits α-glucosidase

Sodium reabsorption in the thick ascending limb of the loop of Henle is inhibited by which drug? A. Bumetanide B. Chlorthalidone C. Hydrochlorothiazide D. Spironolactone E. Triamterene

A, All the drugs listed are diuretics. However, only bumetanide acts on the ascending limb of the loop of Henle. It is called a "loop" or "high ceiling" diuretic because of its site of action in the nephron and maximal effect, respectively.

A 14-year-old male patient has significant plaque build-up and one cavity. Upon inquiry, the patient tells you that he brushes his teeth about once a day and does not floss because it is difficult and too time-consuming. Which of the following would likely be the least effective way to address his oral hygiene practices and get him to improve his oral self-care? A. Educate him regarding the ways in which he can improve his oral hygiene and avoid dental problems in the future. B. Use a collaborative relationship to arrange for modification of consequences. C. Use the Premack principle. D. Set up a behavioral contract. E. Set up a system of positive reinforcers

A, Although education is an essential component of patient care overall, research indicates that, in an effort to influence behavior change, education alone is not nearly as effective as behavioral intervention.

Root resorption is correlated to the pattern of stress distribution in the PDL and type of tooth movement. A. True B. False

A, Although somewhat controversial, it is believed that types of tooth movements that concentrate force in small areas of the PDL are more likely to result in root resorption during orthodontic treatment.

Which statement is false about behavioral contracts? A. It is a legal and binding agreement between health care professional and patient. B. It helps solidify an agreement with a patient. C. It should always be open to modification. D. It helps clarify agreements. E. The clinician should give a copy to the patient and keep one for himself or herself

A, Although the behavioral contract is not a legal document, it can be a useful approach in solidifying behavioral strategies and goals

A painless, well-circumscribed 1 ¥ 3-cm radiolucent lesion with radiopaque focus was found in the posterior mandible of an 11-year-old boy. Which of the following should be included in a differential diagnosis? A. Ameloblastic fibro-odontoma B. Paget's disease C. Dentigerous cyst D. Ameloblastoma E. Langerhans cell disease

A, Ameloblastic fibro-odontoma is the only lesion listed that is lucent with opaque foci. The patient's age is also characteristic for this lesion. Paget's disease may show a mixed opaquelucent pattern, but it occurs only over the age of 50 years

How should a vital second permanent molar with a 2.0-mm exposure on a 12-year-old patient be treated? A. Apexification B. Direct pulp capping C. Indirect pulp capping D. Extract E. Apexogenesis

A, Apexification. Induces further root development in a pulpless tooth; stimulates the formation of a hard substance at the apex so as to allow obturation of the root canal space. 2.0-mm pulp exposure is too big to perform vital pulp therapy. Pulpotomy should not be performed on permanent teeth (unless apexogenesis) because it causes calcification of the root canal system.

When faced with a frightened child patient, which would be the most appropriate or most effective response? A. Ask the child about his or her fears. B. Reschedule the appointment for a later date. C. Reassure the child. D. Tell the child that dentistry shouldn't be frightening. E. Chastise the child.

A, Asking the child about his or her fears will create an environment in which the child is encouraged to discuss any worries or concerns and to ask questions. This will also serve to alleviate anxiety, provide an opportunity to correct any misperceptions regarding dentistry, and to further establish or maintain trust and rapport.

What is the most likely consequence of the avoidance of a feared stimulus? A. Reinforcement of the associated anxiety. B. Habituation to the stimulus. C. Decreased anxiety in response to the stimulus. D. Learned helplessness. E. Increased coping resources.

A, Avoidance of a feared stimulus inadvertently reinforces the anxiety reaction, thereby maintaining the associated anxiety

Oral and genital lesions are seen in patients with which of the following diseases? A. Behçet's syndrome B. Peutz-Jegher's syndrome C. Herpangina D. Wegener's granulomatosis E. Hairy leukoplakia

A, Behçet's syndrome includes lesions in the mouth, eye, and genitals. The other diseases do not affect the genitalia

The enamel rods in the gingival third of primary teeth slope occlusally instead of cervically as in permanent teeth, and the interproximal contacts of primary teeth are broader and flatter than permanent teeth. A. The first statement is true and the second statement is true. B. The first statement is true and the second statement is false. C. The first statement is false and the second statement is true. D. The first statement is false and the second statement is false.

A, Both of these statements are true. As a result of these differences, there are modifications in preparation design for Class II amalgams. Beveling the gingival seat of Class II amalgams are not recommended. There is a greater convergence from cervical to occlusal of the buccal and lingual walls of Class II amalgam preparations because of the broad and flat contact areas

After an injection, which drug would be expected to have the longest duration of action? (Assume no vasoconstrictor was injected with the local anesthetic.) A. Bupivacaine B. Lidocaine C. Mepivacaine D. Prilocaine E. Procaine

A, Bupivacaine has the highest lipid solubility of the drugs listed. This is the major chemical characteristic of the local anesthetic that determines duration of action. Procaine is the only ester given as a choice and is rarely used

Which of the following agencies monitors and prevents disease outbreaks, implements disease prevention strategies, and maintains national health statistics? A. CDC B. FDA C. DEA D. IHS E. None of the above

A, CDC is correct. The U.S. Food and Drug Administration (FDA) is responsible for protecting the health of the nation against impure and unsafe foods, drugs, cosmetics, and other potential hazards. The Drug Enforcement Administration (DEA) determines the levels of controlled substances that have abuse potential. The Indian Health Services (IHS) focuses on the goal of raising the health status of Native Americans and Native Alaskans.

Which of the following is an arrangement between a plan and a group of dentists whereby the providers agree to accept certain payments (usually less than their usual fees) in anticipation of a higher volume of patients? A. PPO B. Capitation C. HMO D. IPA E. None of the above

A, Capitation is a payment mechanism whereby the dentist is paid a fixed amount irrespective of the number of patients seen or services provided. Health Maintenance Organizations (HMOs) are also called capitation plans because of the payment mechanism they use. An Individual Practice Association is a type of plan that combines the risk of capitation with fee for service reimbursement

Which is not a property of sodium hypochlorite (NaOCl)? A. Chelation B. Tissue dissolution at higher concentrations C. Microbicidal activity D. Flotation of debris and lubrication

A, Chelation. Sodium hypochlorite is the most widely used irrigant and has effectively aided canal preparation for years. NaOCl is a good tissue solvent as well as having antimicrobial effect. It acts as a lubricant for root canal instrumentation. It is toxic to vital tissue, so always use a rubber dam. Hypochlorite's antibacterial action is based upon its effects on the bacterial cell wall. Once the cell wall is disrupted, the vital contents of the bacteria are released. The bacterial membrane and intracellular associated functions cease. Sodium hypochlorite is an effective necrotic tissue solvent. NaOCl remains the irrigating solution of choice

The most effective topical antimicrobial agent currently available is _____. A. Chlorhexidine B. Stannous fluoride C. Phenolic compounds D. Sanguinarine

A, Chlorhexidine is the most effective antimicrobial agent currently available

Which type of clasps are generally used on a tooth-supported removable denture? A. Circumferential cast clasp B. Combination clasp C. Wrought wire clasp

A, Circumferential cast clasps are more rigid than combination clasps or wrought wire clasps. Since there is good stability of the prosthesis when the tooth is supported, there is no need for the added flexibility in a normal situation

Which of the following types of malocclusions is most common? A. Class I malocclusion B. Class II malocclusion C. Class III malocclusion D. Open bite malocclusion

A, Class I is the most common malocclusion, at about 50% of the U.S. population, compared to Class II (15%) and Class III (about 1%).

Which of the following is the definition of conscious sedation? A. A minimally depressed level of consciousness that retains the patient's ability to independently and continuously maintain an airway and respond appropriately to physical stimulation or verbal command. B. A significantly depressed level of consciousness that retains the patient's ability to independently and continuously maintain an airway and respond appropriately to physical stimulation or verbal command. C. A minimally depressed level of consciousness that retains the patient's ability to independently and continuously maintain an airway. D. A significantly depressed level of consciousness that retains the patient's ability to independently and continuously maintain an airway

A, Conscious sedation is defined as a minimally depressed level of consciousness as opposed to deep sedation or general anesthesia. Remember that there are four stages of anesthesia (analgesia → delirium → surgical anesthesia → respiratory paralysis) and only in the first stage (analgesia) is the patient conscious. The patient should be able to maintain an airway and respond to stimulation and command.

To ensure high radiographic image quality, it is important to daily _____. A. Check the temperature of the processing solutions B. Clean the processing equipment C. Clean the intensifying screens D. Calibrate the mA linearity

A, Daily check of the processing solution temperature, whether using automatic processing or manual tanks, and comparison with the manufacturer's recommended values will improve image quality. The other procedures are useful but can be performed less frequently.

Which of the following statements is not true regarding bonding systems? A. Even though dentin bonding occurs slowly, it results in a stronger bond than to enamel. B. Enamel bonding occurs quickly, is strong, and is long-lasting. C. One-bottle dentin bonding systems may be simpler but are not necessarily better. D. Dentin bonding is still variable because of factors such as sclerosis, tubule size, and tubule location.

A, Dentin bonding in laboratory studies may create bond strengths similar to or greater than bond strengths to enamel. However, clinical studies cannot corroborate that the dentin bond is stronger. In fact, the bond may deteriorate over time. Sufficient information is not available to accurately predict the bond potential to dentin in every application. Bonding to enamel, however, is predictable and good. The attempt to simplify the bonding mechanism has resulted in less materials being involved and less decision making on the part of the operator—both in an effort to get more predictable results. However, the newer bonding systems have not yet been proven to be better.

Deterministic effects are those that _____. A. Show a severity of response proportional to dose B. Are seen only in the oral cavity C. Are found following exposure to low levels of radiation D. Result from particulate radiation such as alpha and beta particles, but not x-rays E. None of the above

A, Deterministic effects are those with dose thresholds, thus requiring at least moderate levels of exposure, and where the severity of response is proportional to dose

The primary function of developer is to _____. A. Reduce crystals of silver halide to solid silver grains B. Reduce solid silver grains to specks of silver halide C. Remove unexposed silver halide crystals D. Remove exposed silver halide crystals

A, Developer reduces silver bromide to solid silver grains.

You need to inject a local anesthetic for a 10-year-old patient. You note that this patient appears to be very anxious and frequently asks what you are doing or are about to do. To which technique is the patient least likely to respond well? A. Distraction. B. Taking a few deep breaths. C. Bringing his mother in the room for reassurance. D. Provide him with age-appropriate information about the injection. E. Give the patient an amount of time (how long it will take) and ask him to count.

A, Distraction is not typically a very effective technique for very anxious patients

When is distraction osteogenesis preferred over a traditional osteotomy? A. When a large advancement is needed. B. When a small advancement is needed. C. When exacted interdigitation of the occlusion is needed. D. When the treatment needs to be done in a very short period of time. E. Distraction osteogenesis is always preferred over a traditional osteotomy.

A, Distraction osteogenesis is preferred over traditional osteotomies when large skeletal movements are required, and the associated soft tissue cannot adapt to the acute changes and stretching that results. Larger movements may be at increased risk of some relapse. This is particularly true in a patient with a cleft palate, where there is significant soft tissue scarring from previous surgeries

Drug agonists having the same intrinsic activity also have the same _____. A. Maximal effect B. Potency C. Receptor affinity D. Therapeutic index E. Aqueous solubility

A, Drug agonists have an intrinsic activity of greater than 0 and less than or equal to 1. This refers to the maximal effect attainable by the drug. Potency and receptor affinity are not directly related to intrinsic activity. The therapeutic index (TI) requires a quantal dose-response curve, unlike the other characteristics listed which require graded concentration-response curves. Drugs with the same intrinsic activity may vary a great deal in their aqueous solubility

Advantages of fixed wire retention compared to a removable Hawley-type retainer include which of the following? A. Does not require the patient to remember to wear it. B. Is easier to clean. C. The design can be altered to achieve minor tooth movements. D. It can incorporate an acrylic bite plate to avoid relapse of overbite correction

A, Fixed retention requires no patient cooperation to achieve retention. However, fixed retainers are more difficult to clean and cannot be modified to move teeth or control overbite relapse

During an emergency dental visit in which a tooth is to be extracted due to extensive pulpal involvement, a moderately mentally challenged 5-year-old child becomes physically combative. The parents are unable to calm the child. What should the dentist do? A. Discuss the situation with the parents. B. Force the nitrous oxide nosepiece over the child's mouth and nose. C. Use the hand over mouth exercise (HOME). D. Use a firm voice control.

A, For any child patient, it is imperative to discuss any kind of physical restraint with the parent to obtain an informed consent. An informed consent includes recommended treatment, reasonable alternatives to that treatment, and the risk of no treatment. If the dentist wants to use a firm voice control, it is recommended that a discussion take place beforehand, as well

It is generally accepted that the maximum thickness of a composite increment that allows for proper cure is _____. A. 1-2 mm. B. 2-4 mm. C. 4-6 mm. D. There is no maximum thickness restriction

A, Generally, composite can be properly polymerized in 1- to 2-mm increments.

Which interleukin (IL) is important in the activation of osteoclasts and the stimulation of bone loss seen in periodontal disease? A. IL-1 B. IL-2 C. IL-8 D. IL-10

A, IL-1 is important in the activation of osteoclasts and stimulation of bone loss

The following are all desirable properties of an ideal local anesthetic, except _____. A. It should have potency sufficient to give complete anesthesia even if harmful results occur at therapeutic doses B. It should be relatively free from producing allergic reactions C. It should be stable in solution and readily undergo biotransformation in the body D. It should either be sterile or capable of being sterilized by heat without deterioration

A, Ideally, a local anesthetic should be relatively free from producing allergic reactions and it should be stable in solution and readily undergo biotransformation in the body. It is an absolute requirement that it should either be sterile or capable of being sterilized by heat without deterioration. If proper doses are used and are properly injected, there is a high success rate of obtaining anesthesia, while being able to minimize adverse effects

All of the following reasons are likely to indicate the need for restoration of a cervical notch except _____. A. Patient age. B. Esthetic concern. C. Tooth is symptomatic. D. Deeply notched axially.

A, If a patient has a notched cervical area that is very sensitive or very esthetically objectionable, restoration is usually indicated. If the notched area is very deep, adverse pulpal or gingival responses may occur. Although more notched areas are encountered in older patients, a patient's age is not a factor in the need for restoration.

All of the following reasons are likely to indicate the need for restoration of a cervical notch except _____. A. Patient age. B. Esthetic concern. C. Tooth is symptomatic. D. Tooth is deeply notched axially.

A, If a patient has a notched cervical area that is very sensitive or very esthetically objectionable, restoration is usually indicated. If the notched area is very deep, adverse pulpal or gingival responses may occur. Although more notched areas are encountered in older patients, a patient's age is not a factor in the need for restoration

Tooth #30 is endodontically treated after a conservative access cavity was made through a typical MO amalgam restoration. The restoration of choice is a _____. A. Chamber-retained amalgam foundation B. Custom cast post and core C. Wire post and core D. Parallel-sided prefabricated post with cast core

A, If there is an existing pulp chamber and remaining sound tooth structure, there is no need to place a post. Placement of a post tends to require taking additional tooth structure, which weakens a tooth

Which of the following is the most likely cause of an occlusal rest fracture? A. Inadequate rest-seat preparation B. Improper rest location C. Structural metal defects D. Occluding against the antagonist tooth

A, In McCracken's Removable Partial Prosthodontics, ed 11 (St Louis, Mosby, 2005), McCracken states, "Failure of an occlusal rest rarely results from a structural defect in the metal and rarely if ever is caused by distortion. Therefore the blame for such failure must often be assumed by the dentist for not having provided sufficient space for the rest during mouth preparations

Myofascial pain dysfunction may be described as _____. A. Masticatory pain and limited function B. Clicking and popping of the joint C. An infectious process D. Dislocation of the disc

A, In myofascial pain dysfunction the source of the pain and dysfunction is muscular. Here dysfunction is associated with decreased opening or inability to chew.

The most obvious clinical sign of trauma from occlusion is increased tooth mobility. The most obvious radiographic sign of trauma from occlusion is an increase in the width of the periodontal ligament space. A. Both statements are true. B. Both statements are false. C. The first statement is true, the second statement is false. D. The first statement is false, the second statement is true.

A, Increased tooth mobility is the most common clinical sign of trauma from occlusion. Increased periodontal ligament width is the most common radiographic sign

Local anesthetics act on what type of receptor? A. An ion channel receptor B. A nuclear receptor C. A 7-membrane domain receptor linked to Gs D. A 7-membrane domain receptor linked to Gq E. A membrane receptor with tyrosine kinase activity

A, Inhibiting sodium channels leads to the inhibition of the nerve action potential and inhibition of nerve conduction. Sodium channels are examples of ion channel receptors. Ion channel receptors contain several subunits arranged in a barrel shape. Drugs that bind to the channel can alter conductance to the ion associated with that channel

What receptor or signaling pathway is linked most directly to a2-adrenoceptor stimulation? A. Gi and a reduction in cAMP B. Gs and an increase in cAMP C. Gq and calcium D. Sodium ion channel E. Membrane receptor containing tyrosine kinase

A, Inhibition of adenylyl cyclase through Gi, resulting from stimulation of α2-adrenergic receptor, leads to a reduction in intracellular cAMP.

Congenitally missing teeth are the result of failure in which stage of development? A. Initiation B. Morphodifferentiation C. Apposition D. Calcification

A, Initiation and proliferation are the only possibilities for congenitally absent teeth, the bud and cap stages, respectively. In the histodifferentiation stage, the teeth are present; failure in this stage results in structural abnormalities of the enamel and dentin. Failure in the morphodifferentiation stage results in size and shape abnormalities

Which of the following is the main side effect of bleaching an endodontically treated tooth? A. External cervical resorption B. Demineralization of tooth structure C. Gingival inflammation

A, Internal bleaching alone causes 3.9% of external cervical root resorption (also referred to as peripheral inflammatory root resorption); The presence of a barrier (base material) between the root filling material and the internal bleaching material should be ~4 mm to prevent this resorption.

Reduction of overbite can be accomplished most readily by which of the following tooth movements? A. Intruding maxillary incisors B. Uprighting maxillary and mandibular incisors C. Using a high-pull headgear to the maxillary molars D. Using a lip bumper

A, Intruding incisors would decrease overbite while uprighting teeth and using a high-pull headgear could make overbite correction more difficult. A lip bumper would likely have little effect on overbite

Your patient is 9 years old. The mandibular left first primary molar has a large, carious lesion on the distal and on the occlusal and the tooth has greater mobility than what you would normally expect. You should _____. A. Take a radiograph of the area B. Perform a pulpotomy C. Perform a pulpectomy D. Extract the tooth and consider space maintenance

A, It is difficult to know which treatment is indicated without more information than is presented in the question. The tooth could be mobile due to furcation involvement, internal or external root resorption, exfoliation, or a combination of all the above. Obtaining more clinical information by taking a radiograph is necessary before any further treatment is rendered.

Elevation of cardiovascular signs with epinephrine, injected in a local anesthetic solution in a cardiovascularly compromised patient, occurs at about what threshold? A. 40 μg B. 100 μg C. 200 μg D. 1000 μg

A, Jastak and Yagiela have published data demonstrating that well-monitored, cardiovascularly compromised patients begin to show elevation of vital signs when more than about 40 μg (0.04 mg) of epinephrine is administered in the local anesthetic solution

The width of keratinized gingiva is measured as the distance from the _____. A. Free gingival margin to the mucogingival junction B. Cementoenamel junction to the mucogingival junction C. Free gingival groove to the mucogingival junction D. Free gingival margin to the base of the pocket

A, Keratinized gingiva extends from the free gingival margin to the mucogingival junction. The attached gingival extends from the free gingival groove to the mucogingival junction

Prolonged, unstimulated night pain suggests which of the following conditions of the pulp? A. Pulpal necrosis B. Mild hyperemia C. Reversible pulpitis D. Periodontal abscess

A, Lingering spontaneous pain is evidence of C-fiber stimulation. Even in degenerating pulps, C fibers may respond to stimulation. The excitability of C fibers is less affected by disruption of blood flow as compared with A fibers. C fibers are often able to function in hypoxic conditions (e.g., at the early stage of pulpal necrosis).

A 25-year-old patient presenting with generalized marginal gingivitis without any systemic problems or medications should be classified with which periodontal prognosis? A. Good B. Fair C. Poor D. Questionable

A, Marginal gingivitis not complicated by systemic problems or medications usually can be treated successfully with phase 1 therapy, and a patient with this diagnosis would have a good prognosis

For optimum esthetics when setting maxillary denture teeth, the incisal edges of the maxillary incisors should follow the _____. A. Lower lips during smiling B. Upper lips during smiling C. Lower lips when relaxed D. Upper lips when relaxed

A, Maxillary teeth should contact the wet dry lip line when fricative sounds f, v, and ph are made. These sounds help to determine the position of the incisal edges of the maxillary anterior teeth

Metamerism invariably involves _____. A. A color difference between two objects under one or more illuminant(s) B. One object having a lower chroma than another C. One object having a lower lightness than another D. A significant color change of one object as it moves from one illuminant to another

A, Metamerism is the phenomenon where a color match under a lighting condition appears different under a different lighting condition

Which pulpotomy medicament demonstrates better success rates than formocresol? A. Mineral trioxide aggregate B. Calcium hydroxide C. Resin-modified glass ionomer cement D. Fifth-generation bonding agents

A, Mineral trioxide aggregate (MTA) pulpotomies have shown very good promise and generally show higher success rates than formocresol pulpotomies. However, at this time MTA is very expensive and is not used as often as formocresol or ferric sulfate

The majority of injectable local anesthetics used today are _____. A. Tertiary amines B. Secondary amines C. Primary amines D. Esters

A, Most local anesthetics packaged in dental cartridges are tertiary amines. Currently, the only local anesthetic packaged in dental cartridges that has an ester bond is articaine but the bond in the connecting chain in the drug molecule is an amide

Which of the following is true regarding possible complications resulting from dental extractions? A. Patients with numbness lasting more than 4 weeks should be referred for microneurosurgical evaluation. B. Infections are common, even in healthy patients. C. Dry socket occurs in 10% of third molar patients. D. Teeth lost into the oropharynx are usually swallowed, and thus do not require further intervention

A, Most nerve injuries are transient; however, in an injury that lasts greater than 4 weeks, a surgical evaluation is indicated

A nasopalatine duct cyst is located between _____. A. Two maxillary central incisors B. Maxillary central and lateral incisors C. Maxillary lateral and canine D. Maxillary canine and first premolar

A, Nasopalatine duct cyst: a circular radiolucent area seen as a marked swelling in the region of the palatine papilla. It is situated mesial to the roots of the central incisors, at the site of the incisive foramen. The pulps of the anterior teeth test vital (whereas a periapical cyst tests nonvital). This is the most common type of maxillary developmental cyst. They often remain limited in size and are asymptomatic; they may become infected and show a tendency to grow extensively

Which is a nicotinic receptor? A. Receptor for the neurotransmitter at the skeletal-neuromuscular junction B. Receptor for the neurotransmitter at the junction between the postganglionic sympathetic nerve and sweat glands C. Receptor for the neurotransmitter at the junction between the postganglionic parasympathetic nerve and the parotid gland D. Receptor for the neurotransmitter at the junction between the postganglionic sympathetic nerve and blood vessels E. Receptor for the neurotransmitter at the junction between the postganglionic parasympathetic nerve and the heart

A, Nicotinic receptors are located at the skeletal-neuromuscular junction, ganglia, junction of the sympathetic nerve to the adrenal gland and the adrenal chromaffin cells, as well as in the central nervous system

Conservative surgical excision would be appropriate treatment and probably curative for which of the following? A. Nodular fasciitis B. Fibromatosis C. Fibrosarcoma D. Rhabdomyosarcoma E. Adenoid cystic carcinoma

A, Nodular fasciitis is a rapidly developing reactive lesion that typically does not recur following excision. Fibromatosis is an aggressive nonencapsulated lesion that has significant recurrence potential. The other lesions listed are malignancies and require more than simple excision to prevent recurrence

What is a nonrigid connector? A. An appliance composed of a key and keyway that is used to connect one piece of a prosthesis to another B. An appliance that is used to connect two crowns rigidly fixed C. A bar appliance that is used to maintain a space for a tooth that has not erupted D. None of the above

A, Nonrigid connectors are used when it is not possible to prepare two abutments for a fixed partial denture (FPD) with a common path of placement or to segment a large or complex FPD into shorter components. Nonrigid connectors can be prefabricated plastic patterns (female or keyway portion, and male or key portion) that are embedded in the waxed crown and pontic patterns or custom-milled in the cast crown. The second part is then custom-fitted to the milled retainer and cast

Orthodontic closure of a midline diastema in a patient with a heavy maxillary frenum _____. A. Is accomplished prior to the frenum surgery. B. Is accomplished after the frenum surgery. C. After orthodontic closure, frenum surgery is typically not indicated. D. After frenum surgery, orthodontic closure is typically not indicated.

A, Orthodontic closure of a midline diastema is accomplished prior to the periodontal surgery. If a frenectomy is performed prior to orthodontic treatment, it is possible that scar tissue could form in the area, which may impede orthodontic tooth movement.

Each of the following is a feature of papillary hyperplasia except one. Which one is not true? A. It is a proliferative bone disease B. It can be caused by wearing the dentures at night C. It can be caused by poor oral hygiene D. It can be caused by an ill-fitting denture

A, Paget's disease of bone is a bone disease characterized by bone resorption followed by attempts at bone repair involving proliferation leading to bone deformities. Its etiology is unknown and it occasionally involves the maxilla and mandible. Papillary hyperplasia is characterized by multiple papillary projections of the epithelium caused by local irritation, poor-fitting denture, poor oral hygiene, and leaving dentures in all day and night.

Patients experiencing stress and anxiety typically require _____ interpersonal distance for comfortable interaction. A. Greater B. Less C. The same as patients who are not experiencing stress and anxiety D. Individualized E. Behaviorally controlled

A, Patients who are experiencing stress and anxiety typically feel more comfortable in having greater interpersonal space than they normally would when not experiencing stress and anxiety

Research suggests that life events and perceived stress/distress _____ predictors of selfreported health concerns. A. Are B. Are not C. Are sometimes D. Have little to do with E. None of the above

A, Perceived stress and distress in one's life has been demonstrated to be a significant predictor (positively correlated) with self-reported health concerns.

Which of the following indices is not reversible? A. DMFT B. GI C. PI D. OHI-S E. None of the above

A, Periodontal disease (measured by the PI) and gingival disease, measured by the GI, are reversible processes. The amount of the debris and calculus, measured by the OHI-S, can decrease too. Caries is not a reversible process

Central and peripheral giant cell granulomas share which of the following features? A. Microscopic appearance B. Clinical behavior C. Recurrence rate D. Similar forms of treatment E. Radiographic appearance

A, Peripheral and central giant cell granulomas have very different clinical presentations and behaviors, but identical light microscopic features

What is the most important procedure to perform during the initial postoperative visits following periodontal surgery? A. Plaque removal B. Visual assessment of the soft tissue C. Periodontal probing D. Bleeding index

A, Plaque removal during the initial postoperative visits following periodontal surgery is essential to healing of the periodontal tissues

The _____ nerve block is recommended for management of several maxillary molar teeth in one quadrant. A. Posterior superior alveolar (PSA) B. Inferior alveolar (IA) C. Long buccal (LB) D. Nasopalatine (NP)

A, Posterior superior alveolar (PSA). This is the only injection listed that leads to pulpal anesthesia in the maxilla. The nasopalatine (NP) is a maxillary injection that leads to soft-tissue anesthesia of the premaxilla only. The inferior alveolar (IA) and long buccal (LB) are mandibular injections

Which of the following best describes the anesthetic effects of a posterior superior alveolar nerve block? A. Pulpal anesthesia of the maxillary second and third molars. B. Pulpal anesthesia of the maxillary first molar. C. Pulpal anesthesia of the maxillary first and second premolars. D. Pulpal anesthesia of the second premolar.

A, Pulpal anesthesia of the maxillary second and third molars. Posterior superior alveolar nerve block anesthetizes the entire second and third maxillary molars; the first maxillary molar fully anesthetized in about 70% of patients and partially anesthetized (except for mesiobuccal root) in about 30%. This block is highly effective but carries significant risk of hematoma, so frequent aspiration during injection is crucial.

A permanent incisor with a closed apex is traumatically intruded. What is the treatment of choice? A. Gradual orthodontic repositioning and calcium hydroxide pulpectomy B. Surgical repositioning and calcium hydroxide pulpectomy C. Gradual orthodontic repositioning and conventional endodontic therapy D. Surgical repositioning and conventional endodontic therapy

A, Rapid root resorption, pulp necrosis, and ankylosis are common sequelae to intruded permanent teeth with mature apices. Treatment includes: a. Gradual repositioning orthodontically (2-3 weeks) b. Stabilize for 2 to 4 weeks c. Calcium hydroxide pulpectomy 2 weeks after injury

Which of the following odontogenic cysts occurs as a result of stimulation and proliferation of the reduced enamel epithelium? A. Dentigerous cyst B. Lateral root cyst C. Radicular cyst D. Odontogenic keratocyst E. Gingival cyst

A, Reduced enamel epithelium that overlies the crown of an unerupted tooth may give rise to a cyst occurring in the same position. This is, by definition, a dentigerous cyst. The stimulus for cystic epithelial proliferation is unknown

A 7-year-old patient presents with a quadrant of teeth showing abnormal formation of both enamel and dentin. All of his other teeth appear clinically normal. Radiographically, the affected teeth can be described as "ghost teeth." He has _____. A. Regional odontodysplasia B. Dens evaginatus C. Dentin dysplasia D. Ectodermal dysplasia E. Cleidocranial dysplasia

A, Regional odontodysplasia is often called "ghost teeth" because of the thin layers of dentin and enamel produced. One quadrant of teeth is affected, and the teeth are nonfunctional

Which class of antihypertensive drug most effectively reduces the release of renin from the kidney? A. β-adrenergic receptor blockers B. ACE inhibitors C. α-adrenergic receptor blockers D. Calcium channel blockers E. Angiotensin II receptor blockers

A, Renin release from the kidney is enhanced by stimulation of the β1-adrenergic receptors in the juxtaglomerular cells. From the above list, only β blockers reduce renin release. Although angiotensin converting enzyme inhibitors and angiotensin II receptor blockers act on the renin-angiotensin system, they do not inhibit renin release. In fact, they tend to increase plasma renin.

Scalers are used to remove supragingival deposits. Curettes are used to remove either supragingival or subgingival deposits. A. Both statements are true. B. Both statements are false. C. First statement is true. Second statement is false. D. First statement is false. Second statement is true.

A, Scalers, with their pointed ends and back, are designed for supragingival instrumentation; curettes, with their rounded ends and back, can be used for both supragingival and subgingival instrumentation

The most effective method to prevent caries on the occlusal surfaces among school-age children is _____. A. Sealants B. Community water fluoridation C. School dietary fluoride D. School fluoride mouth rinse E. School fluoridation

A, Sealants. Community water fluoridation is the most cost-effective and economical method to prevent dental caries. However, fluoride is believed to be the least effective on the occlusal surface. Most decay among school children occurs on the chewing surfaces' pits and surfaces

A major difference between total-etch and selfetching primer dentin bonding systems include all of the following except _____. A. The time necessary to apply the material(s). B. The amount of smear layer removed. C. The bond strengths to enamel. D. The need for wet bonding.

A, Self-etch dentin bonding systems differ from total-etch dentin bonding systems by removing less of the smear layer (they use a less potent acid), creating a weaker bond to enamel (especially nonprepared enamel), and not requiring wet bonding that may be necessary for some of the total-etch systems. Even though fewer actual materials may be needed with some of the self-etch systems, they need to be applied in multiple coats and thus the time necessary to apply the materials is similar for both systems.

A major difference between total-etch and selfetching primer dentin bonding systems include all of the following except _____. A. The time necessary to apply the material(s) B. The amount of smear layer removed C. The bond strengths to enamel D. The need for wet bonding

A, Self-etch dentin bonding systems differ from totaletch dentin bonding systems by removing less of the smear layer (they use a less potent acid), creating a weaker bond to enamel (especially nonprepared enamel), and not requiring wet bonding which may be necessary for some of the total-etch systems. Even though fewer actual materials may be needed with some of the self-etch systems, they need to be applied in multiple coats and therefore the time necessary to apply the materials is similar for both systems

The functional inquiry questionnaire reveals that the mother has had negative dental experiences and remains very nervous regarding her dental care. How would this most likely influence her 3-year-old child's reaction to dentistry? A. Increase the likelihood of a negative behavior. B. Increase the likelihood of a positive response to dentistry. C. Will likely cause an initial positive reaction, which changes to a negative reaction with the slightest stress. D. Maternal anxiety has little effect on a child's behavior in a dental setting.

A, Studies show that there is a high correlation between maternal anxiety and a child's negative behavior in the dental office. This effect is greatest for children less than 4 years of age

What bacterial species are found in increased numbers in the apical portion of toothassociated attached plaque? A. Gram-negative rods B. Gram-positive rods C. Gram-positive cocci D. Gram-negative cocci

A, Subgingival plaque can be in the cervical area or more apical. In both areas it can be either tooth-associated or tissue-associated. The apical tooth-associated plaque is composed primarily of gram-negative rods

The free gingival graft technique can be used to increase the width of attached gingival tissue. Apically displaced full-thickness or partialthickness flaps can also be used to increase the width of attached gingiva. A. Both statements are true. B. Both statements are false. C. First statement is true. Second statement is false. D. First statement is false. Second statement is true

A, Surgical techniques designed to increase the width of attached gingiva include free gingival grafts and apically repositioned flaps

Balanced occlusion is less important during chewing than during nonchewing events. This difference occurs because the time teeth are in contact during nonchewing events is much greater than the time teeth are in contact during chewing. A. Both statements are true. B. The first statement is true, and the second statement is false. C. The second statement is true, and the first statement is false. D. Both statements are false.

A, Teeth come together every time a patient swallows. This can dislodge dentures due to breaking the denture seal.

Anticipating correct administration of the (long) buccal injection, what areas will be anesthetized? A. Soft tissues and periosteum buccal to the mandibular molar teeth B. Soft tissues and periosteum lingual to the mandibular molar teeth C. Soft tissues and periosteum lingual to the mandibular premolar teeth D. Soft tissues and periosteum buccal to the mandibular premolar teeth

A, The (long) buccal injection anesthetizes the soft tissues and periosteum buccal to the mandibular molar teeth

Which of the following federal agencies is the U.S. government's principal agency for protecting the health of all Americans and providing essential human services? A. DHHS B. NIH C. HRSA D. AHRQ E. None of the above

A, The DHHS is the U.S. government's principal agency for protecting the health of all Americans and providing essential human services. DHHS includes 11 agencies and more than 300 programs. The agencies listed in the answers are part of the DHHS. The National Institutes of Health (NIH) is the world's premier medical research organization. The Health Resources and Services Administration (HRSA) provides access to essential health care services for people who are low-income, uninsured, or who live in rural areas or urban neighborhoods where health care is scarce. The Agency for Healthcare Research and Quality (AHRQ) supports research on health care systems, health care quality and cost issues, access to health care, and effectiveness of medical treatments

Twisting a triangular wire best describes the manufacturing process of a _____. A. Reamer B. Barbed broach C. Hedström file D. K-Flex file

A, The K-file and K-reamer are the oldest instruments for cutting and machining dentin. They have been made from a steel wire that is ground to a tapered square or triangular cross section and then twisted to create either a file or a reamer. A file has more flutes per unit length than does a reamer. The K-FlexTM file is a modification of the shape of the K-file, with a noncutting tip design.

Occlusal radiographs are useful for all of the following except _____. A. For views of the TMJ B. For displaying large segments of the mandibular arch C. When the patient has limited opening D. When there are sialoliths in the floor of the mouth E. When there is buccal-lingual expansion of the mandible

A, The TMJ is much too far from the occlusal plane (the location of occlusal film) to be imaged with this technique. The other choices are all proper indications for using occlusal film.

Bonding of resins to dentin is best described as involving _____. A. Mechanical interlocking B. Ionic bonding C. Covalent bonding D. Van der Waals forces

A, The bond of adhesives to dentin (and enamel) is primarily a mechanical interlocking of the material within the dentin (or enamel). The etching causes some removal of the surface, creating irregularities or spaced collagen fibrils into which the adhesive enters. When polymerized, the adhesive is mechanically locked into the surface

What would be the effect of prior administration of a competitive drug antagonist on the concentration-response profile of a drug agonist on a graded concentration-response curve? (Assume that both drugs act at the same receptor.) A. The agonist curve would shift to the left. B. The agonist curve would shift to the right. C. The agonist curve would not change. D. The agonist curve would not shift but would reach a lower maximal effect than the curve with agonist alone. E. The agonist curve would both shift to the left and have a lower maximal effect

A, The characteristic response to a competitive antagonist is a parallel shift to the right of the agonist curve, with the two curves reaching the same maximal effect

Which of the following most likely applies to a cracked tooth? A. The direction of the crack usually extends mesiodistally. B. The direction of the crack usually extends faciolingually. C. Radiographic exam is the best way to detect it. D. A and C only. E. B and C only.

A, The direction of the crack usually extends mesiodistally. Cracks extend deep into the dentin and are usually propagated mesiallydistally in posterior teeth, often in the region of the marginal ridge. Dyes and transillumination are very helpful in the visualization of cracks. Unfortunately, it is often impossible to determine how extensive a crack is until the tooth is extracted.

A comparison of screen film/intensifying screen combinations with direct-exposure films reveals that screen film/intensifying screen combinations _____. A. Render less resolution B. Require more exposure C. Require special processing chemistry D. Are preferred for intraoral radiography

A, The dispersion of visible light from the crystals in the phosphor layer of the intensifying screen reduces image resolution compared to directexposure film.

For a dental hand instrument with a formula of 10-8.5-8, the number 10 refers to _____. A. The width of the blade, in tenths of a millimeter B. The primary cutting edge angle, in centigrades C. The blade length, in millimeters D. The blade angle, in centigrades

A, The first number is the width of the blade or primary cutting edge in tenths of a millimeter (0.1 mm). The second number of a four-number code indicates the primary cutting edge angle, measured from a line parallel to the long axis of the instrument handle in clockwise centigrades. The angle is expressed as a percent of 360 degrees. The instrument is positioned so that this number always exceeds 50. If the edge is locally perpendicular to the blade, then this number is normally omitted, resulting in a threenumber code. The third number (second number of a three-number code) indicates the blade length in millimeters. The fourth number (third number of a three-number code) indicates the blade angle, relative to the long axis of the handle in clockwise centigrade.

For a dental hand instrument with a formula of 10-8.5-8-14, the number 10 refers to _____. A. The width of the blade in tenths of a millimeter. B. The primary cutting edge angle in centigrades. C. The blade length in millimeters. D. The blade angle in centigrades.

A, The first number is the width of the blade or primary cutting edge in tenths of a millimeter (0.1 mm). The second number of a four-number code indicates the primary cutting edge angle, measured from a line parallel to the long axis of the instrument handle in clockwise centigrades. The angle is expressed as a percent of 360 degrees. The instrument is positioned so that this number always exceeds 50. If the edge is locally perpendicular to the blade, then this number is normally omitted, resulting in a three-number code. The third number (second number of a three-number code) indicates the blade length in millimeters. The fourth number (third number of a three-number code) indicates the blade angle, relative to the long axis of the handle in clockwise centigrades.

The incisive papilla provides a guide for the anteroposterior placement of maxillary anterior denture teeth. The labial surfaces of natural teeth are generally 8 to 10 mm anterior to this structure. A. Both statements are true. B. The first statement is true, and the second statement is false. C. The first statement is false, and the second statement is true. D. Both statements are false

A, The incisive papilla provides a guide for the anteroposterior position of the maxillary anterior teeth. The labial surfaces of the central incisors are usually 8 to 10 mm in front of the papilla. This distance varies depending of the amount of resorption of the residual ridge, the size of the teeth, and the labiolingual thickness of the alveolar process

Two cystic radiolucencies in the mandible of a 16-year-old boy were lined by thin, parakeratinized epithelium showing palisading of basal cells. All teeth were vital and the patient had no symptoms. This patient most likely has which of the following? A. Odontogenic keratocysts B. Periapical granulomas C. Periapical cysts D. Traumatic bone cysts E. Ossifying fibromas

A, The key to this question is the description of the cystic lining of thin, parakeratinized epithelium with basal cell palisading—typical of odontogenic keratocyst. Tooth vitality, lack of symptoms, and more than one lesion are also supportive.

The major factor determining whether aspiration can be reliably performed is _____. A. The needle gauge B. The needle length C. The injection performed D. The patient

A, The larger the lumen of the needle, the easier it will be to determine whether the needle is actually in a vessel. The needle length is irrelevant, as is the patient. The injection performed is relevant as to the frequency of obtaining a positive aspiration but not the reliability of the aspiration per se

Which of the following is the single most important predictor of clinical success of a cast post and core? A. Amount of remaining coronal tooth structure. B. Post length. C. Post diameter. D. Positive horizontal stop

A, The length, canal enlargement, and a finish line for the post are unimportant if there is no sound remaining coronal tooth structure to get a ferrule of the final restoration

All of the following statements about slotretained complex amalgams are true except _____. A. Slots should be 1.5 mm in depth. B. Slots should be 1 mm or more in length. C. Slots may be segmented or continuous. D. Slots should be placed at least 0.5 mm inside the DEJ.

A, The longer a slot, the better. They should be inside the DEJ and prepared with an inverted cone bur to a depth of 1 mm.

All of the following statements about slotretained complex amalgams are true except _____. A. Slots should be 1.5 mm in depth. B. Slots should be 1 mm or more in length. C. Slots may be segmented or continuous. D. Slots should be placed at least 0.5 mm inside the DEJ.

A, The longer a slot, the better. They should be inside the DEJ and prepared with an inverted cone bur to a depth of 1 mm.

The minor connector for a mandibular distal extension base should extend posteriorly about _____. A. Two-thirds the length of the edentulous ridge B. Half the length of the edentulous ridge C. One-third the length of the edentulous ridge D. As long as possible

A, The minor connector for the mandibular distal extension base should extend posteriorly about two thirds the length of the edentulous ridge; this adds strength to the denture base

An adult patient with a Class II molar relationship and a cephalometric ANB angle of 2 degrees has which type of malocclusion? A. Class II dental malocclusion B. Class II skeletal malocclusion C. Class I dental malocclusion D. Class II skeletal malocclusion

A, The molars are Class II but the skeletal relationship described by a normal ANB measurement is normal, so the malocclusion is dental in origin

An adult patient with a Class II molar relationship and a cephalometric ANB angle of 2 degrees has which type of malocclusion? A. Class II dental malocclusion B. Class II skeletal malocclusion C. Class I dental malocclusion D. Class II skeletal malocclusion

A, The molars are Class II, but the skeletal relationship described by the ANB (the A-P angular difference between the maxilla and mandible) measurement is normal, so the malocclusion is dental in origin.

When compared to the bisecting-angle technique, the advantages of the paralleling technique in endodontic radiology include all of the following except_____. A. A significant decrease in patient radiation B. A more accurate image of the tooth's dimensions C. That it is easier to reproduce radiographs at similar angles to assess healing after treatment D. The most accurate image of all the tooth's dimensions and its relationship to surrounding anatomic structures

A, The paralleling, not right-angle, technique is best for endodontics. The film is placed parallel to the long axis of the tooth and the beam placed at a right angle to the film. The technique allows for the most accurate and reproducible representation of tooth size

The distance from the CEJ to the base of the pocket is a measure of _____. A. Clinical attachment level B. Gingival recession C. Probing pocket depth D. Alveolar bone loss

A, The periodontal examination includes probing pocket depth (distance from the gingival margin to the base of the pocket) and clinical attachment level (distance from the CEJ to the base of the pocket). Both of these measures are made using a periodontal probe. Gingival recession can be measured as the distance from the CEJ to the free gingival margin. Alveolar bone loss is measured radiographically.

Which of the following is not appropriate treatment for an odontogenic abscess? A. Placing the patient on antibiotics and having them return when the swelling resolves B. Surgical removal of the source of the infection as early as possible C. Drainage of the abscess with placement of surgical drains D. Close observance of the patient during resolution of the infection E. Medical management of the patient to correct any compromised states that might exist

A, The primary principle of management of odontogenic infections is to perform surgical drainage and removal of the cause. Abscesses will not resolve on antibiotics alone and may progress even if the patient is on antibiotics

The denture base of a mandibular distal extension RPD should cover _____. A. The retromolar pads B. All undercut areas and engage them for retention C. The hamular notch D. The pterygomandibular raphe

A, The retromolar pad should always be covered for support of the mandibular denture base. The retromolar pads and the buccal shelf are considered primary areas of support for a mandibular distal extension removal partial denture or complete denture.

What is the approximate elimination half-time for penicillin V? A. 0.5 hour B. 2 hours C. 4 hours D. 8 hours E. 12 hours

A, The short elimination half-time for penicillin V is due to rapid excretion of penicillin in the urine. About 90% of this renal excretion is a result of active tubular transport, a rapid and efficient process. (Very little metabolism of penicillin occurs.)

When soldering a fixed partial denture, what is the effect of flux when heated on the area to be soldered? A. To remove oxides from the metal surface. B. To displace metal ions from the area. C. To change the composition of the alloy. D. To reduce the surface tension of the metal

A, The soldering flux used with gold alloys is usually borax glass (Na2B4O7), because of its affinity for copper oxides. Flux is applied to a metal surface to remove or prevent oxide formation. With an oxide-free surface, the solder wets the surface freely and spreads over the metal surface

The following biological test is used to check the effectiveness of the sterilization process. A. Spore test B. Total bacterial count test C. Aseptic test D. EPA test E. Disinfection test

A, The spore test is a biological monitor. The process consists of placing into the autoclave bacterial spores on strips or in envelopes along with a normal instrument load. If the autoclave is working properly, the autoclave reaches the temperature and pressure to kill the spores. Spore testing must be conducted weekly

You have placed a dental implant for replacement of tooth #9. Preoperatively you obtained a panoramic and a periapical film. During the surgery, you used a crestal incision, series of drills, and paralleling pins as necessary. Upon restoration of the crown, obtaining ideal esthetics is difficult because the implant is placed too close to the labial cortex, causing the restoration to appear overcontoured. Which of the techniques below could most adequately have prevented this problem? A. Using an anterior surgical template B. Obtaining preoperative tomograms of the alveolus C. Using a tissue punch technique D. Using a smaller size of implant

A, The surgical guide template is a critical factor for the placement of implant in the esthetic area.

Following the administration of a local anesthetic, most patients can be maintained in conscious sedation at _____. A. 20%-40% nitrous oxide B. 20%-40% oxygen C. 50% nitrous oxide D. 10% nitrous oxide

A, The total flow rate is 4 to 6 L/min for most children. The practitioner can check the bag and make adjustments if necessary. The maintenance dose of nitrous oxide during an operative procedure is typically about 30%. In other words, a standard maintenance dose would usually be 4 L oxygen and 2 L of nitrous oxide. Of course after a lengthy administration, it is wise to reduce the concentration due to tissue saturation and nausea.

Your patient is 7 years old and has a very large, carious lesion on tooth T. What radiological factors should be used in determining the best treatment of choice between pulpotomy and primary endodontics? A. Furcation involvement B. External root resorption C. Internal root resorption D. Two of the above E. All of the above

A, The treatment decision in this case should be made on the presence or absence of furcation involvement. Absence of furcation involvement generally indicates a vital pulp. Of course, it is necessary to have vital tissue to perform a pulpotomy. Presence of furcation involvement generally indicates progression to a nonvital pulp. If furcation involvement is present, a pulpectomy would be the treatment of choice in the absence of external or internal root resorption

Which is one of the purposes or characteristics of the postpalatal seal? A. Provide a seal against air being forced under the denture. B. Usually should extend posterior to the fovea palatinae. C. Improves the stability of the maxillary denture. D. It is carved deeper in the midpalatal suture area

A, The vibrating line is located by finding the pterygomaxillary (hamular) notches, and continues to the median line of the anterior part of the soft palate slightly anterior to the foveae palatinae. A V-shaped groove 1 to 1.5 mm deep and 1.5 mm broad at its base is carved into the cast at the vibrating line. The narrow and sharp bead will sink easily into the soft tissue to provide a seal against air being forced under the denture. Stability is resistance to movement toward the residual ridge. The post-dam improves retention, not stability. It is carved shallow in the midpalatal suture area. Stability is determined by the size, height, or shape of the ridge.

You are planning to replace a maxillary central incisor with a fixed prosthetic device (FPD). The edentulous space is slightly wider than the contralateral tooth. In order to achieve acceptable esthetics, you should ensure that _____. A. The line angles of the pontic are placed in the same relationship as the contralateral tooth B. The pontic should be made smoother than the contralateral tooth C. The pontic should have a higher value than the contralateral tooth D. The line angles should be shaped to converge incisally on the pontic

A, The width of an anterior tooth is usually identified by the mesiofacial and distofacial position of the line angles, the shape of the surface contour, and light reflection between these line angles. The contralateral tooth features should closely be duplicated in the pontic, and the space discrepancy can be compensated by modifying the shape of the proximal areas

Which of the following ADA's Principles of Ethics states that a dentist has a duty to respect the patient's right to self-determination and confidentiality? A. Patient Autonomy B. Nonmaleficence C. Beneficence D. Justice E. Veracity

A, There are five principles in the ADA Principles of Ethics: ● Patient Autonomy ("self-governance"). The dentist has a duty to respect the patient's rights to self-determination and confidentiality. ● Nonmaleficence ("do no harm"). The dentist has a duty to refrain from harming the patient. ● Beneficence ("do good"). The dentist has a duty to promote the patient's welfare. ● Justice ("fairness"). The dentist has a duty to treat people fairly. ● Veracity ("truthfulness"). The dentist has a duty to communicate truthfully.

While performing nonsurgical endodontic therapy you detect a ledge. What should you do? A. Use a smaller instrument and get by the ledge. B. Fill as far as you have reamed. C. Use a small, round bur and remove the ledge. D. Continue working gently with larger files to remove the ledge.

A, Use a smaller instrument and get by the ledge. Ledges can sometimes be bypassed; the canal coronal to the ledge must be sufficiently straightened to allow a file to operate effectively. This may be achieved by anticurvature filing (file away from the curve). Precurve the file severely at the tip and use it to probe gently past the ledge. Otherwise, clean to the ledge and fill it, but you must warn the patient of a poorer prognosis.

In HIV diagnosis, the Western blot assay is used to confirm the results of a positive ELISA test. Therefore, we can say that the Western blot test will confirm a _____. A. True-positive result B. True-negative result C. False-positive result D. False-negative result E. None of the above

A, Very specific tests are appropriate for confirming the existence of a disease. If the result of a highly specific test is positive, the disease is almost certain. High specificity is required in situations where the consequences of a false-positive diagnosis are serious or unduly alarming (e.g., HIV positivity).

Loss of tooth substance by mechanical wear is _____. A. Abrasion B. Attrition C. Erosion D. Abfraction

A, Wasting diseases of the teeth include erosion (corrosion; may be caused by acidic beverages), abrasion (caused by mechanical wear as with toothbrushing with abrasive dentifrice), attrition (due to functional contact with opposing teeth), and abfraction (flexure due to occlusal loading).

A 7-year-old child has a history of recurrent pain and discomfort in a second molar, which has a necrotic pulp. You present the treatment options to the parents. "There are several ways in which we can treat this problem. We could do a pulpectomy in which we . . . . We could do something called a pulpotomy, which involves. . . . We could apply a pulp cap which is . . . . We could remove the tooth. Or we could leave the tooth untreated for now and see how things go." You have phrased the options so that they are in what you believe to be the order of descending desirability and you have indicated that to the patient. Which option is most likely to be chosen by the parents? A. Pulpectomy B. Pulpotomy C. Pulp cap D. Extraction E. No treatment

A, When a number of alternatives are presented and the first on the list is more desirable, there is a tendency for individuals to select the first option and view the successive options as less desirable.

When performing a diagnostic occlusal adjustment on diagnostic casts, the mandibular cast should be mounted to the maxillary cast in an articulator using which of the following? A. A centric relation interocclusal record B. A hinge articulator C. A maximum intercuspation wax record D. A facebow transfer

A, When performing an occlusal adjustment, the goal is to make CR and MI to coincide. None of the other choices allows one to reliably mount the casts in CR or allows one to accurately perform this procedure

The 02 taper on hand K-files is _____. A. 0.2-mm increase in diameter per 1-mm increase in length B. 0.02-mm increase in diameter per 1-mm increase in length C. 0.2-mm increase in diameter per 2-mm increase in length D. 0.02-mm increase in diameter per 2-mm increase in length

B, 0.02-mm increase in diameter per 1-mm increase in length. Taper is the amount the file diameter increases each millimeter from the tip toward the handle. For a 0.02 taper file with 16- mm working surface, its diameter at the tip (D0) plus 0.32 mm (i.e., for a No. 8 file, it's 0.08 + 16 × 0.02 = 0.40) should be equal to D16

In an adult of normal size, penetration to a depth of _____ mm places the needle tip in the immediate vicinity of the foramina, through which the posterior superior alveolar (PSA) nerves enter the posterior surface of the maxilla. A. 10 B. 16 C. 20 D. 30

B, 16 mm. The proper depth of penetration for the PSA nerve can be said to be half the length (16 mm) of a long needle or three-fourths the length (15 mm) of a short dental needle. Penetration beyond 16 mm has a significantly higher incidence of positive aspiration and hematoma formation

25-gauge needles are preferred to smaller-diameter ones due to all of the following reasons except _____. A. Greater accuracy in needle insertion for 25-gauge needles B. Increased rate of needle breakage for 25-gauge needles C. Aspiration of blood is easier and more reliable through a larger lumen D. There is no difference in pain of insertion

B, 25-gauge needles have a much lower incidence of breakage versus any other needle size commonly used in dentistry, whereas 30-gauge needles have by far the worst record

A 1.0-ml volume of a 2% solution contains _____. A. 18 mg B. 20 mg C. 36 mg D. 54 mg

B, A 2% solution is 20 mg/mL. 1.0 mL of a 20 mg/mL solution is 20 mg.

A complete denture patient presents with angular cheilitis. A review of recent medical examination revealed that vitamin deficiency is not a factor. A possible predisposing factor is _____. A. Excessive vertical dimension of occlusion B. A closed or insufficient vertical dimension of occlusion C. Improper balance of the occlusion D. Poor contour of the denture base

B, A closed or insufficient vertical dimension of occlusion is thought to be one predisposing condition for angular cheilitis, which usually is associated with Candida albicans. Improperly balanced occlusion or poor contour of the denture base are not predisposing conditions for angular cheilitis

Features of focal sclerosing osteomyelitis often include: A. A nonvital pulp test. B. A history of recent restoration of the tooth in question. C. A radiolucent lesion which, in time, becomes radiopaque. D. None of the choices is true.

B, A history of recent restoration of the tooth in question. Focal sclerosing osteomyelitis (FSO) consists of a localized, usually uniform zone of increased radiopacity adjacent to the apex of a tooth that exhibits a thickened periodontal ligament space or an apical inflammatory lesion. The size of the lesions usually measure less than 1 cm in diameter. There is no radiolucent halo surrounding this type of lesion. The osteitis microscopically appears as a mass of dense sclerotic bone FSO is most often found in patients younger than 20 years of age, around the apices of mandibular teeth (most commonly molars) with large carious lesions and chronically inflamed pulps or with recent restorations. Most sources agree that the associated tooth may or may not be vital. Gender is not a predisposing factor. FSO can be asymptomatic or the patient can experience mild pain, depending on the cause. FSO is usually discovered upon radiographic analysis. It

The following are those properties deemed most desirable for a local anesthetic, except _____. A. It should not be irritating to the tissue to which it is applied B. It should cause a permanent alteration of nerve structure C. Its systemic toxicity should be low D. It must be effective regardless of whether it is injected into the tissue or applied locally to mucous membranes

B, A local anesthetic should not be irritating to the tissue to which it is applied, nor should it cause permanent alteration of nerve structure. Its systemic toxicity should be low. Finally, it must be effective regardless of whether it is injected into the tissue or applied locally to mucous membranes. If an agent causes permanent alteration of nerve structure, it would not be of benefit

Which of the following statements regarding caries risk assessment is correct? A. The presence of restorations is a good indicator of current caries activity. B. The presence of restorations is a good indicator of past caries activity. C. The presence of dental plaque is a good indicator of current caries activity. D. The presence of pit-and-fissure sealants is a good indicator of current caries activity

B, A restored tooth indicates potential past carious activity but not current activity. Plaque presence does not necessarily indicate caries presence and sealants are used for preventive purposes, not caries treatment

Which of the following statements regarding caries risk assessment is correct? A. The presence of restorations is a good indicator of current caries activity. B. The presence of restorations is a good indicator of past caries activity. C. The presence of dental plaque is a good indicator of current caries activity. D. The presence of pit-and-fissure sealants is a good indicator of current caries activity.

B, A restored tooth indicates potential past carious activity but not current activity. Plaque presence does not necessarily indicate caries presence and sealants are used for preventive purposes, not caries treatment

What is a second-order bend? A. A bend to position a tooth buccolingually B. A bend to provide angulation of a tooth in mesiodistal direction (tip) C. A bend to provide correct angulation of a tooth in labiolingual direction (torque) D. A bend to rotate a tooth

B, A second-order bend is placed to provide angulation of a tooth in the mesiodistal direction, also called tip. A first-order bend is placed in an archwire to position a tooth in the labiolingual direction (in-out bend) and/or to rotate a tooth as seen in the occlusal plane. A bend to provide angulation in the labio-lingual direction is called a third-order bend (torqueing bend).

Which drug is most selective as a glucocorticosteroid? A. Aldosterone B. Dexamethasone C. Fludrocortisone D. Hydrocortisone

B, Aldosterone and fludrocortisone are selective mineralocorticosteroids. Hydrocortisone has significant mineralocorticoid and glucocorticoid activity. Dexamethasone has very little mineralocorticoid activity

Your patient exhibits enamel hypoplasia near the incisal edges of all permanent incisors and cuspids, except for the maxillary lateral incisors, which appear normal. At what age would you suspect some kind of systemic problem? A. Prior to birth B. From birth to 1 year of age C. From 1 to 2 years of age D. From 2 to 3 years of age

B, All anterior permanent teeth begin calcification during the first 6 months, except for maxillary lateral incisors. The maxillary lateral incisor may be used as a key to timing; if this tooth is affected, the causative event is likely to have occurred at 1 year of age or older

What is the direct effect of local anesthetics on blood vessels in the area of injection? A. Constriction B. Dilation C. Sclerosis D. Thrombosis

B, All local anesthetics are vasodilators to some degree

Which of the following local anesthetics causes the least amount of vasodilation? A. Lidocaine B. Mepivacaine C. Bupivacaine D. Articaine

B, All local anesthetics cause some amount of vasodilation. Those packaged as plain drugs (i.e., without vasoconstrictor) cause less vasodilation than do those drugs that must be packaged with vasoconstrictor to have efficacy. Of the listed drugs, Mepivacaine is the only one packaged in dental cartridges without vasoconstrictor

In a patient who displays excessive maxillary incisor at rest, has an excessive lower face height, and has a deep overbite, which of the following would be the preferred method of overbite correction? A. Eruption of posterior teeth to rotate the mandible open B. Intrusion of maxillary incisors C. Intrusion of mandibular incisors D. Flaring of maxillary and mandibular incisors

B, All of the choices are possible solutions to correct a deep overbite. Erupting posterior teeth would increase the already excessively long lower face height, whereas intrusion of maxillary incisors would improve the excessive maxillary incisor show at rest.

Of the following, which is the least reliable way to predict the timing of the peak of the adolescent growth spurt for an individual? A. Plotting changes in height over time on a growth curve B. Following eruption timing of the dentition C. Taking a hand-wrist radiograph to assess skeletal development D. Observing changes in secondary sex characteristics

B, Although developmental indicators generally correlate well with each other, using dental age to predict timing of growth is the least reliable of the methods offered

Which of the following statements about an amalgam tooth/cavity preparation is true? A. The enamel cavosurface margin angle must be 90 degrees. B. The cavosurface margin should provide for a 90-degree amalgam margin. C. All prepared walls should converge externally. D. Retention form for Class Vs can be placed at the DEJ.

B, Although the amalgam margin must be 90 degrees, the enamel margin might not be 90 degrees, especially on the occlusal surface. Most walls converge occlusally, but many Class V amalgam preparations have walls that diverge externally. No retention form should be placed at the DEJ; otherwise, the adjacent enamel will be undermined and subject to fracture

Which of the following statements about an amalgam tooth/cavity preparation is true? A. The enamel cavosurface margin angle must be 90 degrees. B. The cavosurface margin should provide for a 90- degree amalgam margin. C. All prepared walls should converge externally. D. Retention form for Class Vs can be placed at the DEJ.

B, Although the amalgam margin must be 90 degrees, the enamel margin may not be 90 degrees, especially on the occlusal surface. Most walls converge occlusally, but many Class V amalgam preparations have walls that diverge externally. No retention form should be placed at the DEJ; otherwise, the adjacent enamel will be undermined and subject to fracture.

When carving a Class I amalgam restoration, which statement is false? A. Carving may be made easier by waiting 1 or 2 minutes after condensation before it is started. B. The blade of the discoid carver should move parallel to the margins resting totally on the partially set amalgam. C. Do not carve deep occlusal anatomy. D. The carved amalgam outline should coincide with the cavosurface margins.

B, Amalgam carving should result in coincidence with the cavosurface margin and should not result in deep occlusal anatomy because such form may create acute amalgam angles that are subject to fracture. Depending on the condensation rate of the amalgam used, waiting a couple of minutes prior to initiating carving may allow the amalgam to harden enough that the carving will be easier and overcarving will be minimized. When carving the occlusal cavosurface margin, the discoid carver should rest on the adjacent unprepared enamel, which will serve as a guide for proper removal of amalgam back to the margin.

When carving a Class I amalgam restoration, which statement is false? A. Carving may be made easier by waiting 1 or 2 minutes after condensation before it is started. B. The blade of the discoid carver should move parallel to the margins resting on the partially set amalgam. C. Do not carve deep occlusal anatomy. D. The carved amalgam outline should coincide with the cavosurface margins

B, Amalgam carving should result in coincidence with the cavosurface margin and should not result in deep occlusal anatomy because such form may create acute amalgam angles that are subject to fracture. Depending on the condensation rate of the amalgam used, waiting a couple of minutes prior to initiating carving may allow the amalgam to harden enough that the carving will be easier and overcarving will be minimized. When carving the occlusal cavosurface margin, the discoid carver should rest on the adjacent unprepared enamel, which will serve as a guide for proper removal of amalgam back to the margin.

A permanent incisor with an open apex is extruded 4 mm following an injury 15 minutes ago. What is the treatment of choice? A. No immediate treatment, monitor closely for vitality. B. Reposition, splint, monitor closely for vitality. C. Reposition, splint, initiate calcium hydroxide pulpotomy. D. Reposition, splint, initiate calcium hydroxide pulpectomy.

B, An extruded permanent incisor with an open apex should be repositioned, splinted, and monitored closely for loss of vitality. Because of the open apex, the tooth may remain vital and continue development; therefore, immediate pulp treatment is contraindicated

On a panoramic radiograph of a 13-year-old patient, there is evidence of crown formation of the third molars but no root formation yet. These teeth fall into the category of impacted teeth. A. True B. False

B, An impacted tooth is one that fails to erupt into the dental arch within the expected time. Consequently the third molar in a 13-year-old patient would be classified as unerupted or in the process of erupting

An interdental crater has how many walls? A. One wall B. Two walls C. Three walls D. Four walls

B, An interdental crater has two bony walls remaining. These walls are usually the facial and lingual walls.

A good landmark for the anteroposterior positioning of the anterior maxillary teeth in a complete denture is the _____. A. Residual ridge B. Incisive papilla C. Incisal foramen D. Mandibular wax rim

B, Anatomic guidelines to be used as guides in arranging the anterior teeth are the incisive papilla, the midsagittal suture, and the ala of the nose (canine lines). The incisive papilla is a good guide for the anteroposterior positioning of the maxillary anterior teeth. The labial surfaces of the central incisors are usually 8 to 10 mm in front of the papillae. This distance varies depending on the size of the teeth and the labiolingual thickness of the alveolar process, so it is not an absolute relationship

Informed consent requires that the patient be advised of the following except for which one? A. The benefits of endodontic treatment B. The cost of endodontic treatment C. The risks of endodontic treatment

B, Any notion of moral decision making assumes that rational agents are involved in making informed and voluntary decisions. In health care decisions, our respect for the autonomy of the patient would, in common parlance, mean that the patient has the capacity to act intentionally, with understanding, and without controlling influences that would mitigate against a free and voluntary act. It implies knowledge and understanding of the risks and benefits to treatment. This principle is the basis for the practice of "informed consent" in the physician-patient transaction regarding health care.

Which perforation location has the best prognosis? A. Coronal third of root B. Apical third of root C. Chamber floor D. Middle third of root

B, Apical third of root. Apical perforations occur through the apical foramen or the body of the root (a perforated new canal). In general, the more subcrestally located the lesion, the better the prognosis. However, all perforations have an inherently worsened prognosis.

Some dental plans allow the dentist to charge the patient any difference between what the plan agrees to pay and the dentist's UCR (usual, customary, reasonable) fees. This arrangement is called _____. A. Payment differential B. Balance billing C. Prospective reimbursement D. Managed care E. None of the above

B, Balance billing. Prospective reimbursement is a mechanism in which the dentist is compensated before treatment is provided (i.e., in capitation systems). Managed Care is an arrangement in which a third party mediates between providers and patients negotiating reimbursement for certain services and overseeing the treatments delivered.

The most radiosensitive of the following cells in terms of cell killing is the _____. A. Salivary gland acinar cell B. Basal epithelial cell C. Endothelial cell D. Neuron. E. Polymorphonuclear leukocyte

B, Basal epithelial cells are the most mitotically active of the cells on the list, and thus are the most radiosensitive

A young permanent incisor with an open apex has a pinpoint exposure due to a traumatic injury that occurred 24 hours previously. The best treatment is _____. A. Place calcium hydroxide on the pinpoint exposure B. Open the pulp chamber to find healthy pulp tissue and perform a pulpotomy C. Initiate a calcium hydroxide pulpectomy D. Initiate conventional root canal treatment with gutta-percha

B, Because the exposure site is likely significantly contaminated from the injury that occurred 24 hours previously, direct pulp capping with calcium hydroxide is contraindicated. A calcium hydroxide pulpectomy should not be the automatic procedure accomplished because continued root elongation and closure of the pulp canal will likely not occur. A calcium hydroxide pulpotomy is preferable for a traumatized tooth with an open apex with either a large exposure or a small exposure of several hours or days postinjury. Clinically, the tooth should be anesthetized and, under sterile conditions, and the clinician should open the pulp chamber in search of healthy pulp tissue. It is likely that vital tissue will be present within 24 hours of the injury.

Which one of the following cannot be observed on a conventional radiograph? A. Canal calcification of tooth #15. B. Buccal curvature of the mesial root of tooth #30. C. Type of canals of tooth #21. D. Open apex of tooth #8.

B, Buccal curvature of the mesial root of tooth #30. Radiographs provide a two-dimensional, mesialdistal view of a tooth. The buccal-lingual aspect of a tooth cannot be fully appreciated. Curvatures buccal or lingual are often not appreciated. Canal calcifications can be seen as relatively radiopaque obliterations of the pulp chamber and canal space. Tooth #21 is a mandibular premolar. The radiograph can give telling clues as to the anatomy, be it one or two canals. A canal that suddenly disappears midroot or appears off-center is often indicative of two canals. Open apices are often clearly visualized with radiographs.

Which of the following local anesthetics is marketed for dentistry in the United States in more than one concentration? A. Bupivacaine B. Mepivacaine C. Lidocaine D. Articaine

B, Bupivacaine is only packaged in dental cartridges as a 0.5% solution. Likewise, lidocaine is always a 2% solution (in the United States) and articaine is always a 4% solution. Mepivacaine is packaged in both 2% and 3% solutions (in the United States)

Which of the following medications often result in overgrowth of gingival tissues? A. Penicillin, calcium channel blockers, phenytoin B. Calcium channel blockers, phenytoin, and cyclosporin C. Cyclosporin, penicillin, and cephalosporins D. Ampicillin, tetracycline, and erythromycin

B, Calcium channel blockers, cyclosporin, and phenytoin often result in overgrowth of gingival tissues.

Calculus is detrimental to the gingival tissues because it is _____. A. A mechanical irritant B. Covered with bacterial plaque C. Composed of calcium and phosphorous D. Locked into surface irregularities

B, Calculus is calcified dental plaque. It is always covered by a layer of uncalcified plaque, which is detrimental to the gingival tissues

Which drug used in the therapy for Parkinsonism does not cross the blood-brain barrier? A. Amantadine B. Carbidopa C. L-dopa D. Selegiline E. Tolcapone

B, Carbidopa is used to inhibit dopa decarboxylase. Its usefulness is based on reducing conversion of L-dopa to dopamine outside the central nervous system. Carbidopa does not penetrate the blood-brain barrier and therefore does not interfere with the beneficial effect of L-dopa in the brain, but prevents the adverse effects of dopamine in the periphery.

Which of the following are cells of the innate immune system? a. Neutrophils and monocytes/macrophages b. T cells and B cells c. Mast cells and dendritic cells d. Plasma cells A. a and b B. a and c C. b and d D. b and c

B, Cells of the innate immune system include neutrophils, monocytes/macrophages, mast cells, and dendritic cells. Cells of the specific (adaptive) immune system include T cells, B cells, and plasma cells.

Chroma is that aspect of color that indicates _____. A. The degree of translucency B. The degree of saturation of the hue C. Combined effect of hue and value D. How dark or light is a shade

B, Chroma is the saturation or intensity of the color or shade. Value is the relative lightness or darkness of a color. Opalescence is the light effect of a translucent material.

When Class III elastics are used, the maxillary first molars will _____. A. Move distally and intrude B. Move mesially and extrude C. Move mesially and intrude D. Move only mesially; there will be no movement in the vertical direction

B, Class III elastics are worn from the maxillary first molars to the mandibular canines. The force system created by Class III elastics will produce mesial movement and extrusion of the maxillary first molars.

Which technique is typically not useful in treating the anxious patient? A. Using less structure in establishing rapport. B. Reassuring the patient by telling the patient not to worry. C. Providing reasons before asking for sensitive information. D. Using empathy. E. Making expectations clear

B, Clinicians should use caution in providing premature reassurance because, if the outcomes are inconsistent with what the clinician asserted, trust and rapport may be compromised

When does a fixed dental prosthesis (FDP), which was cast in one piece, need to be sectioned? A. When a cantilever pontic is used. B. When the fit cannot be achieved or verified with a one-piece cast. C. When single crowns are adjacent to the FDP. D. Always, in order to achieve a good fit.

B, Common reasons for a FDP not to fit in one piece are lack of parallelism between the abutments and distortion of the wax pattern during removal from the dies. In any of these cases, the framework may not fit in the prepared abutment teeth and must be sectioned between one of the connectors between the pontic and retainer to fit the two pieces individually, and a solder record must be made to solder the pieces

In comparison to amalgam restoration, composite restorations are _____. A. Stronger. B. More technique-sensitive. C. More resistant to occlusal forces. D. Not indicated for Class II restorations

B, Composite restorations are more techniquesensitive than amalgam restorations because the bonding process is very specific (requiring exact, correct usage of the various materials and an isolated, noncontaminated field), and the insertion and contouring of composites are more demanding and time-consuming. Composites are not stronger than amalgam and have similar wear resistance compared to amalgams. Composites are indicated for Class II restorationB

In comparison to amalgam restorations, composite restorations are _____. A. Stronger B. More technique-sensitive C. More resistant to occlusal forces D. Not indicated for Class II restorations

B, Composite restorations are more techniquesensitive than amalgam restorations because the bonding process is very specific (requiring exact, correct usage of the various materials and an isolated, noncontaminated field), and the insertion and contouring of composites are more demanding and time-consuming. Composites are not stronger than amalgam and have similar wear resistance compared to amalgams. Composites are indicated for Class II restorations

From the list of classifications of impacted teeth below, which one(s) must always involve both bone removal and sectioning during the surgical procedure? A. Mesioangular impaction B. Horizontal impaction C. Vertical impaction D. A and B only E. A, B, and C

B, Depending on the ramus relationship the mesioangular and vertical impactions may not require removal of bone or sectioning of the tooth. The horizontal impaction will always require removal of bone and sectioning

Which of the following is true regarding temporomandibular disorders? A. The primary treatment for the majority of patients with facial pain is TMJ surgery. B. Disc displacement without reduction can cause a decrease in interincisal opening. C. Myofascial pain is commonly related to parafunctional habits, but not commonly related to stress. D. Systemic arthritic conditions do not affect the TMJ because it is not a weight-bearing joint

B, Disc displacement without reduction can result in decreased range of motion because the condyle becomes restricted by the anteriorly displaced disc, limiting translation

Aqueous EDTA is primarily used to _____. A. Dissolve organic matter B. Dissolve inorganic matter C. Kill bacteria D. Prevent sealer from extruding out of the canal space

B, EDTA is the chelating solution customarily used in endodontic treatment. Chelators remove inorganic components, leaving the organic tissue elements intact.

Which of the following diagnostic criteria is least reliable in the assessment of the pulpal status of the primary dentition? A. Swelling B. Electric pulp test C. Spontaneous pain D. Internal resorption

B, Electric pulp test. The relatively late appearance of A fibers in the pulp may help to explain why the electric pulp test tends to be unreliable in young teeth, since A fibers are more easily electrically stimulated than C fibers. Accuracy of pulp testing also depends on the patient's ability to describe how the tooth reacts to stimuli. Clinicians must rely on experience, radiographs, clinical signs or symptoms, and their knowledge of the healing process to assess pulp vitality of young patients.

What is the mechanism of action of enoxaparin? A. Inhibition of synthesis of clotting factors II, VII, IX, and X B. Activation of antithrombin III with resulting inhibition of clotting factor Xa C. Indirect activation of tissue plasminogen activator D. Direct inhibition of plasminogen with resulting degradation of fibrin E. Dilation of coronary blood vessels

B, Enoxaparin is a low-molecular-weight heparin. It activates antithrombin III and inhibits factor Xa.

How many days does it usually take for surface epithelialization to be complete following a gingivectomy? A. 3-7 B. 5-14 C. 14-18 D. 20-27

B, Epithelial cells migrate approximately 0.5 mm/day. Following a gingivectomy, it takes 5 to 14 days for surface epithelialization to be complete.

Which of the following conditions can be caused in an edentulous patient by an ill-fitting denture flange? A. Papillary hyperplasia B. Epulis fissuratum C. Candidiasis D. Fibrous tuberosity

B, Epulis fissuratum is a reactive growth to an overextended or ill-fitting denture flange. It is best removed surgically. Papillary hyperplasia is found in the palatal vault. It is caused by local irritation, poor-fitting dentures, poor oral hygiene, or leaving dentures in 24 hours a day. Candidiasis is associated with papillary hyperplasia. Fibrous tuberosity is commonly seen with large tuberosities.

A 6-year-old patient likes to tell you stories about school. Each time he begins a story, you stop working to listen. After three long sessions, you realize that the child is attempting to avoid or delay the dental work by telling stories. You decide that from this point on you are going to continue working while engaged in conversation with the patient. At first, the child tells you more stories about school and, further, tries other strategies to get your attention and stop your work. He eventually settles down and allows you to work, whether or not you are engaged in conversation. This is an example of _____. A. Shaping B. Extinction C. Modeling D. Stimulus control E. Power

B, Extinction is the process of identifying all positive reinforcements (in this case, the dentist ceasing work on the child's teeth) that maintain a behavior and ceasing or withholding these

Which statement is false regarding motivation? A. Motivation is strengthened when a person succeeds and is weakened when a person fails to achieve his/her goals. B. Motivation is increased when the patient focuses on long-term goals. C. Motivating a patient can be achieved by generating interest, showing your concern, and providing information. D. Encourage a sense of personal acceptance in the face of the inevitable difficulties involved in breaking old habits and establishing new ones. E. Help a patient cope with relapses by emphasizing the knowledge gained

B, Focusing on long-term goals is not only a poor motivator, it is often a pitfall in the effort to change behavior, as patients are less motivated when goals seem too big, impossible, or far from their current circumstances.

The globulomaxillary lesion of bone _____. A. Is associated with the crown of an unerupted tooth B. Occurs between maxillary lateral and canine teeth C. Typically causes pain D. Typically presents as a mixed lucent-opaque lesion with ill-defined margins E. Is always associated with a nonvital tooth

B, Globulomaxillary lesion is a clinical term used to designate any lucency that occurs between the maxillary lateral incisor and canine

Stimulation of gluconeogenesis and lipolysis are most characteristic of which hormone? A. Calcitonin B. Cortisol C. Insulin D. Parathyroid hormone E. Progesterone

B, Glucocorticoids characteristically stimulate gluconeogenesis and lipolysis. Insulin has the opposite effects. The other hormones have minor or negligible effects

Which of the following properties of a gold alloy exceeds a base metal alloy in numerical value? A. Hardness B. Specific gravity C. Casting shrinkage D. Fusion temperature

B, Gold alloys are heavier for a given volume. Gold alloys are softer. Base metals are cast at higher temperatures, leading to greater shrinkage

A patient has difficulty inhibiting the gag reflex during x-ray procedures. You suggest that the patient take several x-ray packets home and practice holding the packets in his or her mouth for increasingly longer periods of time. Which of the following techniques does this best exemplify? A. Reinforcement B. Graded exposure C. Modeling D. Behavioral control E. Systematic desensitization

B, Graded exposure is the systematic process of exposing the patient to a hierarchy of increasingly anxiety-provoking stimuli

Which drug poses the greatest risk of a cardiac arrhythmia when administered at the same time as epinephrine? A. Desflurane B. Halothane C. Isoflurane D. Propofol E. Sevoflurane

B, Halothane sensitizes the heart to epinephrine and other catecholamines

In tooth movement, the formation of a hyalinized zone on the pressure side is due to _____. A. The application of light, continuous forces B. The application of heavy forces C. The normal forces of mastication D. Abnormal swallowing patterns

B, Heavy forces cause compression of the PDL with hyalinization

Which of the following statements best describes treatment options for a separated instrument (e.g., finger spreader) at the filling stage of treatment? A. Immediately attempt to remove the instrument. B. Do not attempt removal and proceed to obturate. C. Attempt to bypass the obstructed instrument. D. Both A and C are options

B, If an instrument is broken at the filling stage, it is not necessary to remove or bypass the instrument because the canal has already been cleaned and shaped. Prognosis depends largely on the extent of undebrided material remaining within the canal. Attempt to obturate as much of the canal as possible

If your radiographs start coming out too light, it may be that _____. A. Your exposure time is too long B. Your developer needs changing C. Your developer is too hot D. The fixer needs changing E. The films are not sufficiently washed

B, If proper processing procedures are followed, the developer will become depleted with age and need changing.

Inadequate margins of restorations should be corrected primarily because they _____. A. Cause occlusal disharmony B. Interfere with plaque removal C. Create mechanical irritation D. Release toxic substances

B, Inadequate or overhanging margins serve as a nidus for dental plaque accumulation and make plaque removal difficult

Which of the following statements is not true regarding internal root resorption? A. It happens rarely in permanent teeth. B. It appears as an asymmetrical "moth-eaten" lesion in radiographs. C. Chronic pulpal inflammation is the primary cause. D. Prompt endodontic therapy will stop the process.

B, Internal resorption is most commonly identified during routine radiographic examination. Histologically, it appears with chronic pulpitis, including chronic inflammatory cells, multinucleated giant cells adjacent to granulation tissue, and necrotic pulp coronal to resorptive defect. Only prompt endodontic therapy will stop the process and prevent further tooth destruction

The impression material that is mainly composed of sodium or potassium salts of alginic acid is _____. A. Polyether B. Irreversible hydrocolloid C. Polyvinyl siloxane D. Polysulfide

B, Irreversible hydrocolloid (IH) or alginate is the material of choice to produce diagnostic casts. Its composition is mainly sodium or potassium salts of alginic acid. They react chemically with calcium sulfate to produce insoluble calcium alginate

A dentist replaces an amalgam on tooth #5 and notices a small pulpal exposure. He elects to use a direct pulp cap procedure. Which of the following best predicts success? A. Size of the lesion B. Isolation of the lesion C. Use of calcium hydroxide D. Age of the patient

B, Isolation is the most important factor since it prevents bacterial contamination, increasing the success of the pulp cap procedure

A portion of which cranial nerve is anesthetized when performing an infraorbital nerve block? A. VII B. V C. III D. II

B, It is the intent with all intraoral injections of local anesthesia that you anesthetize a portion of the fifth cranial nerve. With an improperly placed needle in a mandibular block, it is possible to inadvertently anesthetize a portion of the seventh cranial nerve, and it is possible to inadvertently anesthetize the sixth cranial nerve with certain second-division nerve blocks

Localized aggressive periodontitis in the primary dentition is seen most commonly in the primary molar area. It is most common in Asian children. A. The first statement is true and the second statement is true. B. The first statement is true and the second statement is false. C. The first statement is false and the second statement is true. D. The first statement is false and the second statement is false.

B, Localized aggressive periodontitis in the primary dentition, previously known as localized prepubertal periodontitis (LPP) is most common in the primary molar area and occurs most commonly in African-American children. Treatment includes debridement and antibiotic therapy

According to Malamed, slow injection is defined as the deposition of 1 ml of local anesthetic solution in not less than _____. A. 15 seconds B. 30 seconds C. 60 seconds D. 2 minutes

B, Malamed recommends that one cartridge of local anesthetic be delivered over not less than 1 minute. Therefore, 1 mL (one-half cartridge) should be delivered over not less than one-half minute (30 seconds).

On the health history form, the mother of a 6-year-old new patient notes that the child is moderately mentally challenged. The dentist should _____. A. Refer to a pediatric dentist B. Use a Tell-Show-Do technique of behavior management C. Use conscious sedation D. Use restraints after obtaining informed consent

B, Many mentally challenged individuals can be mainstreamed and treated as any other patient. Because a moderately challenged 6-year-old may function as a preschool child, the normal management techniques are likely applicable. The correct answer for such a question will include some kind of normalization response

Which of the following are the most important proteinases involved in destruction of the periodontal tissues? A. Hylauronidase B. Matrix metalloproteinases C. Glucuronidase D. Serine proteinases

B, Matrix metalloproteinases are the most important proteinases involved in the destruction of periodontal tissues

Most natal and neonatal teeth are primary teeth. They should be extracted. A. The first statement is true and the second statement is true. B. The first statement is true and the second statement is false. C. The first statement is false and the second statement is true. D. The first statement is false and the second statement is false.

B, Most natal and neonatal teeth are primary teeth (90%); very few are supernumerary teeth (10%). Most are mandibular incisors (85%). Extraction of primary teeth should be accomplished only if they are extremely mobile and there is danger of aspiration. Most commonly, natal and neonatal teeth are left in position

Which statement is false regarding the relationship between pain and fear? A. Fear initially inhibits pain due to a release of endorphins from the pituitary, resulting in an analgesic effect. B. Although muscle tension contributes to the experience of anxiety, it does not contribute to the perception of pain. C. Any autonomic activation causes one to have a lower pain threshold. D. Catastrophic thinking and a perceived lack of control are common factors that influence pain perceptions. E. Misattribution occurs when patients identify an event as painful because they can identify a fearful stimulus

B, Muscle tension is associated with the experience of anxiety. Heightened anxiety contributes to lower pain thresholds/sensitivity to the perception of pain

What makes it possible for nickel-titanium archwires to exhibit superelastic behavior? A. This behavior is based on a reversible transformation within the austenitic phase. B. This behavior is based on a reversible transformation between the austenitic and martensitic phases. C. This behavior is based on a reversible transformation within the martensitic phase. D. This behavior is based on an irreversible transformation within the martensitic phase.

B, Nickel-titanium archwires can exist in more than one phase: austenitic and martensitic phases. Superelastic behavior of these wires is attributed to the reversible transformation between these two phases.

A 32-year-old male patient presented with a 1 ¥ 2-cm macular red-blue lesion in his hard palate. The lesion was asymptomatic and had been present for an unknown duration. He had no dental abnormalities and no significant periodontal disease. This could be all the following except _____. A. Vascular malformation B. Nicotine stomatitis C. Ecchymosis D. Kaposi's sarcoma E. Erythroplasia

B, Nicotine stomatitis appears as opacification of the palate, with red dots representing inflamed salivary ducts

Nitrates and nitrites have what effect on blood vessel smooth muscle? A. Increase in the level of intracellular calcium B. Increase in the level of cyclic guanosine monophosphate (cGMP) C. Antagonism at α1-adrenergic receptors D. Antagonism at β-adrenergic receptors E. Inhibition of L-type calcium channels

B, Nitroglycerin is a nitrovasodilator. It produces nitric oxide, which activates guanylyl cyclase which, in turn, catalyzes the production of cGMP.

A very low blood:gas solubility coefficient (partition coefficient = 0.47), analgesic effect, and a drug that inhibits methionine synthase best describes which drug? A. Ketamine B. Nitrous oxide C. Halothane D. Isoflurane E. Propofol

B, Nitrous oxide oxidizes the cobalt in vitamin B12, resulting in the inhibition of methionine synthase. Nitrous oxide has greater analgesic potency than other inhaled anesthetics (e.g., halothane, isoflurane). Ketamine is not inhaled; rather, it is injected. It also does not inhibit methionine synthase. The same is true for propofol.

In order for an alloy to be considered noble metal, it should _____. A. Contain at least 25% Ag B. Contain at least 25% Pt or Pd C. Contain 40% Au D. Contain at least 80% gold

B, Noble metals are gold (Au), platinum (Pt), and palladium (Pd) [silver (Ag) is not considered noble; it is reactive, but improves castability]. Noble alloys (old term was semiprecious metal) have a noble metal content ≥25%. (To be classified as noble, Pd-Cu, Pd-Ag, Pd-Co alloys have no stipulation for gold.) High noble alloys have a high content of gold (more than 60%).

Obstructive sleep apnea syndrome (OSAS) often results in all of the following except _____. A. Excessive daytime sleepiness B. Aggressive behavior C. Personality changes D. Depression

B, OSAS may result in mood disorders, daytime fatigue, and personality changes. Aggressive behavior is not considered a sequela of OSAS

Many factor affect tooth/cavity preparation. Which of the following would be the least important factor? A. Extent of the defect B. Size of the tooth C. Fracture lines D. Extent of the old material

B, Obviously, a tooth preparation is dictated by the extent of the carious lesion or old restorative material, the creation of appropriate convenience form for access and vision, and the anticipated extensions necessary to provide an appropriate proximal contact relationship. Fracture lines present should normally be included the restoration. However, it is rare that the size of the tooth will affect the design of the tooth preparation

Many factors affect tooth/cavity preparation. Which of the following would be the least important factor? A. Extent of the defect. B. Size of the tooth. C. Fracture lines. D. Extent of the old material

B, Obviously, a tooth preparation is dictated by the extent of the carious lesion or old restorative material, the creation of appropriate convenience form for access and vision, and the anticipated extensions necessary to provide an appropriate proximal contact relationship. Fracture lines present should normally be included in the restoration. However, it is rare that the size of the tooth will affect the design of the tooth preparation

Sarah S. is a young child who consistently presents as anxious, hypervigilant, and upset during dental visits. Sarah is often accompanied by her parent, who appears to be very concerned about the child and wants to be involved at all times in her evaluation and treatment. During this visit, Sarah's treatment requires an injection and a rubber dam application, which you anticipate may lead to increased anxiety. Which strategy would be the least effective in completing the rubber dam application? A. Tell-Show-Do B. Distraction C. Ask the child to be a helper D. Structure time E. Rehearsals

B, Of the choices, distraction would most likely be the least effective approach—the attention of a very anxious individual cannot typically be easily diverted. In such cases, distraction can have detrimental effects such as compromising rapport and/or increasing anxiety by failing to provide a positive coping experience. Providing education and coping strategies—increasing predictability, familiarity, and controllability—are typically more effective strategies in working with anxious patients

A patient is sitting in the chair immediately following an extraction. She says, "Thank you. That wasn't as bad as I expected, but my sister told me that the first night after having a tooth pulled is very painful. What if the medication you're giving me isn't strong enough?" Choose the most appropriate response. A. "Did she make you feel worried about that?" B. "It sounds like you're worried that you might not have enough pain relief when you're home." C. "I understand your concern." D. "Don't worry. I'll give you plenty of pain medicine." E. "It sounds like your sister had a unusually bad experience. Don't believe what others tell you, and certainly don't let that worry you. You'll be fine."

B, Of the options given, the best response would be to interpret what the patient is trying to communicate and reflect the communication back to him or her. This will gently encourage the patient to openly express and discuss the concern with the clinician. It also serves to establish an environment of openness and acceptance

Which of the following would not be expected to cause delayed healing of an extraction site? A. A patient older than 60 years of age B. A patient younger than 10 years of age C. A patient with diabetes D. A patient with a heavy smoking habit

B, Older age, diabetes, and smoking are risk factors for delayed healing

Principles of operant conditioning teach us that _____. A. If you praise your 5-year-old patient and reward him for keeping his legs still while you are drilling, this will make the child happy and more likely to like you and less likely to resist your requests. B. If you praise your 5-year-old patient and reward him for keeping his legs still while you are drilling, this will increase the likelihood that he will remain still in similar situations in the future. C. If you make the dental environment a childfriendly place, your young patient will be more comfortable. D. If you pair the dental chair with having a parent present, the child will be less likely to be anxious. E. None of the above.

B, Operant conditioning posits that behavior is largely influenced by the consequences associated with the particular behavior

The reshaping or recontouring of nonsupportive alveolar bone is called _____. A. Ostectomy B. Osteoplasty C. Osteography D. All of the above

B, Ostectomy is the removal of supporting alveolar bone. Osteoplasty is the reshaping or recontouring of nonsupporting alveolar bone

Instrumentation of the teeth to remove plaque, calculus and stains is defined as _____. A. Coronal polishing B. Scaling C. Gingival curettage D. Root planing

B, Polishing is used to remove plaque and stains from the teeth. Gingival curettage is used to remove the epithelial lining of a periodontal pocket. Root planing is used to create a smooth root surface through the removal of calculus and rough cementum. Scaling is used to remove plaque, calculus, and stains from the tooth

Polymorphisms in which of the following genes have been associated with severe chronic periodontitis? A. IL-6 B. IL-1 C. TNF D. PGE2

B, Polymorphisms in the IL-1 genes have been associated with severe chronic periodontitis.

Which of the following impression materials has the highest tear strength? A. Polyether B. Polysulfide C. Addition silicone D. Condensation silicone

B, Polysulfide has the highest tear strength of all elastomeric impression materials.

Which of the following microorganisms is frequently associated with localized aggressive periodontitis? A. Porphyromonas gingivalis B. Actinobacillus actinomycetemcomitans C. Actinomyces viscosus D. Streptococcus mutans

B, Porphyromonas gingivalis has been associated with chronic periodontitis. Actinomyces viscosus is usually associated with health or gingivitis. Streptococcus mutans is associated with dental caries. Actinobacillus actinomycetemcomitans has been associated with localized aggressive periodontitis

Which of the following depicts the usual order of extraction of teeth if serial extraction is chosen as the treatment to alleviate severe crowding? A. Primary second molars, primary first molars, permanent first premolars, primary canines B. Primary canines, primary first molars, permanent first premolars C. Primary first molars, primary second molars, primary canines D. Primary canines, permanent canines, primary first molars, permanent first premolars

B, Primary canines are extracted to encourage alignment of the crowded incisors. However, the incisors align and upright, borrowing space otherwise needed for eruption of the permanent canine. Primary first molars are then extracted to encourage eruption of the first premolar so it may be extracted to make room for the permanent canine to erupt

Which of the following depicts the usual order of extraction of teeth if serial extraction is chosen as the treatment to alleviate severe crowding? A. Primary second molars, primary first molars, permanent first premolars, primary canines B. Primary canines, primary first molars, permanent first premolars C. Primary first molars, primary second molars, primary canines D. Primary canines, permanent canines, primary first molars, permanent first premolars

B, Primary canines are extracted to encourage alignment of the crowded incisors. However, the incisors align and upright, borrowing space otherwise needed for eruption of the permanent canine. Primary first molars are then extracted to encourage eruption of the first premolar so it may be extracted to make room for the permanent canine to erupt

What is the radiographic sign of successful pulpotomy in a permanent tooth? A. Open apex B. That the apex has formed C. Loss of periradicular lucency D. No internal resorption

B, Pulpotomy is normally not recommended in permanent teeth unless root development is incomplete. If incomplete, the calcium hydroxide pulpotomy is recommended. This is performed in permanent teeth with immature root development and with healthy pulp tissue. The success is indicated when the root apex, if not completely formed, completes its full development. This procedure is only done on teeth free of symptoms

Which of the following is the best radiographic technique to identify a suspected horizontal root fracture in a maxillary anterior central incisor? A. Multiple Water's projections B. Multiple angulated periapical radiographs in addition to a normal, parallel-angulated, periapical radiograph C. A panoramic radiograph D. A reverse Towne's projection

B, Radiographic examination for root fractures is extremely important. Because a root fracture is typically oblique (facial to palatal), one periapical radiograph may easily miss its presence. It is imperative to take at least three angled radiographs (45, 90, 110 degrees) so that in at least one angulation the radiographic beam will pass directly through the fracture line and make it visible on the radiograph.

Which of the following is the main disadvantage of resin-modified glass ionomer compared to conventional glass ionomer? A. Reduced fluoride release B. Increased expansion C. Reduced adhesion D. Cost

B, Resin-modified glass ionomers combine some of the advantages of glass-ionomer cements, such as fluoride release and adhesion, but provide higher strength and low solubility. These materials are less susceptible to early moisture exposure than are glass-ionomer cements but, due to the addition of resin, they exhibit increased thermal expansion.

Which of the following reactions is least likely to be observed during orthodontic treatment? A. Root resorption B. Devitalization of teeth that are moved C. Mobility of teeth that are moved D. Development of occlusal interferences

B, Root resorption is common during orthodontic treatment, although lesions often repair on the root surface. Mobility of teeth is also common as the PDL reorganizes and widens during tooth movement. It is uncommon for teeth to become devitalized as a result of orthodontic movement unless they have also been substantially compromised by injury or infection

Which of the following reactions is least likely to be observed during orthodontic treatment? A. Root resorption B. Devitalization of teeth that are moved C. Mobility of teeth that are moved D. Development of occlusal interferences

B, Root resorption is common during orthodontic treatment, although lesions often repair on the root surface. Mobility of teeth is also common as the PDL reorganizes and widens during tooth movement. It is uncommon for teeth to become devitalized as a result of orthodontic movement unless they have also been substantially compromised by injury or infection

An adult patient presents with a 0.5 ¥ 0.5-cm submucosal mass in the posterior lateral tongue. Biopsy shows a neoplasm composed of glandlike elements and connective tissue elements. It is covered by normal-appearing epithelium. This could be which of the following? A. Oral wart B. Pleomorphic adenoma (mixed tumor) C. Granular cell tumor D. Idiopathic leukoplakia E. Peripheral giant cell granuloma

B, Salivary gland tumors present as submucosal masses. The combination of epithelial and connective tissue elements is indicative of pleomorphic adenomas, also known as mixed tumors. Oral warts and leukoplakias are surface or epithelial lesions. Peripheral giant cell granulomas are exclusively gingival lesions, and granular cell tumors are composed exclusively of cells with grainy or granular cytoplasm.

Which of the following is not a characteristic of sickle scalers? A. Two cutting edges. B. Rounded back. C. Cutting edges meet in a point. D. Triangular in cross section. E. Used for removal of supragingival deposits

B, Scalers have a pointed back; curettes have a rounded back, making them suitable for subgingival instrumentation

The radiolucent portions of the images on a processed dental x-ray film are made up of _____. A. Microscopic grains of silver halide B. Microscopic grains of metallic silver C. A gelatin on a cellulose acetate base D. Unexposed silver bromide

B, Silver halide in the emulsion of an exposed film is converted into grains of metallic silver in the developer.

When comparing pin retention with slot retention for a complex amalgam restoration, which of the following statements is false? A. Slots are used where vertical walls allow opposing retention locks. B. Slots provide stronger retention than pins. C. Slots and grooves can be used interchangeably. D. Pin retention is used primarily where there are few or no vertical walls.

B, Slots and pins may be used interchangeably. They both provide good secondary retention form. Slots are usually better when there exist box forms or vertical walls in the preparation, and pins are usually better when there are few or no vertical walls. The retention is similar for both

During the mixed dentition, a 1-mm diastema develops between the maxillary incisors. Which of the following is most likely? A. The diastema will need orthodontic intervention to be closed B. The diastema will resolve once the canines erupt C. The diastema will only resolve when all of the permanent teeth erupt D. The diastema will continue to widen as permanent teeth erupt

B, Small diastemas between the maxillary incisors of 2 mm or less will generally close on their own as more permanent teeth, specifically the canines, erupt. Presence of a midline diastema before canine eruption is referred to as "the ugly duckling stage."

What is the safest recommended intracoronal bleaching chemical? A. Hydrogen peroxide B. Sodium perborate C. Sodium hypochlorite D. Carbamide peroxide

B, Sodium perborate is more easily controlled and safer than concentrated hydrogen peroxide solutions. Therefore, it should be the material of choice for internal bleaching

What are the characteristics of the primary (initial) bacterial colonizers of the tooth in dental plaque formation? A. Gram-negative facultative B. Gram-positive facultative C. Gram-negative anaerobic D. Gram-positive anaerobic

B, Streptococcal and Actinomyces species are initial colonizers of dental plaque. They are grampositive, facultative micro-organisms.

Restoration margins are plaque-retentive and produce the most inflammation when they are located _____. A. Supragingival B. Subgingival C. At the level of the gingival margin D. On buccal surfaces of teeth

B, Supragingival margins are least detrimental to the gingival tissues; subgingival margins are the most detrimental due to the accumulation of dental plaque.

Which articulator is capable of duplicating the border mandibular movements of a patient? A. Nonadjustable B. Arcon-type C. Nonarcon-type D. Fully adjustable

B, The arcon-type is capable of duplicating a wide range of mandibular movements, but is generally set to follow the patient's border movements. The terminal hinge axis is located and a pantograph is used to record the mandibular movements. These mandibular movement tracings or recordings are used to set the articulator

Tight capillary cell junctions resulting in an added barrier to the entry of drugs is most characteristic of which organ or tissue? A. Adrenal gland B. Brain C. Heart D. Liver E. Lung

B, The brain has especially tight capillary junctions as well as glial cells that result in a blood-brain barrier.

The use of selective COX-2 inhibitors has recently been restricted or discontinued because of what type of adverse effects? A. Carcinogenesis B. Cardiovascular disorders C. Convulsive disorders D. Striated muscle disorders E. Skeletal disorders

B, The cardiovascular risks may be associated with adverse hematologic effects, but the exact mechanism is not yet known

Putting a force through which of the following points would cause pure translation of a tooth without rotation, tipping, or torque? A. Center of rotation B. Center of resistance C. Center of the bracket D. Apex of the root

B, The center of resistance is defined as the point at which force application will cause pure translation of a tooth.

To obtain the most geometrically accurate image, which of the following is false? A. The film should be parallel to the object. B. The central ray should be parallel to the object. C. The central ray should be perpendicular to the film. D. The object-to-film distance should be short. E. The object-to-anode distance should be long

B, The central ray should be perpendicular to the object.

The major objectives of access preparation include all of the following except which one? A. The attainment of direct, straight-line access to canal orifices. B. The confirmation of clinical diagnosis. C. The conservation of tooth structure. D. The attainment of direct, straight-line access to the apical portion of the root.

B, The confirmation of clinical diagnosis. Confirmation of clinical diagnosis should be made before treatment is rendered. Access is the first and arguably the most important phase of nonsurgical root canal therapy. The objectives are: (1) to achieve straight-line access to the apical foramen or curvature of the canal, (2) to locate all root canal orifices, and (3) to conserve sound tooth structure.

The one constant contraindication for a composite restoration is _____. A. Occlusal factors. B. Inability to isolate the operating area. C. Extension onto the root surface. D. Class I restoration with a high C-factor.

B, The constant contraindication for using a composite restoration is the inability to properly isolate the operating area. Occlusal wear of composite is similar to that of amalgam. Extension onto the root surface may result in gap formation with composite but also results in initial leakage with amalgam, indicating that there is no ideal material for root-surface extended restorations. A high C-factor (Class I) can be largely overcome by using (1) a liner under the composite, (2) a filled adhesive, and (3) incremental insertion of the composite

Bones of the cranial base include which of the following? A. Maxilla, mandible, and cranial vault B. Ethmoid, sphenoid, and occipital C. Palatal, nasal, and zygoma D. Frontal and parietal

B, The cranial base includes, from anterior to posterior, the ethmoid, sphenoid, and occipital bones

The main function of the direct retainer of a removable partial denture is _____. A. Stabilization B. Retention C. Support D. Add strength to the major connector

B, The direct retainer's function is to retain the RPD by means of the abutments. Stabilization is provided by the minor connector. Support is provided by the rest. The indirect retainers improve the efficiency of the direct retainers. Direct retainers do not add strength to the major connector.

Excessive vertical angulation causes _____. A. Overlapping B. Foreshortening C. Elongation D. Cone-cutting

B, The film should be parallel to the long axis of the tooth and the central ray of the beam should be perpendicular to both the film and the tooth. Increasing the vertical angulation foreshortens the image of the tooth

The _____ nerve block is useful for dental procedures involving the palatal soft tissues distal to the canine. A. Nasopalatine (NP) B. Greater palatine (GP) C. Long buccal (LB) D. Inferior alveolar (IA)

B, The greater palatine (GP) injection provides softtissue anesthesia of the hard palate from the junction of the premaxilla to the junction of hard and soft palate and from the gingival margin to the midline of the palate

Direct pulp cap is recommended for teeth with _____. A. Carious exposures B. Mechanical exposures C. Calcification in the pulp chambers D. Closed apices more than teeth with open apices

B, The indications for a direct pulp cap are (1) asymptomatic tooth; (2) with little or no hemorrhaging; (3) small (< 1 mm); and (4) well-isolated traumatic pulp exposure. It acts to stimulates the formation of a reparative dentin bridge over the exposure site and to preserve the underlying pulpal tissue. It is especially successful in immature teeth. Failure of direct pulp cap is indicated by (1) symptoms of pulpitis at any time; and (2) lack of vital pulp response after several weeks. Failures result in pulpal necrosis (continual pulpal insult), calcification of the pulp, or (rarely) internal resorption. Direct pulp capping is primarily used on permanent teeth. (Not used often in primary teeth because the alkaline pH of calcium hydroxide.) It can irritate the pulp either mildly or (often) severely. With severe irritation, it increases the risk of internal resorption. With primary teeth, severe resorption is more common; in permanent teeth, formation of reparative dentin occurs more often.

Which of the following injections, when properly performed, does not lead to pulpal anesthesia? A. Inferior alveolar (IA) B. Lingual C. Posterior superior alveolar (PSA) D. Infraorbital (IO) (true anterior superior alveolar nerve block)

B, The inferior alveolar, PSA, and IO injections all lead to pulpal anesthesia when performed properly. The lingual injection leads to soft-tissue anesthesia only

When using a cervical-pull headgear, the forces generated on the maxillary first molar cause this tooth to move in which of the following ways? A. Mesially and to extrude B. Distally and to extrude C. Mesially and to intrude D. Distally and to intrude

B, The line of force generated by a cervical headgear will cause the maxillary first molar to move distally, usually also tip distally, and to extrude. A high-pull headgear would cause the molar to move distally and intrude.

A cutaneous maculopapulary rash of the head and neck preceded by small ulcers in the buccal mucosa would suggest which of the following? A. Primary herpes simplex infection B. Rubeola C. Varicella D. Primary syphilis E. Actinomycosis

B, The maculopapulary rash of rubeola (measles) is preceded by the herald sign of Koplick's spots (punctate ulcers of the buccal mucosa).

When border molding a mandibular complete denture, the extension of the lingual right and left flanges are best molded by having the patient _____. A. Purse the lips B. Wet the lips with the tongue C. Open wide D. Swallow E. Count from 50 to 55

B, The main purpose is to capture the influence of the mylohyoid muscle. The extent of this flange is determined by the elevation of the floor of the mouth when the patient wets the lips with the tip of tongue. Pursing the lips will form the extension of the buccal vestibule. The buccal vestibule is influenced by the buccinator muscle, which extends from the modiolus anteriorly to the pterygomandibular raphe posteriorly and has its lower fibers attached to the buccal shelf and the external oblique ridge

The major mechanisms for the destruction of osseointegration of implants are _____. A. Related to surgical technique B. Similar to those of natural teeth C. Related to implant material D. Related to nutrition

B, The major causes for loss of osseointegrated implants are similar to those of natural teeth: poor hygiene, occlusal load, and the resultant inflammatory processes that occur.

For most periodontitis-affected patients, what is the recommended interval for maintenance appointments? A. 1 month B. 3 months C. 6 months D. 1 year

B, The majority of patients who have been treated for periodontitis should be seen at 3-month intervals for supportive periodontal therapy (maintenance).

The component that is responsible for connecting the major connector with the rest and clamp assembly is: A. The bar B. The minor connector C. The proximal plate D. The guide plane

B, The minor connectors are the components that serve as the part of the removable partial denture that connect the major connector and other components such as the clasp assembly, indirect retainers, occlusal rests, or cingulum rests.

The primary contraindication(s) for the use of a composite restoration is (are) _____. A. Occlusal factors B. Inability to isolate the operating area C. Nonesthetic areas D. Extension onto the root surface

B, The only constant contraindication for the use of composite is when the operating area cannot be properly isolated, thereby decreasing the potential success of the bond.

The pulp tissue of primary teeth _____. 1. In general, is smaller proportionately than permanent pulps in relation to tooth crown size. 2. Is closer to the outer surface of the tooth than the permanent teeth. 3. Follows the general surface contour of the crown. 4. Has the mesial pulp horn closer to the surface than the distal pulp horn. A. Only 1, 2, and 4 are correct. B. Only 2, 3, and 4 are correct. C. Only 1, 3, and 4 are correct. D. 1, 2, 3, and 4 are correct

B, The pulp chambers of primary teeth are proportionately larger compared to the size of the crown. This is significant because there is a higher risk of accidental pulp exposures on primary teeth. In particular, the mesial-buccal pulp horn of the first primary molar is close to the external surface of the tooth.

Which of the following statements best describes pulpal A-delta fibers when compared to C fibers? A. Larger unmyelinated nerve fibers with slower conduction velocities B. Larger myelinated nerve fibers with faster conduction velocities C. Smaller myelinated nerve fibers with slower conduction velocities D. Smaller unmyelinated nerve fibers with faster conduction velocities

B, The pulp contains two types of sensory nerve fibers: myelinated (A fibers) and unmyelinated (C fibers). A fibers include A-beta and A-delta, of which A-delta is the majority. A-delta fibers are principally located in the region of the pulp- dentin junction, have a sharp pain associated with them, and respond to relatively low threshold stimuli. C fibers are probably distributed throughout the pulp, are associated with a throbbing pain sensation, and respond to relatively high threshold stimuli.

During local anesthetic administration, the patient should be placed in a _____ position. A. Trendelenburg B. Supine C. Reclined D. Semi-supine

B, The supine position is correct. This position will prevent fainting during or immediately after the injection of local anesthetic. Reclined or semisupine is not back far enough and Trendelenburg is too far

If the fluoride level in the drinking water is greater than 0.6 ppm at any age, no supplemental systemic fluoride is indicated. If the patient is less than 12 months old, no supplemental systemic fluoride is indicated, whatever the water fluoride level. A. The first statement is true and the second statement is true. B. The first statement is true and the second statement is false. C. The first statement is false and the second statement is true. D. The first statement is false and the second statement is false.

B, The systemic fluoride "Rule of 6s" states: a. If fluoride level is greater than 0.6 ppm, no supplemental systemic fluoride is indicated. b. If the patient is less than 6 months old, no supplemental systemic fluoride is indicated. c. If the patient is greater than 16 years old, no supplemental systemic fluoride is indicated. Therefore, the statement, "If the patient is less than 12 months old, no supplemental systemic fluoride is indicated" is false.

The optimal volume of local anesthetic solution delivered for a true anterior superior alveolar (ASA) nerve block is usually about _____. A. 0.5 mL B. 1.0 mL C. 1.5 mL D. 1.8 mL

B, The true anterior superior alveolar (ASA) nerve block, also called the infraorbital nerve block, requires a volume of one-half cartridge of local anesthetic solution, or about 1.0 mL.

The benzodiazepine receptors BZ1 and BZ2 are located on which ion channel? A. Calcium B. Chloride C. Magnesium D. Potassium E. Sodium

B, The two benzodiazepine receptor subtypes (targets for drugs such as diazepam) are located on the same chloride channel as is the GABAA receptor.

With regard to the mercury controversy related to the use of amalgam restorations, which statement is incorrect? A. There is lack of scientific evidence that amalgam poses health risks to humans except for rare allergic reactions. B. Alternative amalgam-like materials (with low or no mercury content) have promise about mercury. C. True allergies to amalgam rarely have been reported. D. Efforts are underway to reduce the environmental mercury to which people are exposed to lessen their total mercury exposure

B, There are no known alternative low- or nomercury systems that have been developed which provide the same properties or clinical performance as amalgam. The other statements are true

Formocresol has been shown to have a very good success rate when used as a medicament for pulpotomy procedures. Why is there continued interest to find another medicament that performs as well as or better than formocresol? A. Application of formocresol is a clinically timeconsuming procedure. B. Formocresol is toxic and there is possible bloodborne spread to vital organs. C. It has been demonstrated that formocresol may cause spontaneous abortion. D. It has been demonstrated that formocresol may cause failure to develop adequate lung capacity in children.

B, There have been concerns regarding the bloodborne spread of formocresol at least since 1983, when a study was published describing the tissue changes induced by the absorption of formocresol from pulpotomy sites in dogs. Ferric sulfate and mineral trioxide aggregate (MTA) have been demonstrated to be reasonable alternatives to formocresol

Closure of a 2-mm maxillary midline diastema should be accomplished orthodontically in an 8-year-old in which of the following circumstances? A. If the lateral incisors are missing B. If the space creates an esthetic concern and the child is being teased about it C. If there is also deep overbite present D. If mild crowding is also present

B, There is a high likelihood that a small diastema of 2 mm or less will close on its own over time as the permanent teeth erupt. However, if a child suffers psychological trauma because of esthetic concerns, the diastema can be closed. Parents should be informed of the reason for treatment and understand that there are some risks of performing orthodontic treatment that they are assuming

Once the root canal is obturated, what usually happens to the organism that had previously entered periradicular tissues from the canal? A. They persist and stimulate formulation of a granuloma. B. They are eliminated by the natural defenses of the body. C. They reenter and reinfect the sterile canal unless root-end surgery is performed. D. They will have been eliminated by various medicaments that were used in the root canal.

B, They are eliminated by the natural defenses of the body. Obturation prevents coronal leakage and bacterial contamination and seals the remaining irritants in the canal. After root canal obturation, the remaining bacteria should have lost their source of nutrition, becoming susceptible to the body's immune system.

Which of the following statements is false regarding behavior change? A. Goals are long-term targets, whereas objectives are reachable steps/goals along the way. B. Shaping is a behavior change strategy in which the patient learns though the dental professional's demonstration of the desired behavior. C. The basic behavioral model consists of antecedents, behaviors, and consequences. D. Some consequences will strengthen a behavior whereas others will weaken it. E. Generally, the consequences of today's behavior will affect the way in which we behave tomorrow

B, This is an example of modeling. Shaping is the process of achieving successive approximations toward a desired behavior.

Triturating a dental amalgam will _____. A. Reduce the size of the alloy particles B. Coat the alloy particles with mercury C. Reduce the crystal sizes as they form D. Dissolve the alloy particles in mercury

B, Triturating (mixing) the amalgam particle with the mercury is intended to result in coating the particles with a surface of mercury and creating the desirable phases in the set amalgam. All of the alloy particle is not dissolved in the mercury and the size is not significantly reduced

Triturating a dental amalgam will _____. A. Reduce the size of the alloy particles. B. Coat the alloy particles with mercury. C. Reduce the crystal sizes as they form. D. Dissolve the alloy particles in mercury.

B, Triturating (mixing) the amalgam particle with the mercury is intended to result in coating the particles with a surface of mercury and creating the desirable phases in the set amalgam. All of the alloy particle is not dissolved in the mercury, nor is the size significantly reduced.

In the conventional Class I composite preparation, retention is achieved by which of the following features? 1. Occlusal convergence 2. Occlusal bevel 3. Bonding 4. Retention grooves A. 2 and 4 B. 1 and 3 C. 1 and 4 D. 2 and 3

B, Typically, the Class I composite preparation has occlusally converging walls that provide primary retention form. The actual bonding also provides retention form. However, an occlusal bevel is not indicated on Class I preparations nor are retention grooves used

In the conventional Class I composite preparation, retention is achieved by which of the following features? 1. Occlusal convergence 2. Occlusal bevel 3. Bonding 4. Retention grooves A. 2 and 4 B. 1 and 3 C. 1 and 4 D. 2 and 3

B, Typically, the Class I composite preparation has occlusally converging walls that provide primary retention form. The actual bonding also provides retention form. However, an occlusal bevel is not indicated on Class I preparations, nor are retention grooves utilized.

You are unsure of the location of an opaque mass seen over a molar root on a periapical view. A second view of the same region, made with the x-ray machine oriented more from the mesial, reveals that the object has moved mesially with respect to the molar roots on the first view. The location of the object is _____. A. Buccal to the roots B. Lingual to the roots C. In the same plane as the roots D. Insufficient information to form an opinion

B, Use the rule of "SLOB": Same Lingual, Opposite Buccal

Rectangular collimation is recommended because it _____. A. Deflects scatter radiation B. Decreases patient dose C. Increases film density D. Increases film contrast

B, Using a rectangular collimator restricts the area of the patient's face exposed to the size of the receptor, thus reducing more than half the patient exposure

The best way to carve amalgam back to occlusal cavosurface margin is to _____. A. Use visual magnification B. Use a discoid-cleoid instrument guided by the adjacent unprepared enamel C. Make deep pits and grooves D. Use a round finishing bur after the amalgam has set

B, Using the adjacent unprepared enamel at the cavosurface margin to guide the discoid carving instrument when carving away excess amalgam at the occlusal margin is the best way to develop the junction correctly.

Dental wear caused by tooth-to-tooth contact is _____. A. Abrasion B. Attrition C. Erosion D. Abfraction

B, Wasting diseases of the teeth include erosion (corrosion; may be caused by acidic beverages), abrasion (caused by mechanical wear as with toothbrushing with abrasive dentifrice), attrition (due to functional contact with opposing teeth), and abfraction (flexure due to occlusal loading).

A 7-year-old has a 4-mm maxillary midline diastema. Which of the following should be done? A. Brackets should be placed to close it. B. A radiograph should be taken to rule out the presence of a supernumerary tooth. C. Nothing should be done. It will close on its own. D. Nothing should be done. Treatment should be deferred until the rest of the permanent dentition erupts

B, When a large diastema greater than 2 mm is present, it will probably not close on its own. Diagnostic tests, such as a radiograph, should be accomplished to rule out the presence of a supernumerary tooth, usually a mesiodens

When osseointegration occurs, which of the following best describes the implant-bone interface at the level of light microscopy following osseointegration? A. Epithelial attachment B. Direct contact C. Connective tissue insertion D. Cellular attachment

B, When evaluated by light microscopy, there appears to be direct contact at the bone-implant interface

When preparing a Class III or IV composite tooth preparation, which of the following is false regarding placement of retention form? A. Often involves gingival and incisal retention B. Is placed at the axiogingival line angle regardless of the depth of the axial wall C. May be needed in large preps D. Is usually prepared with a No. 1/4 round bur

B, When needed for large restorations, retention form usually consists of a gingival groove and incisal cove prepared with a small round bur (No. 1/4). The placement of the groove or cove is dependent on the DEJ, placing the retention 0.2 mm internal to the DEJ entirely in dentin. It is not placed at the axiogingival or axioincisal line angles if those line angles are deeper than ideal; otherwise, the retention form may be too deep or cause a pulpal exposure

Children in the primary dentition most often present with _____. A. An increased overbite B. A decreased overbite C. An ideal overbite D. A significant open bite

B, Young children often present with minimal overbite or anterior edge-to-edge relationship. Habits such as thumb-sucking increase the likelihood that less overbite will be present

Children in the primary dentition most often present with _____. A. An increased overbite B. A decreased overbite C. An ideal overbite D. A significant open bite

B, Young children often present with minimal overbite or anterior edge-to-edge relationship. Habits such as thumb-sucking increase the likelihood that less overbite will be present

Dental intervention studies suggest that educating patients regarding dental care (patient education) is more effective than behavioral modification (behavioral intervention) in increasing compliance. A. True. B. False. C. Sometimes. D. Both are equally effective. E. Cannot be determined

B,Research has demonstrated that behavioral intervention is typically more effective than patient education alone. A combination of the two is considered the most effective approach to increasing patient compliance

The local anesthetic agent that is most appropriate for use in most children is _____. A. 3% mepivacaine B. 2% mepivacaine with 1:20,000 levonordefrin C. 2% lidocaine with 1:100,000 epinephrine D. 0.5% bupivacaine with 1:200,000 epinephrine

C, 2% lidocaine with 1:100,000 epinephrine is the local anesthetic that allows the greatest volume to be administered safely. Therefore, it is the local anesthetic drug of choice for administration in children. Mepivacaine in either 2% or 3% allows less volume to be safely administered and bupivacaine is not FDA-approved for administration to children.

A test result that erroneously excludes an individual from a specific diagnostic or reference group is called _____. A. Erroneous B. False positive C. False negative D. Mistaken E. None of the above

C, A false positive test is a test result which erroneously assigns an individual to a specific diagnostic or reference group.

6. A finishing bur has how many blades compared to a cutting bur? A. Fewer blades. B. Same number of blades. C. More blades. D. Number of blades is unrelated to the bur type

C, A finishing bur is designed to provide a smoother surface and therefore has more blades than a cutting bur. The increased blade numbers results in a smoother cut surface

The success of an amalgam restoration is dependent on all of the following features of tooth/cavity preparation except _____. A. Butt-joint cavosurface margin that results in a 90-degree margin for the amalgam B. Adequate tooth removal for appropriate strength of the amalgam C. Divergent (externally) preparation walls D. Adequate retention form features to mechanically lock the amalgam in the preparation

C, A successful amalgam restoration requires 90-degree amalgam margins. Amalgam margins less than 90 degrees result in increased potential for fracture of the amalgam. Greater than 90-degree amalgam margins are good for the amalgam but the corresponding enamel margin will be less than 90 degrees and therefore potentially undermined and have potential for fracture. Since the amalgam is not bonded to the tooth, it must be retained in the tooth with undercuts, either in the primary or secondary preparation. An amalgam restoration needs a minimum of 1-mm thickness in nonstress areas and 1.5 to 2 mm in areas that may be under load. Therefore, the preparation must provide this dimension. Except for Class V amalgams, the prepared walls generally converge to the exterior. Thus, the prepared walls may diverge or converge externally.

Which of the following is true regarding crowding of the dentition? A. Crowding of the primary dentition usually resolves as the permanent teeth erupt. B. Spacing in the primary dentition usually indicates spacing will be present in the adult. C. Approximately 15% of adolescents have crowding severe enough to consider extraction of permanent teeth as part of treatment. D. Lower incisor crowding is more common in African-American than white populations.

C, According to data available, approximately 15% of adolescents have severe crowding that would require major amounts of expansion or extractions to resolve. The other statements are false: crowding in the primary dentition is very rare and would indicate crowding will occur in the permanent dentition; spacing in the primary dentition is normal; and African-Americans generally have less crowding than whites.

Levonordefrin is added to certain cartridges containing mepivacaine. The desired effect of levonordefrin is due to what pharmacological effect? A. Inhibition of nicotinic cholinergic receptors B. Inhibition of muscarinic cholinergic receptors C. Stimulation of α-adrenergic receptors D. Stimulation of β-adrenergic receptors E. Stimulation of dopamine receptors

C, Alpha adrenergic receptor stimulation accounts for the vasoconstrictor effect of levonordefrin.

The administration of which compound will give "epinephrine reversal" (drop in blood pressure from epinephrine) if given prior to administration of epinephrine? A. Guanethidine B. Propranolol C. Phenoxybenzamine D. Tyramine

C, Alpha adrenoceptor blockers, like phenoxybenzamine, inhibit the vasoconstrictor effect of epinephrine but not the vasodilator effect of epinephrine. Therefore, the administration of alpha blockers results in epinephrine reversal. Propranolol would only block the vasodilator effect of epinephrine. Guanethidine and tyramine act largely at prejunctional sites and don't block adrenergic receptors.

What is the most objective clinical indicator of inflammation? A. Gingival color B. Gingival consistency C. Gingival bleeding D. Gingival stippling

C, Although changes in gingival color and consistency and loss of gingival stippling can be indicators of gingival inflammation, bleeding on probing is the most objective clinical indicator

Which of the following procedures would be considered the least invasive surgical treatment for TMJ complaints? A. Splint therapy B. Arthrocentesis C. Arthroscopy D. Disc removal E. Total joint replacement

C, Although less invasive, arthrocentesis and splint therapy are not considered surgical interventions

Which one of the following acids is generally recommended for etching tooth structure? A. Maleic acid B. Polyacrylic acid C. Phosphoric acid D. Tartaric acid E. Ethylenediaminetetraacetic acid (EDTA)

C, Although some of the self-etch bonding systems use milder acid, the primary acid system used for etching tooth structure is phosphoric acid

Which one of the following acids is generally recommended for etching tooth structure? A. Maleic acid B. Polyacrylic acid C. Phosphoric acid D. Tartaric acid E. EDTA

C, Although some of the self-etch bonding systems use milder acid, the primary acid system used for etching tooth structure is phosphoric acid.

The most common clinical sign of occlusal trauma is _____. A. Tooth migration B. Tooth abrasion C. Tooth mobility D. Tooth attrition

C, Although tooth migration can be a sign of occlusal trauma, tooth mobility is the most common clinical sign.

Injecting a local anesthetic into an area of inflammation would have which effect? A. Increase the rate of onset of anesthesia. B. Decrease the rate of metabolism of the anesthetic. C. Reduce the net anesthetic effect of the drug. D. Reduce the vasodilator effect of the local anesthetic. E. Reduce the need for a vasoconstrictor with the local anesthetic.

C, An area of inflammation is an area of low pH. The acid environment would convert more of the drug into the charged form, making it less able to diffuse to the nerve cells. This would reduce the rate of onset and the net anesthetic effect of the drug.

A dentist is preparing all maxillary anterior teeth for metal-ceramic crowns. Which of the following procedures is necessary in order to preserve and restore anterior guidance? A. Protrusive record. B. Template for provisional restorations. C. Custom incisal guide table. D. Interocclusal record in centric relation

C, Anterior guidance must be preserved by means of construction of a custom incisal guide table, especially when restorative procedures change the surfaces of anterior teeth that guide the mandible in excursive (lateral, protrusive) movements

A 1-year-old patient has his first dental examination. The dentist reviews with the parent when to expect the next teeth to erupt, teething, oral hygiene tips for toddlers, and discusses fluoride issues with bottled water and toothpaste. The term that describes this proactive approach to dental care is _____. A. Risk assessment B. Probability counseling C. Anticipatory guidance D. Preventive support counseling

C, Anticipatory guidance is counseling patients and parents regarding the child's home oral healthcare that is age-appropriate and is focused on prevention. Subjects to discuss with parents include: a. Oral hygiene b. Oral development c. Fluoride d. Diet and nutrition e. Oral habits f. Trauma and injury prevention

Which is the most likely to cause pulp necrosis? A. Subluxation B. Extrusion C. Avulsion D. Concussion

C, Avulsion. To have pulp space infection, the pulp must first become necrotic. This will occur in a fairly serious injury in which displacement of the tooth results in severing of the apical blood vessels.

Pick the organism that is usually clinically sensitive to clarithromycin but not to penicillin V. A. Streptococcus viridans B. Leptotrichia buccalis C. Mycoplasma pneumoniae D. Streptococcus pneumoniae E. Streptococcus pyogenes

C, Because it lacks a cell wall, Mycoplasma pneumoniae is not sensitive to cell wall inhibitors such as penicillin V. The macrolides, such as clarithromycin, are ribosomal protein synthesis inhibitors that are effective against Mycoplasma pneumoniae. Streptococcus viridans, Streptococcus pneumoniae, and Streptococcus pyogenes are gram-positive cocci. Leptotrichia buccalis is a gram-negative oral bacillus

Which may be a consequence of occlusal trauma on implants? A. Widening of the periodontal ligament. B. Soft-tissue sore area around the tooth. C. Bone loss. D. All of the above.

C, Bone loss is usually seen on the most coronal aspect of the implant in the form of a wedge. There is no periodontal ligament on implants, so there is no feeling of soreness

A classic teardrop-shaped periradicular lesion on a radiograph can be indicative of a vertical root fracture. The prognosis of a vertical root fracture is hopeless, and the tooth should be extracted. A. First statement is true, second is false. B. First statement is false, second is true. C. Both statements are true. D. Both statements are false.

C, Both statements are true. Often the radiographic interpretation of a vertical root fracture is the pattern of bone loss occurring in a teardropshaped, J-shaped, or halolike radiolucency, with the bone loss originating apically and progressing coronally up one side of the root. Because vertical root fractures are susceptible to microleakage and because of their compromised internal structure, they have a poor prognosis and should be extracted.

If a local anesthetic has a low pKa, then it will usually have a _____. A. Greater potency B. Higher degree of protein binding C. Faster onset of action D. Greater vasodilating potential

C, By definition, a low pKa means a fast onset of action. Hydrophobicity and protein binding directly affect duration of action and potency

Which of the following can be viewed on a conventional radiograph? A. Buccal curvature of roots B. Gingival fibers C. Calcification of canals D. Periodontal ligament

C, Calcification of canals. Buccal curvature cannot be seen from the conventional radiographs. Gingival fibers and the periodontal ligament, being connective tissues, are radiolucent radiographically

What are the major organic constituents of bacterial plaque? 1. Calcium and phosphorous 2. Sodium and potassium 3. Polysaccharides and proteins 4. Glycoproteins and lipids A. 1 and 2 B. 2 and 3 C. 3 and 4 D. 2 and 4

C, Calcium, phosphorous, sodium, and potassium are inorganic components of dental plaque. Polysaccharides, proteins, glycoproteins, and lipids are organic components of dental plaque

Initial instrumentation in endodontic treatment is done to the level of the _____. A. Radiographic apex B. Dentinoenamel junction C. Cementodentinal junction D. Cementopulpal junction

C, Cementodentinal junction. At the apex, or bottom of the tooth, the canal narrows. This narrowing is the cementodentinal junction (CDJ) or the apical constriction. This narrow spot provides a natural stop for debris, irrigation and filling materials from being forced into the periapical tissue. Most dentists will work to clean the canal down to this point in their root canal procedures.

A 21-year-old woman went to her dentist because of facial asymmetry. This had occurred gradually over a period of 3 years. The patient had no symptoms. A diffusely opaque lesion was found in her right maxilla. All lab tests (CBC, alkaline phosphatase, calcium) were within normal limits. Biopsy was interpreted as a fibroosseous lesion. She most likely has _____. A. Cementoblastoma B. Fibrous dysplasia C. Cherubism D. Osteosarcoma E. Chronic osteomyelitis

C, Cherubism is a fibro-osseous lesion that occurs in teenagers. Characteristically, it presents with ill-defined margins and a "ground glass" appearance radiographically. The other features described also support this diagnosis.

Which of the following is not a relative contraindication for routine, elective oral surgery? A. Unstable cardiac angina B. History of head and neck radiation C. Chronic sinusitis D. Hemophilia

C, Chronic sinusitis is not a relative contraindication to most elective oral surgical procedures. Unstable chest pain should be evaluated by an internist or cardiologist prior to any dental treatment. Radiation to the jaws or a history of clotting disorders would both need further investigation of the health history and likely alter the patient's treatment plan to lessen the likelihood of osteoradionecrosis or of bleeding complications

What tissue or organ has many muscarinic receptors but lacks innervation to those receptors? A. Heart B. Parotid gland C. Blood vessels D. Sweat glands E. Urinary bladder

C, Circulating muscarinic cholinergic receptor agonists stimulate these receptors on endothelial cells, leading to release of nitric oxide and vasodilation.

Clavulanic acid offers an advantage therapeutically because it has what action? A. It inhibits Streptococci at a low minimum inhibitory concentration (MIC). B. It inhibits transpeptidase. C. It inhibits penicillinase. D. It inhibits anaerobes at a low MIC. E. It inhibits DNA gyrase.

C, Clavulanic acid has very little antimicrobial activity. Its value in combination with certain penicillins is due to its ability to inhibit certain penicillinases. This protects the penicillin from bacterial enzyme attack. Transpeptidase is inhibited by β-lactams, such as penicillin. DNA gyrase is inhibited by the fluoroquinolones such as ciprofloxacin.

A compomer cement _____. 1. Is indicated for cementation of metal-ceramic crowns. 2. Is indicated for cementation of all-ceramic restorations. 3. Is indicated for some all-ceramic crowns, inlays, and veneers with some contraindications. 4. Has low solubility and sustained release of fluoride. A. All are correct. B. 1, 2, and 3 are correct. C. 1, 3, and 4 are correct. D. 2, 3, and 4 are correct.

C, Compomer cements (also known as resin-modified glass ionomer cements) have low solubility, low adhesion, and low microleakage. They are not recommended to be used with all-ceramic restorations because they have been associated with fracture, which is probably due to their water absorption and expansion.

Which of the following factors are involved in the cognitive appraisal of a threat? A. Interference, adaptability, longevity, and reactance B. Adaptability, preventability, inevitability, and constancy C. Controllability, familiarity, predictability, and imminence D. Validity, reliability, adaptability, and predictability E. Accountability, reliability, validity, and familiarity

C, Controllability, familiarity, predictability, and imminence are significant factors influencing the cognitive appraisal of stress

What is the mechanism of the analgesic action of aspirin? A. Stimulates μ opioid receptors B. Blocks histamine H2 receptors C. Inhibits cyclooxygenase D. Inhibits lipoxygenase E. Blocks sodium channels in nerves

C, Cyclooxygenase (COX) is a key enzyme in the synthesis of prostaglandins. Prostaglandins, including PGE2 and PGF2α, are important mediators for such functions as pain, and are a product of COX. Aspirin inhibits both COX-1 and COX-2.

In this type of study design, neither the subject nor the investigator knows to which group a subject belongs. A. Matching studies B. Randomized C. Double-blind D. Single-blind E. None of the above

C, Double-blind designs help prevent the potential for a biased interpretation of a treatment effect that might occur if either the investigator or subjects know to which group the latter belong.

Ectopic lymphoid tissue would most likely be found in which of the following sites? A. Hard gingiva B. Soft gingiva C. Floor of mouth D. Dorsum of tongue E. Vermilion of the lip

C, Ectopic (normal tissue, abnormal site) lymphoid tissue is commonly seen in floor of the mouth as well as in posterior lateral tongue, soft palate, and tonsilar pillar. It appears as one or more small, dome-shaped yellow nodules

Which of the following tests is the least useful in endodontic diagnosis of children? A. Percussion B. Palpation C. Electric pulp test D. Cold test

C, Electric pulp test. Until apical closure occurs, teeth do not respond normally to electric pulp testing. In addition, a traumatic injury may temporarily alter the conduction capability of nerve endings and/or sensory receptors in the pulp. A patient with a vital pulp may not experience any sensation right after trauma.

Which of the following is an important constituent of gram-negative microorganisms that contributes to initiation of the host inflammatory response? A. Exotoxin B. Lipoteichoic acid C. Endotoxin D. Peptidoglycan

C, Endotoxin or lipopolysaccharide is an important constituent of the gram-negative outer membrane that contributes to initiation of the host inflammatory response

In the treatment of an acute periodontal abscess, the most important first step is to _____. A. Prescribe systemic antibiotics B. Reflect a periodontal flap surgery C. Obtain drainage D. Prescribe hot salt mouth washes

C, Establishment of drainage is the first step in treating an acute periodontal abscess. The patient may then use self-applied mouth rinses and be prescribed antibiotics if there is evidence of systemic involvement (e.g., fever, lymphadenopathy). A flap would be reflected in a subsequent appointment if the abscess did not resolve and became a chronic problem

All of the following may be indications to consider extraction of permanent teeth in an orthodontic patient except _____. A. Excessive crowding B. Class II interarch relationship C. Flat lip profile D. Anterior open bite

C, Excessive crowding may necessitate extractions. Also, extraction of maxillary premolars may be indicated to camouflage a Class II molar relationship. Anterior open bites may be improved by uprighting anterior teeth to increase overbite. Flat lips will not be improved by extraction of permanent teeth but other considerations may still necessitate extraction even in those patients

What is the half-life of Hg in the human body? A. 5 days B. 25 days C. 55 days D. 85 days E. 128 days

C, Fifty-five days is the half-life of mercury in the body.

In a patient with incomplete cleft palate, which of the following aspects is most likely to remain open? A. The anterior aspect B. The middle aspect C. The posterior aspect D. The right aspect

C, Fusion of the palate proceeds from anterior to posterior, so any disturbance that occurs during that time will stop fusion at that point, leading to an opening posteriorly

Which of the following best distinguishes periodontitis from gingivitis? A. Probing pocket depth B. Bleeding on probing C. Clinical attachment loss D. Presence of suppuration

C, Gingivitis is characterized by inflammation of the gingival tissues with no loss of clinical attachment. Periodontitis is characterized by inflammation with loss of clinical attachment

The first time you perform a complicated dental procedure, you feel uncomfortable and nervous. At one point, you even think for a moment that you will not be able to complete the procedure. However, you stay with it, and near the end of the procedure you feel much better. Which concept does this best exemplify? A. Covert conditioning B. Systematic desensitization C. Habituation D. Cognitive restructuring E. Psychoeducation

C, Habituation is the decrease in response that occurs as a result of repeated or prolonged exposure to a conditioned stimulus.

Herpes simplex virus is the cause of which of the following? A. Minor aphthous ulcers B. Herpetiform aphthae C. Herpes whitlow D. Herpangina E. Herpes zoster

C, Herpes whitlow is a term used for secondary herpes simplex infections that occur around the nail bed. The cause of aphthous ulcers is unknown, herpangina is caused by Coxsackie virus, and herpes zoster is caused by varicellazoster virus.

The imaging evaluation of the temporomandibular joint is most likely to include any of the following except _____. A. Panoramic radiographs B. TMJ tomograms C. Xeroradiography D. Magnetic resonance ima

C, Imaging tools used in the evaluation of TMJ pathology include panoramic radiographs, traditional and computer generated tomograms, MRIs, nuclear imaging, and arthography

Upon evaluation of an immediate postoperative panoramic film of a dental implant replacing tooth #30, you measure a distance of 1.5 mm from the apex of the implant to the inferior alveolar nerve canal. This is a titanium implant in an otherwise healthy patient. Which of the following actions is indicated? A. You may proceed with immediate loading of the implant. B. You should continue but only perform a two-stage procedure. C. Back the implant out approximately 0.5 mm to ensure a safe distance from the nerve. D. Remove the implant and plan a repeat surgery after 4 months of healing.

C, Implants should be placed a minimum of 2 mm from the inferior alveolar canal

Inadequate rest-seat preparation for a removable partial prosthesis can cause _____. A. Tooth mobility B. Ligament widening C. Occlusal rest fracture D. Occlusal rest distortion

C, In McCracken's Removable Partial Prosthodontics, ed 11 (St Louis, Mosby, 2005), McCracken states, "Failure of an occlusal rest rarely results from a structural defect in the metal and rarely if ever is caused by accidental distortion. Therefore the blame for such failure must often be assumed by the dentist for not having provided sufficient space for the rest during mouth preparations

A group of researchers undertook a study to assess the relationship between squamous cell carcinoma and chewing tobacco. The researchers determined past exposure records among subjects who had been diagnosed with the disease. This type of study was a _____. A. Clinical trial B. Community trial C. Retrospective cohort study D. Case control study E. Randomized clinical trial

C, In a retrospective cohort study, the investigator chooses a sample of individuals who have the outcome of interest (in this case, squamous cell carcinoma) and then look into the past for possible variables that may have caused the disease (e.g., chewing tobacco).

Pulp capping and pulpotomy can be more successful in newly erupted teeth than in adult teeth because _____. A. A greater number of odontoblasts are present B. Of incomplete development of nerve endings C. An open apex allows for greater circulation D. The root is shorter

C, In newly erupted teeth, the apical root end has not fully formed, allowing for greater blood supply to the tooth. Subsequent pulpal regeneration leads to greater long-term success

Acantholysis, resulting from desmosome weakening by autoantibodies directed against the protein desmoglein, is the disease mechanism attributed to which of the following? A. Epidermolysis bullosa B. Mucous membrane pemphigoid C. Pemphigus vulgaris D. Herpes simplex infections E. Herpangina

C, In pemphigus vulgaris, autoantibodies attach to antigens (desmoglein) found in desmosomes that keep keratinocytes linked to each other. Cells eventually separate from each other (acantholysis), resulting in short-lived intraepithelial vesicles/bullae

The following teeth are erupted in an 8-year-old patient. What is the space maintenance of choice? A. Band-loop space maintainer. B. Lower lingual holding arch. C. Nance holding arch. D. Distal shoe space maintainer

C, In space maintenance, the clinician must always be mindful of the exfoliation sequence of teeth. In this situation, the authors would normally exfoliate prior to the eruption of the second permanent premolar, tooth #13. If a band loop space maintainer were used, there may be no anterior abutment if there is a normal exfoliation sequence. This could result in mesial tipping of the permanent molar and space loss. A Nance holding arch or a palatal holding arch would be an appropriate choice.

In the primary dentition, the mandibular foramen is located where in relation to the plane of occlusion? A. Higher than the plane of occlusion B. Much higher than the plane of occlusion C. Lower than the plane of occlusion D. The same level as the plane of occlusion

C, In the primary dentition patient, the mandibular foramen is located lower than the plane of occlusion. Therefore, mandibular block injections for these patients are made somewhat lower than as is done for the adult patient

In this section of a scientific article, the researcher interprets and explains the results obtained. A. Summary and conclusion B. Results C. Discussion D. Abstract E. None of the above

C, In the results section the researcher describes the specific findings and actual outcomes of the project but does not interpret them. The interpretation and analysis of the results are part of the discussion, where the researcher attempts to explain his results.

Light smokers are likely to have less severe periodontitis than heavy smokers. Former smokers are likely to have more severe periodontitis than current smokers. A. Both statements are true. B. Both statements are false. C. The first statement is true, the second statement is false. D. The first statement is false, the second statement is true.

C, Individuals who smoke cigarettes are more likely to have periodontal disease than are nonsmokers. The number of cigarettes smoked and the number of years of smoking affect the severity of disease. Former smokers usually have less disease than do current smokers

Zinc oxide eugenol is a good temporary restoration because _____. A. It is less irritating B. It has increased strength over other restorations C. It provides a good seal D. It is inexpensive

C, It is the physical and chemical properties of zinc oxide eugenol that are beneficial in preventing pulpal injury and in reducing postoperative tooth sensitivity. Importantly, it provides a good biological seal; also, its antimicrobial properties enable it to suppress bacterial growth, thus reducing formation of toxic metabolites that might result in pulpal inflammation

Which local anesthetic is most hydrophobic and has the highest degree of protein binding? A. Mepivacaine B. Lidocaine C. Bupivacaine D. Procaine

C, Lipid solubility (therefore, hydrophobicity) and protein binding are the most important factors in determining duration of action of a local anesthetic. Bupivacaine has the longest duration of action of the listed local anesthetics and also has the highest hydrophobicity; it is bound 95% to protein. The other listed agents have lower hydrophobic qualities and are 75% or less bound to protein.

Which antihypertensive drug also increases bradykinin levels? A. Candesartan B. Furosemide C. Lisinopril D. Metoprolol E. Nifedipine

C, Lisinopril, by virtue of the fact that it inhibits angiotensin-converting enzyme (ACE) (also called peptidyl dipeptidase), inhibits the breakdown of bradykinin.

According to Malamed, the maximum local anesthetic dose of lidocaine (with or without vasoconstrictor) is _____. A. 1.5 mg/kg. B. 2.0 mg/kg. C. 4.4 mg/kg. D. 7.0 mg/kg

C, Malamed recommends that 4.4 mg/kg (2.0 mg/lb) of lidocaine be the maximum administered, regardless of whether vasoconstrictor is in the formulation. The package insert for lidocaine allows up to 7 mg/kg when lidocaine is packaged with vasoconstrictor

Which of the following teeth has the most consistent number of canal(s)? A. Mandibular incisor B. Mandibular canine C. Maxillary canine D. Mandibular premolar

C, Maxillary canine. The percentage of one canal in maxillary canines has been found in some studies to be between 97% and 100%, making it one of the most consistent anatomical teeth in the mouth.

Which of the following is considered the highest and most severe classification of maxillary fracture? A. LeFort I B. LeFort II C. LeFort III D. LeFort IV

C, Maxillary fractures may be classified as LeFort I, II, or III. The LeFort III is the highest and most severe.

The nevoid basal cell carcinoma syndrome includes multiple basal cell carcinomas, bone abnormalities, and which of the following? A. Osteomas B. Café-au-lait macules C. Odontogenic keratocysts D. Hypoplastic teeth E. Lymphoma

C, Multiple odontogenic keratocysts are part of the nevoid basal cell carcinoma syndrome

A 43-year-old-male patient presents with an asymptomatic anterior palatal swelling. A radiograph shows a 1 × 1-cm lucency and divergence of tooth roots #8 and #9. All teeth in the area are vital. This is most likely a(an) _____. A. Periapical granuloma B. Aneurysmal bone cyst C. Nasopalatine duct cyst D. Globulomaxillary lesion E. Dermoid cyst

C, Nasopalatine duct cysts are anterior midmaxillary lesions that occur in the nasopalatine canal. The associated lucency is often heart-shaped because of the superimposition of the nasal spine over the lesion. They do not devitalize teeth

Which of the following are antigen-presenting cells? A. Neutrophils B. T-lymphocytes C. Macrophages D. Plasma cells

C, Neutrophils are one of the primary defense cells of the innate immune system. T-lymphocytes are important activators of the specific (adaptive) immune system. Macrophages are antigenpresenting cells. Plasma cells produce antibodies

The predominant inflammatory cells in the periodontal pocket are _____. A. Lymphocytes B. Plasma cells C. Neutrophils D. Macrophages

C, Neutrophils are the predominant inflammatory cells in the periodontal pocket and have migrated across the pocket epithelium from the subgingival vascular plexus

Which of the following is the most significant cause of ledge formation? A. Infection B. Remaining debris within the canal C. No straight-line access

C, No straight-line access. After the orifice has been found, the clinician must decide if straight-line access has been achieved. Unnecessary deflection of the file can result in numerous consequences related to loss of instrument control. Attempts to clean and shape without straight-line access often lead to procedural errors such as ledging, transportation, and zipping.

When a diagnosis of odontogenic keratocyst is made, the patient should be advised as to _____. A. The need for full-mouth extractions B. The association with colonic polyps C. The associated recurrence rate D. The likelihood of malignant transformation E. The need for additional laboratory studies

C, Odontogenic keratocysts are notable because of their recurrence rate, their aggressive clinical behavior, and their occasional multiplicity. When multiple, they may be part of the nevoid basal cell carcinoma syndrome

During admission, a patient interrupts you on a number of occasions with stories about past dental experiences while you are attempting to take a complete medical history. Your best response would be _____. A. Say nothing, listen to the patient, and finish your intake as best you can. B. Say, "I'd like to focus on your present experience and right now I need to know your medical history." C. Say, "It seems like you've had some important experiences and I would like to hear more about them, but first, let's discuss this health questionnaire before we address it, okay?" D. Say, "I don't need to know the details of your dental history. Please inform me of the experiences asked about in the questionnaire." E. Say, "We have about 30 minutes to complete this questionnaire and get started in your examination, so let's focus on that."

C, Of the options given, it is best to acknowledge that the patient is trying to convey information that is important to him or her and establish that there will be a time to talk about those issues, while gently redirecting him or her to the task at hand

Methemoglobinemia is an adverse effect associated with which local anesthetic due to its metabolism to o-toluidine? A. Lidocaine B. Mepivacaine C. Prilocaine D. Bupivacaine E. Benzocaine

C, Only prilocaine is metabolized to o-toluidine

Which drugs tend to concentrate in body compartments of high pH? A. Permanently charged drugs B. Drugs that are not charged C. Weak organic acids D. Weak organic bases E. Inorganic ions

C, Only weak acids and weak bases are greatly affected in their distribution by changes in pH. Weak organic acids dissociate more from protons at higher pH, making a higher percentage of their molecules charged. This traps them in that compartment

Which of the following is the most important preventive and therapeutic procedure in periodontal therapy? A. Professional instrumentation B. Subgingival irrigation with chlorhexidine C. Patient-administered plaque control D. Surgical intervention

C, Patient cooperation and effectiveness in removing bacterial plaque is of primary importance in maintaining a healthy periodontium

Patients with which of the following should not be treated with ultrasonic instruments? A. Deep periodontal pockets B. Edematous tissue C. Infectious diseases D. Controlled diabetes

C, Patients with active infectious diseases should not be treated with ultrasonic instruments because of the aerosol that is created when using this type of instrument.

What features best characterize the predominant microflora associated with periodontal health? A. Gram-positive, anaerobic cocci and rods B. Gram-negative, anaerobic cocci and rods C. Gram-positive, facultative cocci and rods D. Gram-negative, facultative cocci and rods

C, Periodontal health is characterized by a microflora dominated by gram-positive, facultative cocci and rods

Which of the following are part of Preliminary Phase therapy? a. Treatment of emergencies b. Extraction of hopeless teeth c. Plaque control d. Removal of calculus A. a, b, and c B. b, c, and d C. a and b only D. b and d only

C, Preliminary Phase therapy is used to treat emergencies and remove hopeless teeth.

Restoration of an appropriate proximal contact results in all of the following except _____. A. Reduction/elimination of food impaction at the interdental papilla. B. Provide appropriate space for the interdental papilla. C. Provide increased retention form for the restoration. D. Maintenance of the proper occlusal relationship.

C, Proper proximal contacts reduce the potential for food impaction, thereby preserving the health of the underlying soft tissue. A missing proximal contact may result in tooth movement that will have an adverse effect on the occlusal relationship of the tooth. Having a correct contact does not enhance the retentive properties of the restorative material.

Restoration of an appropriate proximal contact results in all of the following except _____. A. Reduction/elimination of food impaction at the interdental papilla B. Provides appropriate space for the interdental papilla C. Provides increased retention form for the restoration D. Maintenance of the proper occlusal relationship

C, Proper proximal contacts reduce the potential for food impaction, thereby preserving the health of the underlying soft tissue. A missing proximal contact may result in tooth movement that will have an adverse effect on the occlusal relationship of the tooth. Having a correct contact does not enhance the retentive properties of the restorative material.

The best strategy for addressing dental fear that is based upon distrust of the dentist is to _____. A. Use distraction techniques B. Use cognitive coping strategies C. Enhance informational and behavioral control D. Teach diaphragmatic breathing E. Reassure the patient that he or she can trust you

C, Providing the patient with information and control over his or her environment is likely to contribute to increased trust over time. Avoiding the issue of trust or providing reassurance that the patient can trust you without evidence is likely to maintain poor trust

Which of the following methods of radiographic assessment are best for identifying small volumetric changes in alveolar bone density? A. Bitewing B. Periapical C. Subtraction D. Panoramic

C, Radiographs must be taken in a standardized format at repeated visits to be assessed for small changes in bone density over time, using subtraction radiography. Radiographs are usually standardized by using a bite registration block to relocate the x-ray at the same place and angulation each time.

A patient seeks help for recurrent palatal pain. She presents with multiple punctate ulcers in the hard palate that were preceded by tiny blisters. Her lesions typically heal in about 2 weeks and reappear during stressful times. She has _____. A. Aphthous ulcers B. Recurrent primary herpes C. Recurrent secondary herpes D. Erythema multiforme E. Discoid lupus

C, Recurrent intraoral herpes simplex infections occur only in the hard palate and hard gingiva, with the exception of AIDS patients. Blister (vesicle) history and recurrence are also supportive of this diagnosis.

Fanconi syndrome from outdated tetracyclines affects predominantly which organ? A. Brain B. Heart C. Kidney D. Pancreas E. Stomach

C, Renal tubular acidosis, aminoaciduria, and hyperphosphaturia are some of the manifestations of proximal tubule damage in Fanconi syndrome

According to Scammon's growth curves, which of the following tissues has a growth increase that can be used to help predict timing of the adolescent growth spurt? A. Neural tissues B. Lymphoid tissues C. Reproductive tissues

C, Reproductive tissues grow at the same time as the adolescent growth spurt and the appearance of secondary sexual characteristics can be used to help predict the timing of growth.

According to Scammon's growth curves, which of the following tissues has a growth increase that can be used to help predict timing of the adolescent growth spurt? A. Neural tissues B. Lymphoid tissues C. Reproductive tissues

C, Reproductive tissues grow at the same time as the adolescent growth spurt, and the appearance of secondary sexual characteristics can be used to help predict the timing of growth

Which of the following is true of an occlusal rest for a removable partial denture? 1. One-third facial lingual width of the tooth 2. 1.5 mm deep for base metal 3. 2.0 mm labiolingual width of the tooth 4. Floor inclines apically toward the center of the tooth A. All of the above B. 1, 3, and 4 C. 1, 2, and 4 D. 3 and 4

C, Rests are critical for the health of the soft tissues underlying the denture resin basis and the minor and major connectors. It should prevent tilting action and should direct forces through the long axis of the abutment tooth. In order to function as specified, an occlusal rest should have a rounded (semicircular) outline form, be one-third the facial lingual width of the tooth, one-half the width between cusps, and at least 1.5 mm deep for base metal. The rest floor inclines apically toward the center of the tooth and the angle formed with the vertical minor connector should be less than 90 degrees.

When placement of proximal retention locks in Class II amalgam preparations is necessary, which of the following is incorrect? A. One should not undermine the proximal enamel. B. One should not prepare locks entirely in the axial wall. C. Even if deeper than ideal, one should use the axial wall as a guide for proximal lock placement. D. One should place locks 0.2 mm inside the DEJ to ensure that the proximal enamel is not undermined.

C, Retention locks, when needed in Class II amalgam preparations, should be placed entirely in dentin, thereby not undermining the adjacent enamel. They are placed 0.2 mm internal to the DEJ, are deeper gingivally (0.4 mm) than occlusally (i.e., they fade out as they extend occlusally), and translate parallel to the DEJ. If the axial wall is deeper than normal, the retention lock is not placed at the axiofacial or axiolingual line angles but, rather, is positioned 0.2 mm internal to the DEJ. If placed at the deeper location, it may result in pulp exposure, depending on the location of the axial wall depth.

When placement of proximal retention locks in Class II amalgam preparations is necessary, which of the following is incorrect? A. One should not undermine the proximal enamel. B. One should not prepare locks entirely in axial wall. C. Even if deeper than ideal, one should use the axial wall as a guide for proximal lock placement. D. One should place locks 0.2 mm inside the DEJ to ensure that the proximal enamel is not undermined.

C, Retention locks, when needed in Class II amalgam preparations, should be placed entirely in dentin, thereby not undermining the adjacent enamel. They are placed 0.2 mm internal to the DEJ, are deeper gingivally (0.4 mm) than occlusally (i.e., they fade out as they extend occlusally, and translate parallel to the DEJ). If the axial wall is deeper than normal, the retention lock is not placed at the axiofacial or axiolingual line angles but, rather, is positioned 0.2 mm internal to the DEJ. If placed at the deeper location, it may result in pulp exposure, depending on the location of the axial wall depth

The inorganic component of subgingival plaque is derived from _____. A. Bacteria B. Saliva C. Gingival crevicular fluid D. Neutrophils

C, Saliva is the source of inorganic components (calcium, phosphorous) for supragingival plaque. Gingival crevicular fluid is the source of inorganic components of subgingival plaque.

Offset angulation is a characteristic feature of _____. A. Sickle scalers B. Universal curettes C. Area-specific curettes D. Chisels

C, Sickle scalers and universal curettes do not have offset angulation of the blade. The working ends of area-specific curettes are offset at a 60-degree angle relative to the terminal shank. The working ends of sickle scalers and universal curettes are not offset—they are at a 90-degree angle relative to the terminal shank

X-ray film is composed of _____. A. Silver halide crystals suspended in plastic and coated on a gelatin base B. Sodium thiosulphate crystals and suspended within a plastic base C. A plastic base coated with silver halide crystals suspended in gelatin D. Fluorescent particles that react to x-radiation

C, Silver halide is not fluorescent, and thus choices 1 and 2 are incorrect

Doubling the force applied at the bracket of a tooth would have what effect on the moment affecting tooth movement? A. The moment would decrease by 50%. B. The moment would not change. C. The moment would double. D. The moment would increase by 4 times.

C, Since M = Fd, doubling the force would double the moment, or tendency to rotate, tip, or torque

Doubling the force applied at the bracket of a tooth would have what effect on the moment affecting tooth movement? A. The moment would decrease by 50%. B. The moment would not change. C. The moment would double. D. The moment would increase by four times

C, Since M = Fd, doubling the force would double the moment, or tendency to rotate, tip, or torque.

Which of the following is true regarding the possibilities for reconstruction of an atrophic edentulous ridge prior to denture construction? A. Dental implants are used only as a last resort after bone grafting attempts have failed. B. Distraction osteogenesis is too new a technique to be applied to ridge augmentation. C. Potential bone graft harvest sites for ridge reconstruction include rib, hip, and chin. D. The need for ridge augmentation is more common in the maxilla than in the mandible

C, Sites commonly used for the reconstruction of the atrophic mandibular ridge are dictated by the deficiency and include chin, hip, ribs, prosthetic materials, and donor bone (human and bovine). Dental implants are commonly used, not only as a last resort. The use of distraction of ridge augmentation has been reported and is useful in certain applications. The mandibular alveolar ridge is more problematic in terms of resorption and denture retention, which more commonly necessitates reconstructive measures

The following are characteristics of a postpalatal seal of complete dentures, except which one? A. Compensates for shrinkage of the acrylic resin caused by its processing. B. May reduce the gag reflex. C. Improves the stability of the maxillary denture. D. It is most shallow in the midpalatal suture area

C, Stability is resistance to movement toward the residual ridge. The function of the posterior palatal seal is to improve retention, not stability. Stability is determined by the size, height, or shape of the ridge

Systemic effects of obstructive sleep apnea syndrome (OSAS) include all of the following except _____. A. Hypertension B. Cor pulmonale C. Aortic aneurysm D. Cardiac arrhythmia

C, Systemic sequelae of OSAS include hypertension, Cor Pulmonale, and cardiac arrhythmia.

Which of the following is the primary reason for splinting teeth? A. For esthetics B. To improve hygiene C. For patient comfort D. As a preventive measure

C, Teeth are usually splinted to improve patient comfort during mastication

Factors that affect the success of dentin bonding include all of the following except _____. A. Dentin factors such as sclerosis, tubule morphology, and smear layer B. Tooth factors such as attrition, abrasion, and abfraction C. Material factors such as compressive and tensile strengths D. C-factor considerations

C, Tensile and compressive strengths may have relevance for composite materials but not for dentin bonding systems. The success of bonding is dependent on the various dentin structural factors, tooth factors, polymerization shrinkage, C-factor considerations, and technique sensitivity

The most common mandibular surgical osteotomy to advance the mandible is _____. A. A LeFort I osteotomy B. A segmental maxillary osteotomy C. A bilateral sagittal split osteotomy D. An intraoral vertical ramus osteotomy

C, The BSSO is the most commonly used osteotomy for mandibular advancement

Placing the toothbrush bristles at a 45-degree angle on the tooth and pointing apically so the bristles enter the gingival sulcus describes which brushing technique? A. Charter B. Stillman C. Bass D. Roll

C, The Bass technique of brushing is designed to direct the bristles of the brush toward the gingival sulcus.

According to the CDC, the acceptable water quality in a dental office should be _____. A. < 125 CFU/mL B. < 250 CFU/mL C. < 500 CFU/mL D. < 750 CFU/mL E. < 1000 CFU/mL

C, The CDC recommends, at a minimum to meet nationally recognized drinking water standards, less than 500 colony-forming units (CFUs) of heterotrophic bacteria per milliliter. In 1995, the ADA addressed the dental water concern by asking manufacturers to provide equipment with the ability to deliver treatment water with < 200 CFU/ml of unfiltered output from waterlines

Miller Class I recession defects can be distinguished from Class II defects by assessing the _____. A. Location of interproximal alveolar bone B. Width of keratinized gingiva C. Involvement of the mucogingival junction D. Involvement of the free gingival margin

C, The Miller classification system for mucogingival defects takes into consideration the degree of recession (whether or not it extends to the mucogingival junction) and presence or absence of bone loss in the interdental area. Both Class I and Class II defects are characterized by no loss of bone in the interproximal areas. In Class I defects, the marginal tissue recession does not extend to the mucogingival junction. In Class II defects, recession does extend to or beyond the mucogingival junction

The Schwann cell is the cell of origin for which of the following tumors? A. Odontogenic myxoma B. Rhabdomyoma C. Neurofibroma D. Mixed tumor E. Leiomyoma

C, The Schwann cell is of neural origin and gives rise to one of several neoplasms, including neurofibroma and Schwannoma

Which statement is false regarding child pain management? A. It is recommended that a dentist provide specific direction and praise for cooperation. B. Children do not have a fully developed sense of time; therefore, it is recommended that one use more concrete measures of time such as counting or a visible timer (e.g., egg timer). C. The Tell-Show-Do technique is no longer recommended since it has been demonstrated to increase anxiety and reports of pain during treatments. D. In order to enhance the sense of control, it is recommended that one establish a hand signal signifying distress and a desire for the dentist to stop or take a break. E. Once a child begins to complain about poor pain control, the chances of having a successful visit drop dramatically.

C, The Tell-Show-Do method, in which the clinician explains, demonstrates, and allows a child (or an adult patient) to learn and understand what will be happening before proceeding, contributes to decreased self-reports of anxiety and pain

Which of the following is most important in determining the prognosis for a tooth? A. Probing pocket depth B. Bleeding on probing C. Clinical attachment level D. Level of alveolar bone

C, The amount of clinical attachment loss is most important in determining the prognosis. Deep pocket depths and bleeding on probing can be found in both gingivitis and periodontitis. Although the level of alveolar bone is usually consistent with the amount of clinical attachment loss, there are circumstances under which these two measures are not comparable.

A patient is administered haloperidol. Along with the haloperidol, the patient also receives benztropine. What is the most likely reason for administering the benztropine? A. To reduce the effects of histamine release B. To aid in the therapeutic response to haloperidol C. To reduce the motor adverse effects of haloperidol D. To overcome a decrease in salivary flow resulting from haloperidol E. To reduce the rate of kidney excretion of haloperidol

C, The antimuscarinic action of benztropine tends to reduce the Parkinsonlike symptoms and some other motor symptoms caused by haloperidol, a dopamine receptor blocker. It does not improve the antipsychotic effect of haloperidol. Histamine release appears to play little role in this interaction. Benztropine actually reduces salivary flow and xerostomia can easily result from its administration. Benztropine has little effect on renal clearance of haloperidol

Your patient is 8 years old. Tooth #8 was avulsed and you replanted it within 30 minutes. What is the best splint to use? A. Rigid fixation for 7 days B. Rigid fixation for 2 months C. Nonrigid fixation for 7 days D. Nonrigid fixation for 2 months

C, The appropriate splint for an avulsed tooth is a nonrigid splint, which is left in place for about 7 to 14 days. A 0.016 × 0.022 stainless steel orthodontic wire, a 0.018 round stainless steel wire, and a monofilament nylon (20- to 30-lb test) line are considered nonrigid. Long-term rigid splinting of replanted teeth increases risk of replacement root resorption (ankylosis). Rigid splinting is indicated for root fractures and remains in place for 2 to 3 months. A 0.032-0.036 stainless steel wire is considered a rigid splint.

It is important that the film base be _____. A. Opaque B. Very rigid C. Flexible D. Completely clear E. Sensitive to x-rays

C, The base needs to be flexible to go through automatic processors and be put into film mounts. Usually, the base is not completely clear and it is the emulsion that is sensitive to x-rays

Polycarboxylate cement achieves a chemical bond to tooth structure. The mechanism for this bond is _____. A. Ionic bond to phosphate. B. Covalent bond to the collagen. C. Chelation to calcium. D. These cements do not form a chemical bond

C, The carboxylate groups in the polymer molecule chelates to calcium.

From the following list of components of an RPD, which must be rigid? A. Major connector, minor connector, and retentive clasp B. Wrought wire clasp, rests, and minor connector C. Minor connector, rest, and major connector

C, The clasps are meant to be flexible in order to engage in undercut. The rest of the components of an RPD should be rigid

For an occlusal appliance used for muscle relaxation to be effective, the condyles must be located in their most stable position from a musculoskeletal perspective. This is _____. A. Centric occlusion B. At the vertical dimension of rest C. Centric relation D. Maximum intercuspal position

C, The condyles should be in centric relation, which is defined as "the maxillomandibular relationship in which the condyles articulate with the thinnest avascular portion of their respective disks with the condyle-disk complex in the anterior-superior position against the shapes of the articular eminences." (Glossary of Prosthodontic Terms, J Prosthetic Dent 94(1):21-22, 2005.)

A correlation analysis shows that as the income of the population increases, the number of decayed teeth decreases. Therefore, an expected value for this correlation coefficient (r) would be _____. A. 0 B. 1 C. −1 D. 2 E. −2

C, The correlation coefficient (r) quantifies the relationship between variables (x and y). A positive correlation coefficient indicates that the variables increase in the same direction; a negative correlation coefficient indicates that the variables vary in opposite directions. The correlation coefficient ranges from −1 to +1.

Which drug blocks H1 histamine receptors but is least likely to cause sedation? A. Diphenhydramine B. Hydroxyzine C. Fexofenadine D. Albuterol E. Famotidine

C, The first three choices are all H1 histamine receptor blockers. Fexofenadine, however, is largely excluded from the central nervous system, unlike diphenhydramine and hydroxyzine. Albuterol is a β2 adrenergic receptor agonist. Famotidine is a H2 histamine receptor antagonist

How many human drug testing phases are carried out before a drug is marketed? A. One B. Two C. Three D. Four

C, The fourth phase constitutes postmarketing surveillance.

When the gingival margin is gingival to the CEJ in a Class II amalgam preparation, the axial depth of the axiogingival line angle should be _____. A. 0.2 mm into sound dentin B. Twice the diameter of a No. 245 carbide bur C. 0.75 to 0.80 mm D. The width of the cutting edge of a gingival marginal trimmer

C, The guide for axial wall depth for a typical Class II preparation that has a gingival margin occlusal to the CEJ is 0.2 to 0.5 mm internal to the DEJ— the greater depth is necessary when placing retention locks. However, when there is no enamel proximally, the axial wall needs to be deep enough internally to provide for adequate strength of the amalgam material as well as to have room to place retention locks, if needed. This depth is approximately 0.75 mm.

Your patient indicates that he is taking medication for atrial fibrillation. He reports that a blood test has indicated that he has an INR number of 4.0. An emergency dental extraction is now required. Which postoperative medication would pose the greatest risk for an adverse effect in this patient? A. Acetaminophen B. Amoxicillin C. Aspirin D. Codeine E. Ibuprofen

C, The international normalized ratio (INR) value indicates that the patient has received anticoagulant therapy for his atrial fibrillation. Aspirin increases the risk of postsurgical bleeding. The combination of increase in prothrombin time, surgery, and the antiplatelet effect of aspirin make aspirin contraindicated in this situation. Ibuprofen's effect on the platelet is reversible, whereas the effect of aspirin on the platelet is irreversible. Thus, aspirin poses a greater risk than does ibuprofen in this situation

Your patient is a 23-year-old college student whom you suspect may have sustained a mandible fracture during an altercation. Which of the following is false? A. At least two x-rays should be obtained. B. The most common x-ray obtained would be a panoramic radiograph. C. The most likely area for this patient's mandible to be fractured is the mandibular dental alveolus. D. Point tenderness, changes in occlusion, step deformities, and gingival lacerations should all be noted on physical exam.

C, The mandibular condyle is the most common location of mandibular fractures. The alveolus, ramus, and coronoid are the least common sites

Which is true of a minor connector of an RPD? A. Should be thin to not interfere with the tongue B. Should be located on a convex embrasure surface C. Should conform to the interdental embrasure D. All of the above E. A and C only

C, The minor connector must have sufficient bulk to be rigid so that it transfers functional stresses effectively to the abutment or supporting teeth and tissues. It should be located in the interdental embrasure where it doesn't disturb the tongue, and should be thickest in the lingual surface, tapering toward the contact area but not located on a convex surface.

The third molar impaction most difficult to remove is the _____. A. Vertical B. Mesioangular C. Distoangular D. Horizontal

C, The most difficult impaction to remove is the distoangular tooth. This is because the withdrawal pathway runs into the ramus of the mandible and requires greater surgical intervention

Which of the following statements regarding post preparation is incorrect? A. The primary purpose of the post is to retain a core in a tooth with extensive loss of coronal structure. B. The need for a post is dictated by the amount of remaining coronal tooth structure. C. Posts reinforce the tooth and help to prevent vertical fractures. D. At least 4 to 5 mm of remaining gutta-percha after post space preparation is recommended.

C, The most important part of the restored tooth is the tooth itself. No combination of restorative materials can substitute for tooth structure. Posts do not reinforce the tooth but, rather, further weaken it by additional removal of dentin and by creating stress that predisposes to root fracture.

Which of the following is the most likely cause of pulpal necrosis following trauma to a tooth? A. Ankylosis B. Calcific metamorphosis C. Pulpal hyperemia D. Dilaceration

C, The other three answers may occur as the result of trauma but do not cause loss of vitality. Pulpal hyperemia causes increased intrapulpal pressure and swelling, which may result in an interruption of the pulp's blood supply. Without an adequate blood supply, the pulp becomes necrotic. This process can take time, and symptoms (either radiographic or clinical) may not present for weeks or even months. Typically, follow-up examination and radiographs are indicated at 1-, 2-, and 6-month intervals following a traumatic incident.

Which of the following is the most likely cause pulpal necrosis following trauma to a tooth? A. Ankylosis B. Calcific metamorphosis C. Pulpal hyperemia D. Dilaceration

C, The other three answers may occur as the result of trauma but do not cause loss of vitality. Pulpal hyperemia causes increased intrapulpal pressure and swelling, which may result in an interruption of the pulp's blood supply. Without an adequate blood supply, the pulp becomes necrotic. This process can take time, and symptoms (either radiographic or clinical) may not present for weeks or even months. Typically, follow-up examination and radiographs are indicated at 1-, 2-, and 6- month intervals following a traumatic incident

All of the following describe lidocaine as packaged in dental cartridges except _____. A. Provided in a 2% solution B. Provided with or without epinephrine C. Has a pKa = 8.1 D. Has a rapid onset

C, The pKa of lidocaine is 7.9. It is packaged as a 2% solution both with and without epinephrine and has a rapid onset of action.

Causes of postoperative sensitivity with amalgam restorations include all of the following except _____. A. Lack of adequate condensation, especially lateral condensation in the proximal boxes. B. Voids. C. Extension onto the root surface. D. Lack of dentinal sealing.

C, The primary causes of postoperative sensitivity for amalgam restorations are voids (especially at the margins), poor condensation (that may result in a void), or inadequate dentinal sealing. Extension onto the root surface does not necessarily result in increased sensitivity

Causes of postoperative sensitivity with amalgam restorations include all of the following except _____. A. Lack of adequate condensation, especially lateral condensation in the proximal boxes B. Voids C. Extension onto the root surface D. Lack of dentinal sealing

C, The primary causes of postoperative sensitivity for amalgam restorations are voids (especially at the margins), poor condensation (that may result in void), or inadequate dentinal sealing. Extension onto the root surface does not necessarily result in increased sensitivity

The primary reason for designing a surgical flap with a wide flap base is _____. A. To avoid incising over a bony protuberance B. To obtain maximum access to the surgical site C. To maintain an adequate blood supply to the reflected tissue D. To aid in complete reflection

C, The principles of flap design include the following: (1) flap design should ensure adequate blood supply and the base of the flap should be wider than the apex; (2) reflection of the flap should adequately expose the operative field; and (3) flap design should permit atraumatic closure of the wound

The distance between the major connector on a maxillary RPD framework and the gingival margins should be at least _____. A. 3 mm B. 2 mm C. 6 mm D. 15 mm

C, The recommended space or distance between the border of the framework and the marginal gingiva should be at least 6 mm

The component of an RDP that is spoon-shaped and slightly inclined apically from the marginal ridge of a tooth is the _____. A. Indirect retainer B. Minor connector C. Rest D. Lingual bar

C, The rest should be spoon-shaped and is slightly inclined apically from the marginal ridge of the abutment tooth. It should restore the occlusal morphology of the tooth and not interfere with the normal existing occlusion

Which of the following statements is true regarding the choice between doing a composite or amalgam restoration? A. Establishing restored proximal contacts is easier with composite. B. The amalgam is more difficult and techniquesensitive. C. The composite generally uses a more conservative tooth/cavity preparation. D. Amalgam should be used for Class II restorations.

C, The restoration of a proximal contact is easier with amalgam than composite. Amalgam is easier to use and is less technique-sensitive. Either material can be used for Class II restorations. Because an amalgam restoration requires a tooth preparation that has (1) a specified depth (for strength of the amalgam), (2) cavosurface marginal configurations that result in 90-degree amalgam margins, and (3) undercut form to its walls or secondary retention form features, they require more tooth structure removal than do composite tooth preparations. Composite tooth preparations require (1) removal of the fault, defect, or old material; (2) removal of friable tooth structure; and (3) no specific depths—they are more conservative.

Which of the following statements is true regarding the choice between doing a composite or amalgam restoration? A. Establishing restored proximal contacts is easier with composite. B. The amalgam is more difficult and techniquesensitive. C. The composite generally uses a more conservative tooth/cavity preparation. D. Only amalgam should be used for Class II restorations

C, The restoration of a proximal contact is easier with amalgam than with composite. Amalgam is easier to use and is less technique-sensitive. Either material can be used for Class II restorations. Because an amalgam restoration requires a tooth preparation that has (1) a specified depth (for strength of the amalgam), (2) cavosurface marginal configurations that result in 90-degree amalgam margins, and (3) an undercut form to its walls or secondary retention form features, they require more tooth structure removal than do composite tooth preparations. Composite tooth preparations require (1) removal of the fault, defect, or old material, (2) removal of friable tooth structure, and (3) no specific depths— they are more conservative

The size of the x-ray tube focal spot influences radiographic _____. A. Density B. Contrast C. Resolution D. Magnification E. Both C and D

C, The smaller the focal spot size, the greater the resolution. Density, contrast, and magnification are unchanged, other factors remaining equal

A wire extending from the molars to the incisors is activated to intrude the incisors. What is the side effect on the molars? A. The molars will tip forward and intrude B. The molars will rotate mesiobuccally C. The molars will tip distally and extrude D. The molars will rotate distobuccally

C, The sum of the forces and moments on an appliance must equal zero. If the incisors intrude, the molars will extrude. These two forces form a couple with a moment in one direction. The molars will experience a couple in the opposite direction, which will cause them to tip distally

Trauma from occlusion refers to _____. A. The occlusal force B. The damage to the tooth C. The injury to the tissues of the periodontium D. The widened periodontal ligament

C, The term trauma from occlusion refers to the tissue injury that occurs when occlusal forces exceed the adaptive capacity of the tissues. An occlusion that produces such an injury is called a traumatic occlusion. The tooth may become damaged as a result of excessive occlusal forces. The periodontal ligament also may become widened as a result of the force

What is the clinical setting for the use of ketorolac by the oral route? A. For severe pain B. For initial treatment of pain C. To continue therapy after an IV or IM dose of ketorolac D. Only in combination with an opioid E. Only in combination with an NSAID

C, The use of oral ketorolac (an NSAID) is limited to continue therapy after a parenteral dose

The variance for data set A is 25 and for data set B is 9. Therefore, we can conclude _____. A. There are more items in data set A than data set B B. The mean of data set B is smaller than the mean for data set A C. The items in data set A are more widely spread about the mean value than in data set B D. The standard deviation for data set B is larger than for data set A E. None of the above

C, The variance determines the way individual values are located around the mean. The larger the variance, the more widely the data items are spread about the mean value. Variance is measured in squared units (s2). The standard deviation is the square root of the variance. The mean is expressed in the same units as the data items, but the variance is expressed in squared units. Thus, the standard deviation measures the average deviation from the mean in the same units as the mean

The modified Widman flap uses three separate incisions. It is reflected beyond the mucogingival junction. A. Both statements are true. B. Both statements are false. C. First statement is true. Second statement is false. D. First statement is false. Second statement is true.

C, Three incisions are made in the modified Widman flap—internal bevel, crevicular, and interdental. It is designed to provide exposure of the tooth roots and alveolar bone. However, the flap is not reflected beyond the mucogingival junction

Which class of bony defect responds best to regenerative therapy? A. One-walled B. Two-walled C. Three-walled D. Shallow crater

C, Three-walled defects respond best to regenerative therapy.

Your patient is continually taking a small daily dose of aspirin (82 mg) prescribed by the patient's physician. The object of this therapy is most likely what mechanism? A. To mimic the effect of endogenous endorphins B. To inhibit the production of prostaglandin E1 C. To inhibit the production of thromboxane A2 D. To inhibit the production of arachidonic acid E. To inhibit the production of leukotrienes

C, Thromboxane A2 increases platelet aggregation. Its inhibition is the target of low-dose aspirin which inhibits cyclo-oxygenase. Inhibition of this enzyme leads to a reduction in important down stream products, including thromboxane A2

Which of the following is not a vital part of the physical exam for patients with TMJ complaints? A. Soft-tissue symmetry B. Joint tenderness and sounds C. Soft-palate length D. Range of motion of the mandible E. Teeth

C, Tissue symmetry, tenderness, joint noises dental health and occlusion and range of motion are all critical components of the physical exam in the TMJ patient. Although the length of the soft palate is important in the evaluation of patients with sleep apnea, snoring, patients being sedated, or patients needing complete denture construction, it does not contribute directly to TMJ dysfunction.

Excessive monomer added to acrylic resin will result in _____. A. Increased expansion B. Increased heat generation C. Increased shrinkage D. Increased strength

C, Using more monomer than needed will cause increased shrinkage. The more monomer used, the less expansion, less heat, and reduced strength will be produced

An endodontically treated tooth was restored with a cast post-and-core and a metal-ceramic crown. Three months later, the patient complains of pain, especially on biting. Radiographic findings and tooth mobility tests are normal. The most probable cause of pain is _____. A. A loose crown B. Psychosomatic C. A vertical root fracture D. A premature eccentric contact

C, Usually, vertical fractures will refer pain when biting. In this case, the patient had recent endodontic treatment and there is no periapical lesion to indicate that is due to inadequate root canal therapy. There is no sign that the crown is loose, no premature contact, and no mobility.

There is a horizontal root fracture in the middle third of the root of tooth 10 in an 11-yearold patient. The tooth is mobile and vital. How should this be treated? A. Extract. B. Pulpectomy immediately and splint. C. Splint and observe. D. Do nothing and follow-up in 10 to 14 days.

C, When a root fractures horizontally, the coronal segment is displaced to a varying degree, but generally the apical segment is not displaced. Because the apical pulpal circulation is not disrupted, pulp necrosis in the apical segment is extremely rare. Pulp necrosis in the coronal segment results because of its displacement and occurs in only about 25% of cases. Because 75% do not lose vitality, emergency treatment involves repositioning the segments in as close proximity as possible and splinting the teeth for 2 to 4 weeks. After the splinting period is completed, follow-up is as with all dental traumatic injuries, at 3, 6, and 12 months and then yearly thereafter

Which of the following is considered a reversible carious lesion? A. The lesion surface is cavitated. B. The lesion has advanced to the dentin radiographically. C. A white spot is detected upon drying. D. The lesion surface is rough or cha

C, When an alteration (a break in continuity) occurs to the tooth surface from a carious attack, restoration is usually necessary. When a lesion is evident in the dentin with an x-ray, the lesion usually needs a restoration

During a routine 6-month endodontic treatment recall evaluation, you note a marked decrease in the radiographic size of the periradicular radiolucency. Which of the following is the most appropriate treatment plan? A. Extraction. B. Nonsurgical endodontic retreatment. C. Recall the patient in another 6 months. D. Surgical endodontic retreatment

C, When endodontic treatment is done properly, healing of the periapical lesion usually occurs with osseous regeneration, which is characterized by gradual reduction and resolution of the radiolucency on follow-up radiographs. The rate of bone formation is slow, and complete resolution may take longer than the standard 6-month followup, especially with elderly patients. As long as the radiolucency appears to be resolving as opposed to enlarging, an extended re-evaluation is in order

At what stage is endodontic treatment considered complete? A. When a temporary restoration is placed and the rubber dam removed. B. When canals are seared off and plugged. C. When the coronal restoration is completed. D. When the patient is asymptomatic.

C, When the coronal restoration is completed. After root canal therapy, the canals inside the roots have been cleaned and permanently sealed. However, there is a temporary filling in the outer surface of the tooth. The patient must be told that they need a permanent filling or crown for the tooth. This is very important for the protection of the tooth against fracture or reinfection of the root canal

Your examination reveals a probing pocket depth of 6 mm on the facial of tooth 30. The free gingival margin is 2 mm apical to the CEJ (there is 2-mm recession on the facial). How much attachment loss has there been on the facial of this tooth? A. 6 mm B. 2 mm C. 8 mm D. 4 mm

C, When the free gingival margin is apical to the CEJ, recession has occurred. Attachment loss is the measure from the CEJ to the base of the periodontal pocket. With the free gingival margin 2 mm apical to the CEJ and the probing pocket depth measurement 6 mm, there has been 8 mm loss of attachment.

The reason to invert a rubber dam is _____. A. To prevent the dam from tearing B. To prevent the underlying gingival from accidental trauma C. To provide a complete seal around the teeth D. All of above

C, When the rubber dam edge around the tooth is turned gingivally (inverted), it significantly reduces the leakage of moisture occlusally, thereby sealing around the tooth better and resulting in a better isolated operating area

Bremsstrahlung radiation results from _____. A. X-rays interacting with electrons B. Electrons interacting with electrons C. Electrons interacting with nuclei D. L shell electrons falling into the K shell E. Photons interacting with nuclei F. Photons converting into electrons

C, X-ray photons (Bremsstrahlung radiation) results from the interaction of high-speed electrons with tungsten nuclei in the target

X-rays are produced in most conventional dental x-ray machines _____. A. Continuously during operation B. When there is a large space charge C. Half the time during operation D. When the anode carries a negative charge E. Only when the beam is collimated F. Only during the first half of each second

C, X-rays are produced in most dental x-ray machines half the time (i.e., in bursts at the rate of 60 per second, each lasting 1/120th second) due to the alternating current supplied to the tube.

According to Malamed, how many cartridges of 2% lidocaine can be safely administered to a child weighing 40 lb? A. Three cartridges B. One cartridge C. Nine cartridges D. Two cartridges

D, 2% lidocaine contains 36 mg of lidocaine per cartridge. Since 80 mg is the amount of lidocaine that can safely be administered to this child, the number of cartridges that can be administered is 80 mg divided by 36 mg per cartridge, which is roughly two cartridges

Three cartridges of 2% lidocaine with 1:100,000 epinephrine contain _____ lidocaine. A. 36 mg B. 54 mg C. 54 μg D. 108 mg

D, A 2% solution of any drug contains 20 mg/mL, by definition. A dental cartridge of local anesthesia has a fluid volume of 1.8 mL. 20 mg × 1.8 = 36 mg of lidocaine per cartridge. Three cartridges of 2% lidocaine with 1:100,000 epinephrine therefore contain 108 mg

A decrease in glycogenolysis in the liver would be expected from which drug? A. Albuterol B. Epinephrine C. Glucagon D. Insulin E. Parathyroid hormone

D, A decrease in glycogen breakdown is a classic effect of insulin. Epinephrine (by acting as an agonist at α1- and β2-adrenergic receptors), albuterol (by acting as an agonist at β2-adrenergic receptors), and glucagon (by acting at glucagon receptors) all tend to increase glycogen. Parathyroid hormone has little effect on glycogenolysis.

A "skirt" feature for a gold onlay preparation _____. A. Has a shoulder gingival margin design B. Is prepared by a diamond held perpendicular to the long axis of the crown C. Is used only for esthetic areas of a tooth D. Increases both retention and resistance forms

D, A skirt is a "mini-crown" preparation around a line angle. It should be prepared by a diamond instrument in the long axis of the tooth crown, extended to the gingival one-third, and result in an appropriate amount of tooth removal. It is placed to increase both retention form (having opposing skirt vertical walls retentive with each other) and resistance form (enveloping the line angles like a barrel hoop around a barrel). It extends the outline form and therefore may be least appropriate for highly esthetic areas in the mouth

The tooth preparation technique for a Class I amalgam on a mandibular first molar does not include which of the following? A. Maintaining a narrow isthmus width B. Initial punch cut placed in the most carious pit C. Establishment of pulpal depth of 1.5 to 2 mm D. Orientation of bur parallel to the long axis of the tooth

D, A tooth preparation for a mandibular molar should have a narrow isthmus, should be initiated in the most carious (or distal) pit, and should establish the initial pulpal floor depth of 1.5 to 2 mm. However, it should be oriented parallel to the long axis of the crown, which tilts to the lingual. If prepared in the long axis of the tooth, there is greater potential of weakening the lingual cusps.

Diffuse soft swelling of the lips and neck following the ingestion of drugs, shellfish, or nuts is known as _____. A. Fixed drug reaction B. Anaphylaxis C. Urticaria D. Acquired angioedema E. Contact allergy

D, Acquired angioedema is a rapidly developing allergic reaction that results in characteristic nonerythematous swelling of lips, face, and neck

Which of the following chemical agents is not a disinfectant? A. Iodophors B. Sodium hypochlorite C. Synthetic phenol D. Isopropyl alcohol E. Glutaraldehyde

D, Alcohol is not an accepted disinfectant. Alcohol evaporates too quickly to be an effective disinfectant. The term disinfection is reserved for chemicals applied to inanimate surfaces, and the word antiseptic is used for antimicrobial agents (such as alcohol) that are applied to living tissues

Which injection anesthetizes the distobuccal aspect of the mandibular first molar? A. Posterior superior alveolar (PSA) B. Middle superior alveolar (MSA) C. Anterior superior alveolar (ASA) D. Inferior alveolar (IA)

D, All mandibular molars are anesthetized by the inferior alveolar nerve block. The other three answers in the question are maxillary injections

Which two drugs have mechanisms of analgesic action that are most similar? A. Fentanyl, ibuprofen B. Aspirin, codeine C. Oxycodone, acetaminophen D. Ibuprofen, naproxen E. Aspirin, ibuprofen

D, All of the choices are combinations of an opioid and an inhibitor of cyclo-oxygenase (COX), except two: ibuprofen, naproxen and aspirin, ibuprofen. Ibuprofen and naproxen are both reversible inhibitors of COX, and are propionic acid derivatives. Aspirin is a salicylate and is an irreversible inhibitor.

A cervical lesion should be restored if it _____. A. Is carious B. Is very sensitive C. Is causing gingival inflammation D. All of the above

D, All of these factors indicate a cervical lesion should be restored. In addition, if the lesion is large and the pulpal or gingival tissues are in jeopardy, it should be considered for restoration.

A cervical lesion should be restored if it is _____. A. Carious. B. Very sensitive. C. Causing gingival inflammation. D. All of the above.

D, All of these factors indicate that a cervical lesion should be restored. In addition, if the lesion is large and the pulpal or gingival tissues are in jeopardy, it should be considered for restoration

The purpose of fabricating a provisional restoration with correct contours and marginal integrity is _____. A. For protection B. To supervise the patient's dental hygiene and give them feedback during this stage C. To preserve periodontal health D. All of the above

D, All these reasons are correct. The provisional is placed to protect the tooth and preserve healthy tissues if proper contours and marginal integrity are present. This is an excellent time to evaluate and give feedback to the patient on how well they are brushing and flossing.

A good preventive and treatment strategy for dental caries would include _____. A. Limiting cariogenic substrate B. Controlling cariogenic flora C. Elevating host resistance D. All of the above

D, Altering the organism, its nutrients, and its environment will all enhance prevention and treatment objectives.

Which of the following is the primary etiologic factor associated with periodontal disease? A. Age B. Gender C. Nutrition D. Bacterial plaque

D, Although age, gender, and nutrition may have an impact on periodontal disease, the accumulation of the bacterial plaque biofilm is the primary initiator of the disease

Defects in which inflammatory cell have most frequently been associated with periodontal disease? A. The T-lymphocyte B. The mast cell C. The plasma cell D. The neutrophil

D, Although defects in any of the host defense cells could impact periodontal disease susceptibility, defects in neutrophils have been most frequently described.

Which of the following is a potential sequela of an acute periapical abscess? A. Central giant cell granuloma B. Peripheral giant cell granuloma C. Osteosarcoma D. Periapical granuloma E. Periapical cemento-osseous dysplasia

D, An acute exudate (pus) at the apex of a tooth will follow the path of least resistance (e.g., into surrounding bone, gingiva, or skin). If the offending tooth is not treated and the abscess becomes chronic, a periapical granuloma may result

Heating the metal structure in a furnace prior to opaque application in a metal-ceramic crown is necessary to _____. 1. Harden the metal. 2. Oxidize trace elements in the metal. 3. Eliminate oxidation. A. 1 only B. 1 and 2 C. 1 and 3 D. 2 only E. 3 only

D, An important factor that affects the metal-ceramic bond is the surface treatment of the alloy before firing porcelain. Air-abrasion of the cast alloy is typically performed before the oxidation step to help remove surface contaminants that remain from devesting, and to help clean the casting and provide microscopic surface irregularities for mechanical retention of the ceramic. The oxidation step for the alloy can be performed in air or by using the reduced atmospheric pressure (approximately 0.1 atm) available in dental porcelain furnaces

An edentulous patient with a diminished vertical dimension of occlusion is predisposed to suffer from which of the following conditions? A. Epulis fissuratum B. Pemphigus vulgaris C. Papillary hyperplasia D. Angular chelosis

D, Angular chelosis is described as inflamed and cracked corners of the mouth that can become infected with bacteria and fungal organisms. It is commonly seen in denture patients with diminished vertical dimension of occlusion. It is best treated with antifungal creams and correcting the vertical dimension of occlusion

What behavior can you typically expect from an anxious patient in the waiting room? A. He or she is more likely to sit still, hands clasped together. B. He or she is more likely to sit casually, legs crossed, reading a magazine. C. He or she is more likely to keep to himself or herself and not speak unless spoken to. D. He or she is more likely to fidget in the chair, moving his or her hands and feet. E. Both A and C.

D, Anxious patients are typically more likely to fidget in their chairs, unable to focus on a task such as reading or relaxing in the waiting room.

Before the exploration of any intrabony pathologic lesion, which type of biopsy must always be done? A. Cytologic smear B. Incisional biopsy C. Excisional biopsy D. Aspiration biopsy

D, Any radiolucent lesion that requires biopsy should undergo aspiration before surgical exploration. This procedure may yield material for biopsy, and will rule out a vascular lesion (e.g., AV malformation), which could be dangerous to enter without prior diagnosis

Which of the following local anesthetics has the shortest half-life? A. Lidocaine B. Prilocaine C. Bupivacaine D. Articaine

D, Articaine has an ester bond and an amide bond. Since esters are biotransformed much more rapidly than amides, articaine has a much shorter halflife than the others

Choose the incorrect statement about Class V amalgam restorations. A. The outline form is usually kidney- or crescentshaped. B. Because the mesial, distal, gingival, and incisal walls of the tooth preparation are perpendicular to the external tooth surface, they usually diverge facially. C. Using four corner coves instead of two full-length grooves conserves dentin near the pulp and may reduce the possibility of a mechanical pulp exposure. D. If the outline form approaches an existing proximal restoration, it is better to leave a thin section of tooth structure between the two restorations (< 1 mm) than to join the restorations.

D, Because of the typical shape of a carious lesion in the cervical area, the resulting restoration is kidney- or crescent-shaped and the extensions are to the line angles, resulting in the mesial and distal walls diverging externally. The convexity of the tooth in the gingival one third results in the occlusal and gingival walls diverging externally. There are several retention groove designs that are appropriate, including four corner coves, occlusal and gingival line angle grooves, or circumferential grooves. However, as with any restoration, if there is only a small amount of tooth structure (< 1 mm) between the new and existing restoration, it is best to join the two restorations together and prevent the possibility of fracture of the small amount of remaining tooth structure.

Choose the incorrect statement about Class V amalgam restorations. A. The outline form is usually kidney- or crescentshaped. B. Because the mesial, distal, gingival, and incisal walls of the tooth preparation are perpendicular to the external tooth surface, they usually diverge facially. C. Using four corner coves instead of two fulllength grooves conserves dentin near the pulp and may reduce the possibility of a mechanical pulp exposure. D. If the outline form approaches an existing proximal restoration, it is better to leave a thin section of tooth structure between the two restorations (< 1 mm) than to join the restorations

D, Because of the typical shape of a carious lesion in the cervical area, the resulting restoration is kidney- or crescent-shaped and the extensions are to the line angles, resulting in the mesial and distal walls diverging externally. The convexity of the tooth in the gingival one-third results in the occlusal and gingival walls diverging externally. There are several retention groove designs that are appropriate, including four corner coves, occlusal and gingival line angle grooves, or circumferential grooves. However, as with any restoration, if there is only a small amount of tooth structure (< 1 mm) between the new and existing restoration, it is best to join the two restorations together and prevent the possibility of fracture of the small amount of remaining tooth structure.

A 22-year-old college student presents with oral pain, erythematous gingival tissues with blunt papillae covered with a pseudomembrane, spontaneous gingival bleeding, and halitosis. There is no evidence of clinical attachment loss. What form of periodontal disease does this patient most likely have? A. Gingivitis associated with dental plaque B. Localized aggressive periodontitis C. Generalized chronic periodontitis D. Necrotizing ulcerative gingivitis

D, Because there is no loss of attachment, the diagnosis would not be periodontitis. The clinical description of pain, erythema, blunt papillae, pseudomembrane, and halitosis is consistent with necrotizing ulcerative gingivitis

Which drug lacks the amine group that other anesthetics have and is used only topically? A. Procaine B. Mepivacaine C. Lidocaine D. Benzocaine E. Prilocaine

D, Benzocaine lacks the amine group that procaine, mepivacaine, prilocaine, and lidocaine have. This amine group can become protonated, thus making these drugs more water-soluble and facilitating an injectible form. Benzocaine must be provided in a cream or oil-based preparation allowing just a topical form. Procaine and mepivacaine have poor topical anesthetic properties

The principal goals of bonding are _____. A. Sealing and thermal insulation B. Strengthening teeth and esthetics C. Esthetics and reduction of postoperative sensitivity D. Sealing and retention E. Retention and reduction of tooth flexure

D, Bonding is primarily for sealing the dentin and enhancing the retention of the restorative material in the preparation. Esthetic benefits are a welcome side benefit when using a composite restoration. Thermal insulation is provided by the use of composite as compared to amalgam but is not a benefit of the bonding. Bonding will not alter tooth flexure under normal load but may better help bond the unprepared tooth structure together

Which of the following statements best describes treatment options for a separated instrument at the initial stage of cleaning and shaping? A. Immediate attempt to remove the instrument. B. Stop canal instrumentation, do not attempt removal, and obturate. C. Attempt to bypass the obstructed instrument. D. Both A and C are options.

D, Both A and C are options. There are basically three approaches for the treatment of separated instruments: (1) attempt to remove the instrument, (2) attempt to bypass it, and (3) prepare and obturate to the segment. Using a small file and using the guidelines for negotiating a ledge, attempt to bypass the separated instrument. If this is successful, broaches or Hedstrom files are used to try to grasp and remove the segment. Then the canal is cleaned, shaped, and obturated to its new working length. If the instrument cannot be bypassed, preparation and obturation should be performed to the coronal level of the fragment.

_____ has a shorter half-life than other amides because a portion of its biotransformation occurs in the blood by the enzyme plasma cholinesterase. A. Lidocaine B. Bupivacaine C. Mepivacaine D. Articaine

D, Bupivacaine, mepivacaine, and lidocaine are all pure amides. Articaine has an ester bond and an amide bond. Since esters are biotransformed much more rapidly than amides, articaine has a much shorter half-life than the others

During the healing of a surgically treated intrabony (infrabony) pocket, regeneration of a new periodontal ligament, cementum, and alveolar bone will only occur when cells repopulate the wound from which of the following sources? A. Gingival epithelium B. Connective tissue C. Alveolar bone D. Periodontal ligament

D, Cells from the periodontal ligament are proposed to allow for regeneration of the periodontal tissues

The preferred surgical procedure to correct a Class II malocclusion due to a deficient mandible is which of the following? A. Maxillary impaction B. Maxillary setback C. Mandibular setback D. Mandibular advancement

D, Class II correction by surgery requires moving the mandible forward or the maxilla back. In a patient with a deficient mandible it is preferable to move the mandible forward. Moving the maxilla back significantly is difficult or impossible

Class II elastics are used by stretching an elastic between which of the two following points? A. From the posterior to the anterior within the maxillary arch B. From the posterior to the anterior within the mandibular arch C. From the posterior of the maxillary arch to the anterior of the mandibular arch D. From the posterior of the mandibular arch to the anterior of the maxillary arch

D, Class II elastics work in the direction that would be used to correct a Class II malocclusion, to pull the mandibular teeth forward and the maxillary teeth distally

Class II elastics are used by stretching an elastic between which of the two following points? A. From the posterior to the anterior within the maxillary arch B. From the posterior to the anterior within the mandibular arch C. From the posterior of the maxillary arch to the anterior of the mandibular arch D. From the posterior of the mandibular arch to the anterior of the maxillary arch

D, Class II elastics work in the direction that would be used to correct a Class II malocclusion, to pull the mandibular teeth forward and the maxillary teeth distally.

When Class III elastics are used, the maxillary first molars will _____. A. Move distally and intrude B. Move mesially and extrude C. Move mesially and intrude D. Move only mesially; there will be no movement in the vertical direction

D, Class III elastics are worn from the maxillary first molars to the mandibular canines. The force system created by Class III elastics will produce mesial movement and extrusion of the maxillary first molars

Which of the following scenarios is an example of classical conditioning? A. You teach a dentally-anxious patient diaphragmatic breathing unconditional stimulus (US), which naturally induces the physiological relaxation response unconditional response (UR). You seat that anxious patient in the dental chair for an examination conditional stimulus (CS) and ask them to use their breathing skills during the exam (US). While using the breathing skills, the patient will feel more relaxed conditional response (CR). B. You teach a dentally-anxious patient diaphragmatic breathing (US), which naturally induces the physiological relaxation response (UR). You ask the patient to practice that technique at home (CS) and also use it during procedures to reduce the subjective experience of anxiety (CR). C. You teach a dentally-anxious patient diaphragmatic breathing (US), which naturally induces the physiological relaxation response (UR). You seat that anxious patient in the dental chair for an examination (CS) and ask them to use their breathing skills during the exam (US). The focus on breathing serves as a distraction (US) from what the patient feels is threatening and fearful (CR), and, therefore reports less anxiety (CR). D. You teach a dentally-anxious patient diaphragmatic breathing (US), which naturally induces the physiological relaxation response (UR). You seat that anxious patient in the dental chair for an examination (CS) and ask them to use their breathing skills during the exam (US). After a number of these experiences, the patient will feel relaxed during the exam while using the breathing technique (UR) and without using it at all (CR). E. None of the above.

D, Classical conditioning (also known as respondent or Pavlovian conditioning) occurs when a neutral stimulus, one that is not associated with a particular response, is paired with an unconditioned stimulus ([US]; one that naturally elicits a particular response [UR]). After a number of pairings, the neutral stimulus (CS) elicits a conditioned response (CR), which is essentially a weaker form of the UR) without the presence of the US

Which of the following organisms is usually sensitive to clindamycin? A. Candida albicans B. Klebsiella pneumoniae C. Methicillin-resistant Staphylococcus aureus D. Streptococcus viridans E. Pseudomonas aeruginosa

D, Clindamycin is useful for some oral infections, including those involving Streptococcus viridans. Klebsiella pneumoniae and Pseudomonas aeruginosa are gram-negative rods and not subject to clinical inhibition by clindamycin. Methicillinresistant Staphylococci are insensitive to clindamycin and most traditional antistaphylococcal drugs. Candida albicans is a yeastlike fungus and is not inhibited by antibacterial drugs such as clindamycin.

Which of the following materials has the highest linear coefficient of expansion? A. Amalgam B. Direct gold C. Tooth structure D. Composite resin

D, Composite materials exhibit more dimensional change (2.5 times greater than tooth structure) when subjected to extreme changes in temperature than do the other choices. Direct gold is slightly higher than tooth structure, and amalgam is about twice as high as tooth structure

Papillomavirus has been found in all of the following lesions except _____. A. Oral papillomas B. Verruca vulgaris of the oral mucosa C. Condyloma acuminatum D. Condyloma latum E. Focal epithelial hyperplasia

D, Condyloma latum is one of the lesions that may be seen in secondary syphilis, which is caused by Treponema pallidum. All the other lesions listed may be associated with human papillomavirus

Cone-cutting results from _____. A. Too great a target-film distance B. Not selecting the proper kVp C. Not enough time exposure D. The x-ray machine being improperly aimed

D, Cone-cutting results from misalignment of the xray tube. Use a film-holding device with an external guide.

You are performing a 5-year follow-up on a 43-year-old implant patient. When comparing radiographs, you estimate that there has been almost 0.1 mm of lost bone height around the implant since it was placed. Which of the following is indicated? A. Removal of the implant and replacement with a larger-size implant. B. Removal of the implant to allow healing before another one can be placed 4 months later. C. Remaking the prosthetic crown because of tangential forces on the implant. D. The implant is doing well; this amount of bone loss is considered acceptable

D, Criteria for implant success include mean vertical bone loss of less than 0.02 mm annually after the first year of service. In this question, no further treatment is necessary at this time

You are performing a 5-year follow-up on a 43-year-old implant patient. When comparing radiographs you estimate that there has been almost 0.1 mm loss of bone height around the implant since it was placed. Which of the following is indicated? A. Removal of the implant and replacement with a larger size implant. B. Removal of the implant to allow healing before another one can be placed 4 months later. C. Remaking the prosthetic crown because of tangential forces on the implant. D. The implant is doing well; this amount of bone loss is considered acceptable.

D, Criteria for implant success include mean vertical bone loss of less than 0.02 mm annually after the first year of service. In this question, no further treatment is necessary at this time

A patient is unhappy with the esthetics of an anterior metal-ceramic crown, complaining that it looks too opaque in the incisal third. The reason for this is most likely _____. A. Using the incorrect opaque porcelain shade. B. Inadequate vacuum during porcelain firing. C. Not masking the metal well enough with the opaque. D. The tooth was prepared in a single facial plane.

D, D is the best answer because generally it is the dentist's fault and not the technician's. Incorrect opaque may influence the resultant shade. Inadequate vacuum will affect the esthetics. If the opaque does not mask well, the metal result is a grey appearance or lower value in the restoration

The idiopathic condition in which destructive inflammatory lesions featuring necrotizing vasculitis are seen in the lung, kidney, and upper respiratory tract is known as _____. A. Epidermolysis bullosa B. Stevens-Johnson syndrome C. Sturge-Weber syndrome D. Wegener's granulomatosis E. Secondary syphilis

D, Destructive inflammation in the three sites noted is characteristic of Wegener's granulomatosis

Which one of the following drugs enters the target cell and acts on a nuclear receptor? A. Diazepam B. Epinephrine C. Insulin D. Prednisone E. Heparin

D, Diazepam, epinephrine, and insulin act at ion channel receptors, G-protein-linked receptors, and tyrosine kinase-linked receptors, respectively. These three receptor types are cell surface receptors. Thyroid hormone and steroid hormones or drugs, such as prednisone, act on nuclear receptors, accounting for much of their action. Heparin's action is to stimulate antithrombin III in the plasma. Its action is extracellular

Which of the following materials has the highest linear coefficient of expansion? A. Amalgam B. Direct gold C. Tooth structure D. Composite resin

D, Direct gold and tooth structure have similar linear coefficients of expansion. Amalgram exhibits twice that expansion whereas composite expansion would be even greater (2.5 times greater than tooth structure)

The following definition refers specifically to the process in which an antimicrobial agent destroys (germicide) or inhibits the growth (microbiostatic) of pathogenic microorganisms on inanimate surfaces. A. Antisepsis B. Microbacterial control C. Sterilization D. Disinfection E. Asepsis

D, Disinfection refers only to the inhibition or destruction of pathogens. Spores are not killed during disinfection procedures. By custom, the term disinfection is reserved for chemicals applied to inanimate surfaces, and the term antiseptic is used for antimicrobial agents that are applied to living tissues.

Which of the following terms refers specifically to the process where an antimicrobial agent destroys (germicide) or avoids the growth (microbiostatic) of pathogenic microorganisms on inanimate surfaces? A. Antisepsis B. Microbacterial control C. Sterilization D. Disinfection E. Asepsis

D, Disinfection refers only to the inhibition or destruction of pathogens. Spores are not killed during disinfection procedures. By custom, the term disinfection is reserved for chemicals applied to inanimate surfaces, and the word antiseptic is used for antimicrobial agents that are applied to living tissues

Why are rounded internal line angles desirable in the preparation of amalgam restorations in primary teeth? A. They increase retention B. They conserve tooth structure C. They increase resistance D. They decreases internal stresses in the restorative material

D, Due to the small size of primary molars and, therefore, small restorations as well, it is helpful to reduce stresses within the restorative material. It has been demonstrated that rounded internal line angles aid in reducing stress when compared to sharp internal line angles. Many of the burs recommended for use in primary molars have a rounded end to help achieve softened internal line angles

In a patient with missing permanent maxillary lateral incisors, the decision of whether to substitute canines in the lateral spaces depends on all of the following except _____. A. The amount of crowding in the maxillary arch B. The interarch relationship between the maxillary and mandibular dentition C. The esthetic appearance of the permanent canines D. The type of orthodontic appliance used to align the teeth

D, Excessive crowding may influence the decision in favor of canine substitution. However, esthetic concerns may deter a decision to substitute canines for lateral incisors. Patients with a Class II interarch relationship requiring maxillary extractions anyway may be better served to substitute canines for laterals rather than extracting healthy first premolars

What type of epidemiology is primarily used in intervention studies? A. Descriptive B. Analytical C. Observational D. Experimental E. None of the above

D, Experimental epidemiology is used primarily in intervention studies. Once etiology for a particular disease has been determined, the researchers will try to establish the effectiveness of a particular program of prevention or therapy. Descriptive epidemiology is used to quantify disease status in a community. Analytical epidemiology, also called observational epidemiology, is used to determine the etiology of a disease

Which of the following factors affects long-term prognosis of teeth after perforation repair? A. Size of the defect. B. Location of the defect. C. Time elapsed between the perforation and its repair. D. All of the choices are true.

D, Factors affecting the long-term prognosis of teeth after perforation repair include the location of the defect in relation to the crestal bone; the length of the root trunk; the accessibility for repair; the size of the defect; the presence or absence of a periodontal communication to the defect; the time lapse between perforation and repair; the sealing ability of the restorative material; and technical skill. Early recognition and repair improve the prognosis. Smaller perforations (< 1 mm) cause less destruction. Subcrestal lesions, especially those closer to the apex, have better prognosis

It is generally desirable that x-ray films be all of the following except _____. A. High speed B. Fine grain size C. Coated with emulsion on both sides D. Sensitive to visible light

D, Film is sensitive to visible light but this is not a desired characteristic like the other choices

An 8-year-old boy received a traumatic injury to a maxillary central incisor. One day later, the tooth failed to respond to electric and thermal vitality tests. This finding dictates _____. A. Pulpectomy B. Apexification C. Calcium hydroxide pulpotomy D. Delay for the purpose of re-evaluation

D, For decades, controversy has surrounded the validity of thermal and electric tests on traumatized teeth. Only generalized impressions may be gained from these tests subsequent to a traumatic injury. They are, in reality, sensitivity tests for nerve function and do not indicate the presence or absence of blood circulation within the pulp. It is assumed that subsequent to traumatic injury, the conduction capability of the nerve endings or sensory receptors is sufficiently deranged to inhibit the nerve impulse from an electric or thermal stimulus. This makes the traumatized tooth vulnerable to false negative readings from these tests. Teeth that give a positive response at the initial examination cannot be assumed to be healthy or that they will continue to give a positive response over time. Teeth that yield a negative response or no response cannot be assumed to have necrotic pulps because they may give a positive response at later follow-up visits. It has been demonstrated that it may take as long as 9 months for normal blood flow to return to the coronal pulp of a traumatized, fully formed tooth. As circulation is restored, responsiveness to pulp tests returns

Although many plaque bacteria coaggregate, which of the following bacteria is believed to be an important bridge between "early colonizers" and "late colonizers" as plaque matures and becomes more microbiologically complex? A. Porphyromonas gingivalis B. Streptococcus gordonii C. Hemophilus parainfluenzae D. Fusobacterium nucleatum

D, Fusobacterium nucleatum can be found in health and disease. This bacterium is an important bridge between early and late colonizers of the dental plaque biofilm

Intranuclear viral inclusions are seen in tissue specimens of which of the following? A. Solar cheilitis B. Minor aphthous ulcers C. Geographic tongue D. Hairy leukoplakia E. White sponge nevus

D, Hairy leukoplakia is viral in origin and shows intranuclear inclusions in infected epithelial cells. Hairy leukoplakia is caused by Epstein-Barr virus, a herpes virus. Intranuclear epithelial inclusions are also seen other herpes virus infections (e.g., herpes simplex virus infections).

Your patient is 8 years old. Teeth #8 and #9 have approximately 50% of their crowns erupted. One month ago, the patient fell from a skateboard and hit teeth #8 and #9 on the sidewalk. The radiograph today shows open apices of these teeth, normal PDL, and no apparent periapical radiolucency. The patient has no reaction to electrical pulp tests. What is your treatment of choice? A. Calcium hydroxide pulpotomy B. Formocresol apexification technique C. Calcium hydroxide apexification technique D. Reappoint for exam and radiographs in 6 weeks

D, If a tooth is incompletely erupted or is being orthodontically treated, the tooth may be normal even if there is little sensitivity to electrical pulp tests. Certainly, in the absence of other symptoms, treatment is contraindicated.

A patient with the maxillary first permanent molar mesiobuccal cusp sitting distal to the buccal groove of the mandibular first molar has which type of malocclusion? A. Class I B. Class II, division 1 C. Class II, division 2 D. Class III

D, If the mandibular molar buccal groove is mesial to the mesiobuccal cusp of the maxillary molar, the relationship is described as Angle Class III

Why are implants not generally performed on a 12-year-old patient with congenitally missing lateral incisors? A. The patient would likely not be able to tolerate the surgical procedure. B. Waiting for the crowns is too much of an esthetic issue with most children that age. C. The gingival tissue will recede as the child gets older. D. The implants will appear to submerge as the child gets older.

D, Implants are osseointegrated and therefore behave as ankylosed teeth. As teeth erupt and alveolar bond formation occurs, an osseointegrated implant will appear to submerge

A 20-year-old woman has significant plaque build-up. Upon inquiry, she tells you that she brushes twice daily and flosses daily. You determine that the patient should be educated about optimal brushing procedures. Which strategy might you first use in a series of steps for improving her brushing skills? A. Demonstrate your recommended brushing practices. B. Explain to her good brushing technique. C. Tell her you don't believe she brushes twice daily. D. Ask her to demonstrate her teeth brushing. E. Clean her teeth.

D, In order to initiate behavior change, one must first fully assess the behavior. In this case, in order to address the patient's inadequate brushing technique one should observe the patient engaging in the behavior in order to identify the strengths and weaknesses of the process

The primary purpose of a maxillary denture occlusal index is to _____. A. Maintain the patient's vertical dimension B. Maintain both the correct centric and vertical relation records C. Maintain the patient's centric relation D. Preserve the facebow record

D, In order to preserve the mounting relationship in the articulator of the maxillary cast (facebow record) after processing a denture, an occlusal index of the maxillary denture is made after occlusal adjustments, and before decasting the denture. This procedure has nothing to do with the mandible's relationship to the maxilla.

A researcher follows a group of individuals in a population over 10 years to determine who develops cancer, and then evaluates the factors that affected the group. What type of study is this? A. Cross-sectional B. Case control C. Randomized D. Prospective cohort E. Retrospective cohort

D, In this case, the investigator chooses or defines a sample of subjects who do not yet have the outcome of interest: cancer. He or she measures risk factors in each subject (such as habits that may predict the subsequent outcome) and follows these subjects with periodic surveys or examinations to detect the outcome(s) of interest

Which of the following local anesthetic techniques is recommended for anesthetizing a primary mandibular second molar which will be extracted? A. Buccal and lingual infiltration adjacent to the second primary molar B. Inferior alveolar nerve block C. Inferior alveolar nerve block and lingual nerve block D. Inferior alveolar, lingual, and buccal nerve block

D, Inferior alveolar, lingual, and buccal nerve blocks are required to adequately anesthetize this area when performing deep restorations, pulp therapy, and extractions. Some studies have shown that local infiltration anesthesia for primary molars is effective, but this is primarily reserved for restorative procedures because there is an increased probability for anesthesia failure using local infiltration for pulp therapy and extraction procedures.

Of the following, which is considered to be the least stable orthognathic surgical movement? A. Advancement of the mandible B. Advancement of the maxilla C. Superior movement (impaction) of the maxilla D. Inferior movement of the maxilla

D, Inferior movement of the maxilla, especially without bone grafting and rigid fixation, has been shown to relapse over time because of vertical occlusal forces generated by the masticatory musculature

A diagnostic wax-up is indicated when _____. A. Re-establishing anterior guidance B. A provisional fixed prosthesis is to be fabricated C. Uncertainty exists regarding esthetics D. All of the above

D, It is recommended that any time there is a question regarding the treatment outcome involving a prosthetic device, or the need to produce templates for provisional restorations that reproduce a desired form of teeth, a diagnostic wax-up should be generated

When performing a laterally repositioned flap, which of the following must be considered relative to the donor site? A. Presence of bone on the facial B. Width of attached gingiva C. Thickness of attached gingiva D. All of the above

D, Laterally positioned flaps should only be performed when there is adequate bone and adequate width and thickness of attached gingiva on the facial of the donor site

Which of the following is not a classification of mandible fractures? A. Anatomic location B. Description of the condition of the bone fragments at the fracture site C. Angulation of the fracture and muscle pull D. LeFort level

D, LeFort level fractures are associated with maxillary injuries. Mandibular fractures are classified according to anatomic location, condition of the bone and soft tissue, and the muscle pull on the segments

If a canal is ledged during instrumentation, the best way to handle the problem is to _____. A. Continue instrumenting at the ledge. Although it may take some time, you will eventually bore your way to patency in the periodontal ligament space. B. Immediately stop and fill to where the ledge begins. C. Bind your irrigating needle in the canal and use short bursts of irrigant to loosen any debris blocking the canal. This will reopen the natural canal. D. Prebend the tip of a small file, lubricate, and try to negotiate around the ledge. E. Place citric acid or EDTA in the canal to soften the dentin. A small Gates Glidden or other rotary can be used to bypass the ledge

D, Ledges can sometimes be bypassed; the canal coronal to the ledge must be sufficiently straightened to allow a file to operate effectively. This may be achieved by anticurvature filing (file away from the curve). Precurve the file severely at the tip and use it to probe gently past the ledge. Otherwise, clean to the ledge and fill; warn the patient of poorer prognosis.

Fluorosis is the result of excessive systemic fluoride during which stage of tooth development? A. Initiation B. Morphodifferentiation C. Apposition D. Calcification

D, Localized infection, trauma, and excessive systemic fluoride ingestion may cause hypocalcification. Disturbances in apposition result in incomplete tissue formation. For example, an intrusive injury to a primary incisor may disrupt enamel apposition and result in an area of enamel hypoplasia

Irreversible pulpitis pain in which of the following sites is most likely to radiate to the ear? A. Maxillary premolar B. Maxillary molar C. Mandibular premolar D. Mandibular molar

D, Mandibular molar. The perception of pain in one part of the body that is distant from the actual source of the pain is known as referred pain. Teeth may refer pain to other areas of the head and neck. Referred pain is usually provoked by stimulation of pulpal C-fibers, the slowconducting nerves that, when stimulated, cause an intense, dull, slow pain. It always radiates to the ipsilateral side. Posterior teeth may refer pain to the opposite arch or periauricular area. Mandibular posterior teeth tend to transmit referred pain to the periauricular area more often than do the maxillary posterior teeth

How many hours after brushing does it usually take for a mature dental plaque to reform? A. 1-2 B. 5-10 C. 12-24 D. 24-48

D, Mature dental plaque usually reforms on the teeth within 24 to 48 hours after effective plaque removal

What tooth surfaces should be evaluated for furcation involvement on maxillary molars? A. Palatal, facial, and distal B. Mesial, distal, and palatal C. Facial, palatal, and mesial D. Facial, mesial, and distal

D, Maxillary molars usually have three roots (mesiobuccal, disto-buccal, and palatal). Furcation involvement can be assessed on these teeth from the facial (bifurcation between the mesio-buccal and disto-buccal roots), mesial (bifurcation between the mesio-buccal and palatal roots) and distal (bifurcation between the disto-buccal and palatal roots).

What is the minimum alveolar concentration of nitrous oxide (Vol %)? A. 50 B. 75 C. 95 D. 105

D, Minimum alveolar concentration is a measure of potency. It is the concentration required to produce immobility in 50 Vol % of patients responding to surgical incision. A minimum alveolar concentration of 105 Vol % indicates that nitrous oxide alone does not produce profound surgical anesthesia at a normal atmospheric pressure

The "willful failure of parent or guardian to seek and follow-through with treatment necessary to ensure a level of oral health essential for adequate function and freedom from pain and infection" is a definition of _____. A. Munchausen syndrome by proxy B. Emotional abuse C. Parental corruption D. Neglect

D, Munchausen syndrome by proxy is a condition in which a person, usually a parent, presents factitious symptoms and illnesses in a child, which may result in extensive testing and/or hospitalizations. Examples of emotional abuse include denial of affection, isolation, extreme threats, and corruption. A parent who knowingly and willingly does not seek care for a child who has pain, infection, or inadequate function is guilty of neglect

Naloxone antagonizes the therapeutic and toxic effects of which drug? A. Acetaminophen B. Aspirin C. Carbamazepine D. Fentanyl E. Ibuprofen

D, Naloxone is a competitive antagonist at opioid receptors

The _____ is recommended for palatal soft-tissue management from canine to canine bilaterally in the maxilla. A. Posterior superior alveolar B. Inferior alveolar C. Long buccal D. Nasopalatine

D, Nasopalatine (NP). The palatal tissue from canine to canine bilaterally is the premaxilla. The NP injection anesthetizes this area

Which of the following cells produce antibodies? A. Neutrophils B. T-lymphocytes C. Macrophages D. Plasma cells

D, Neutrophils are one of the primary defense cells of the innate immune system. T-lymphocytes are important activators of the adaptive immune system. Macrophages are antigen-presenting cells. Plasma cells produce antibodies

The long-term histopathologic consequences to an irradiated organ depend on _____. A. The presence of oxygen at the time of irradiation B. The sensitivity of the parenchymal component C. The damage to the stromal component D. All of the above E. None of the above

D, Numbers 1, 2, and 3 are correct

Which is the purpose of adjusting the occlusion in dentures? A. To obtain balanced occlusion. B. To stabilize dentures. C. To obtain even occlusal contacts. D. All of the above.

D, Occlusal adjustment of dentures should be done with the premise of obtaining even occlusal contacts with balanced occlusion in order to stabilize the dentures during function

A casting may fail to seat on the prepared tooth due to all of the following factors except _____. A. Temporary cement still on the prepared tooth after the temporary restoration has been removed. B. Proximal contact(s) of casting too heavy/tight. C. Undercuts present in prepared tooth. D. The occlusal of the prepared tooth was underreduced.

D, Occlusal reduction would not affect the ability to seat a casting. However, temporary cement, heavy proximal contacts, or tooth undercuts could keep the casting from seating completely

A casting may fail to seat on the prepared tooth due to all of the following factors except _____. A. Temporary cement still on the prepared tooth after the temporary restoration has been removed. B. Proximal contact(s) of casting are too heavy or too tight. C. Undercuts present in prepared tooth. D. The occlusal of the prepared tooth was underreduced

D, Occlusal reduction would not affect the ability to seat a casting. However, temporary cement, heavy proximal contacts, or tooth undercuts could keep the casting from seating completely

Methicillin-resistant Staphylococci are most likely to be inhibited by which drug? A. Amoxicillin B. Clarithromycin C. Clindamycin D. Vancomycin E. Penicillin V

D, Of the choices given, only vancomycin is effective against many methicillin-resistant Staphylococci. Various penicillins, macrolides, and clindamycin are ineffective

The radiosensitivity of cells depends upon _____. A. Mitotic future B. Mitotic activity C. Degree of differentiation D. All of the above E. None of the above

D, Options A, B, and C are correct.

Oral contraceptives can cause gingivitis. Oral contraceptives can accentuate the gingival response to bacterial plaque. A. Both statements are true. B. Both statements are false. C. The first statement is true, the second statement is false. D. The first statement is false, the second statement is true

D, Oral contraceptives can exacerbate the impact of bacterial plaque on the gingival tissues. However, they cannot cause gingivitis

What route of transmission is a needlestick injury of infectious disease? A. Direct contact B. Indirect contact C. Accidental contact D. Parenteral contact E. Droplets

D, Parenteral contact is defined as the transmission of pathogenic microorganisms by piercing the skin or mucous membrane (e.g., intravenous, subcutaneous, intramuscular) by an accidental or intentional stick with a needle or other sharp instrument that is contaminated with blood or other body fluid.

What is the active ingredient in PerioChip™? A. Doxycycline B. Tetracycline C. Metronidazole D. Chlorhexidine

D, PerioChip® is a biodegradable local delivery agent for chlorhexidine.

A noncarious tooth with deep periodontal pockets that do not involve the apical third of the root has developed an acute pulpitis. There is no history of trauma other than a mild prematurity in lateral excursion. What is the most likely explanation for the pulpitis? A. Normal mastication plus toothbrushing has driven microorganisms deep into tissues with subsequent pulp involvement at the apex. B. During a general bacteremia, bacteria settled in this aggravated pulp and produced an acute pulpitis. C. Repeated thermal shock from air and fluids getting into the deep pockets caused the pulpitis. D. An accessory pulp canal in the gingival or the middle third of the root was in contact with the pockets.

D, Periodontal disease can have an effect on the pulp through dentinal tubules, lateral canals, or both. Primary periodontal lesions with secondary endodontic involvement differ from primary endodontic-secondary periodontic lesions in their temporal sequence. Primary periodontal problems have a history of extensive periodontal disease

A 21-year-old man is referred to your oral and maxillofacial surgery practice for an orthognathic surgery consult. After your routine exam and review of radiographs, you note the following problem list: Class III skeletal facial deformity with a negative overjet of 6 mm and significant maxillary crowding; missing left mandibular first molar due to dental decay with multiple other early carious lesions; and calculus on the lingual surfaces of teeth #22 through #27 with gingival inflammation. Which of the following is the most appropriate order in which this patient's oral health needs should be sequenced? A. Definitive crown and bridge therapy, orthodontics to relieve crowding and to coordinate arches, caries management, surgery to correct the skeletal discrepancy, and periodontal therapy to control gingival inflammation. B. Caries management, orthodontics to relieve crowding and to coordinate arches, definitive crown and bridge therapy, periodontal therapy to control gingival inflammation, and surgery to correct the skeletal discrepancy. C. Periodontal therapy to control gingival inflammation, definitive crown and bridge therapy, orthodontics to relieve crowding and to coordinate arches, surgery to correct the skeletal discrepancy, and caries management. D. Periodontal therapy to control gingival inflammation, caries management, orthodontics to relieve crowding and to coordinate arches, surgery to correct the skeletal discrepancy, and definitive crown and bridge therapy

D, Periodontal management is the first step in the management of this patient. If the patient is unwilling to, or unable to maintain adequate hygiene prior to placement of orthodontic appliances, their subsequent placement will only make the periodontal situation more difficult. For the same reasons, dental decay should be treated prior to orthodontic treatment. The final prosthetic management should not be completed before the underlying skeletal anomaly is addressed because the occlusion will then be constructed to the best—and final—anatomical location

The supplemental fluoride daily dosage schedule for a 5-year-old child who lives in a community where the concentration of fluoride in the drinking water is less than 0.3 ppm is _____. A. 0 mg B. 0.10 mg C. 0.25 mg D. 0.50 mg E. 1 mg

D, Physicians and dentists can help prevent fluorosis by prescribing dietary fluoride supplements according to the Supplemental Fluoride Dosage Schedule recommended by the ADA Council on Scientific Affairs

Radiographs of the pregnant patient _____. A. Should never be made B. Cause fetal injury C. Should only be made with triple leaded aprons on the mother's lap D. Should be made when there is a specific need

D, Prudence suggests that radiographic examinations of a pregnant patient should be kept to a minimum consistent with the mother's dental needs

Your patient is 4 years old. The maxillary right primary central incisor was traumatically avulsed 60 minutes ago. What is the treatment of choice? A. Replant, splint, primary endo B. Replant, splint, formocresol pulpotomy C. Replant, no splint, primary endo D. None of the above

D, Replanting primary teeth has a poor prognosis, but could be considered if within 30 minutes. A primary tooth that is replanted will likely require splinting. The patient should be placed on antibiotics, restricted to a soft diet, and have a primary endodontic procedure accomplished

According to anxiety disorders research, it has been suggested that which of the following is the most important component of systematic desensitization? A. Cognitive restructuring B. Progressive muscle relaxation C. Diaphragmatic breathing D. Exposure E. Psychoeducation

D, Research suggests that the most integral component of the treatment of anxiety is exposure to the feared stimulus

Which are characteristics of a major connector that contribute to health and well-being? A. It is rigid and provides unification of the arch stability. B. It does not substantially alter the natural contour of the lingual surface of the mandibular alveolar ridge or the palatal vault. C. It contributes to the support of the prosthesis. D. All of the above. E. Only A and B.

D, Rigidity is provided by cross-arch stability through the principle of broad distribution of stress. The major connector should not alter dramatically the contours of the supporting structures, and it should contribute to the support of the prosthesis

The most common pin used in restorative procedures is a(an) _____. A. Friction-locked pin B. Cemented pin C. Amalgampin D. Self-threaded pin

D, Self-threaded pins are used by most operators, when pin use is indicated

Given the same amount of attachment loss and same pocket depth, a single-rooted tooth and a multirooted tooth have the same prognosis. The closer the base of the pocket is to the apex of the tooth, the worse the prognosis. A. Both statements are true. B. Both statements are false. C. First statement is true. Second statement is false. D. First statement is false. Second statement is true

D, Single-rooted teeth have a poorer prognosis than do multirooted teeth with comparable loss of attachment. Loss of attachment that extends to the apex of the root alters the crown-to-root ratio and makes the prognosis worse

Smooth surface caries refers to _____. A. Facial and lingual surfaces B. Occlusal pits and grooves C. Mesial and distal surfaces D. Both A and C.

D, Smooth surface caries occurs on any of the axial (facial, lingual, mesial, and distal) tooth surfaces but not the occlusal

Smooth surface caries refers to _____. A. Facial and lingual surfaces. B. Occlusal pits and grooves. C. Mesial and distal surfaces. D. A and C.

D, Smooth surface caries occurs on any of the axial (facial, lingual, mesial, and distal) tooth surfaces but not the occlusal.

Which of the following drugs blocks the aldosterone receptor? A. Amiloride B. Triamterene C. Losartan D. Spironolactone E. Furosemide

D, Spironolactone, a potassium-sparing diuretic useful in treating edema and heart failure, is a competitive antagonist at the aldosterone receptor.

The apical portion of maxillary lateral incisor usually curves to the _____. A. Facial B. Palatal C. Mesial D. Distal

D, Studies have shown that as many as 50% of the roots of maxillary lateral teeth were distally dilacerated. Oversight of the distal direction of root dilaceration of upper lateral incisors can be a contributing factor in the failure of endodontic treatment of these teeth.

In general, what species are predominant in supragingival tooth-associated attached plaque? A. Gram-negative rods and cocci B. Gram-negative filaments C. Gram-positive filaments D. Gram-positive rods and cocci

D, Supragingival plaque is either tooth-associated or outer layer. Tooth-associated is composed primarily of gram-positive cocci and short rods

The substitution of a relaxation response for an anxiety response (using a relaxation strategy such as diaphragmatic breathing) when one is exposed to a hierarchy of feared stimuli is called _____. A. Progressive muscle relaxation B. Habituation C. Flooding D. Systematic desensitization E. Biofeedback

D, Systematic desensitization is the process of systematically pairing a relaxation response with a hierarchy of feared stimuli

A 32-year-old male patient is fearful of receiving injections. You decide to use a cognitive behavioral strategy with him to help him through an injection. You have already instructed him in diaphragmatic breathing and ask him to practice this skill throughout the procedure. First, you show him the syringe. You talk about the characteristics of the needle. You then place the needle in his mouth with the cap on. Then, you simulate the procedure with the cap on. You then simulate the procedure with the cap off. Eventually, you proceed with the injection. What does this procedure exemplify? A. Habituation B. Cognitive control C. Flooding D. Systematic desensitization E. Behavior modification

D, Systematic desensitization is the systematic process of exposing the patient to a hierarchy of increasingly anxiety-provoking stimuli while the patient uses relaxation skills such as diaphragmatic breathing exercises.

Endodontically treated posterior teeth are more susceptible to fracture than untreated posterior teeth. The best explanation for this is _____. A. Moisture loss B. Loss of root vitality C. Plastic deformation of dentin D. Destruction of the coronal architecture

D, Teeth that have been endodontically treated have lost much of their coronal dentin in the access formation, irrespective of the pre-endodontic caries state. This loss of dentin compromises the internal architecture of the tooth. Less internal tooth structure, combined with the absorption of external forces (usually occlusal) may exceed the strength of dentin and result in fracture. Endodontic treatment and loss of pulp vitality are no longer thought to desiccate the tooth to the point of increasing risk of fracture

The purpose of the "penny test" is to check _____. A. Developer action B. Fixer action C. For proper development temperature D. For proper safelighting conditions

D, The "penny test" is a test of darkroom safelighting. A penny is placed on an exposed film (after removing the film from its cover) for 2 minutes and then the film is processed. If the processed film shows a lighter area on the film corresponding to the penny, then the safelighting is too bright and is fogging the film.

The following component of a scientific article provides the reader with detailed information regarding the study design. A. Introduction B. Background C. Literature review D. Methods E. Abstract

D, The Methods section organizes the research paper and allows the reader to assess the validity of the study and the reliability of the measures. This section should provide the reader with specific and detailed information regarding how the study was conducted. Based on this information, the reader should be able to replicate the study

The three dimensions of the Munsell Color Order System, the basis for shade guides such as Vita Lumin™, are _____. A. Absorption, scattering and translucency B. Color, translucency, and gloss C. Size, shape, and interactions with light D. Hue, value, chroma

D, The Munsell Color System, which is the basis of shade guides such as Vita Lumin®, is divided into three dimensions: hue is the shade or color of an object; chroma is the saturation or intensity of the color or shade; and value is the relative lightness or darkness of a color

All of the following are possible reasons why some local anesthetics have a longer duration of action than others, except _____. A. The addition of a vasoconstrictor B. Percent protein binding C. Degree of lipid solubility D. pKa of the drug

D, The addition of vasoconstrictors will prolong the duration of action of a local anesthetic. The percent protein binding also affects duration of action. Lipid solubility also affects the duration of action of injected local anesthetics. The pKa has an effect on onset of action but not on duration of action

The _____ is used as a guide to verify the occlusal plane. A. Ala-tragus line B. Interpupillary line C. Camper's line or plane D. All of the above

D, The ala-tragus line posteriorly and the interpupillary line anteriorly are used as a guide to align the occlusal plane for complete dentures. The Camper's line is also known as the ala-tragus line.

Which of the following statements about transmissible diseases is false? A. The risk of transmission after percutaneous injury is higher for HBV than for HIV. B. HCV and HIV are both caused by an RNA virus. C. A vaccine to immunize against HBV is available. D. The average risk of infection for HBV after a needlestick injury falls between HCV and HIV. E. All of the above

D, The average risk of infection for HBV after a needlestick injury does not fall between HCV and HIV. For HBV, the risk of transmission after percutaneous injury is 30%; this figure is 1.8% for HCV and 0.3% for HIV

On a radiograph, the facial root of a maxillary first premolar would appear distal to the lingual root if the _____. A. Vertical angle of the cone was increased B. Vertical angle of the cone was decreased C. X-ray head was angled from a distal position relative to the premolar D. X-ray head was angled from a mesial position relative to the premolar

D, The buccal object rule [Clark's rule or "SLOB" rule (Same Lingual, Opposite Buccal)] is used to identify the buccal or lingual location of objects in relation to a reference object. If the image of the object moves mesially when the x-ray tube is moved mesially, the object is located on the lingual. If the image of the object moves distally when the x-ray tube moves mesially, the object is located on the buccal (facial).

Which of the following is the longest-acting local anesthetic? A. Mepivacaine B. Lidocaine C. Prilocaine D. Bupivacaine

D, The degree of hydrophobicity and protein binding are the most important factors in determining duration of action of a local anesthetic. Bupivacaine is highly hydrophobic (therefore lipophilic) and is 95% bound to protein. The other listed agents are less hydrophobic and are between 55% and 75% bound to protein.

Oropharyngeal candidiasis is an adverse effect most likely with which drug? A. Inhaled salmeterol B. Inhaled ipratropium C. Inhaled nedocromil D. Inhaled beclomethasone E. Inhaled methacholine

D, The effect of glucocorticosteroids remaining in the mouth after inhalation is to make the oral cavity more susceptible to fungal infection. The mouth should be rinsed with water after inhalation use. Inhaled methacholine, unlike the other drugs listed, is not used therapeutically but, rather, is used to diagnose hyperactive airway

Well-controlled diabetics have more periodontal disease than nondiabetics. Well-controlled diabetics can generally be treated successfully with conventional periodontal therapy. A. Both statements are true. B. Both statements are false. C. The first statement is true, the second statement is false. D. The first statement is false, the second statement is true

D, The extent and severity of periodontal disease in a patient with well-controlled diabetes is usually no more than the extent and severity of disease in patients without diabetes. Patients with well-controlled diabetes can usually be treated with conventional periodontal therapy.

The purpose of a high-voltage transformer in an x-ray machine is to _____. A. Decrease the tube current B. Increase the wavelength of the x-rays C. Improve timer accuracy D. Increase the potential between the filament and the cathode E. Regulate the rate of release of photons from the anode F. Increase resistance in the filament

D, The high-voltage transformer increases the voltage from the line voltage to the high voltage between the anode and cathode necessary to impart sufficient energy to the electrons to convert some of their energy into photons at the target

Which of the following injections has the highest degree of failure? A. Posterior superior alveolar B. Lingual C. Nasopalatine D. Inferior alveolar

D, The inferior alveolar nerve block has a stated success rate of 85%, the lowest of any intraoral injection. Lingual and nasopalatine injections are close to 100% successful, and the PSA nerve block is also much more than 85% effective

What is the major clinical difference between the established lesion of gingivitis and the advanced lesion of periodontitis? A. Gingival color, contour, and consistency B. Bleeding on probing C. Loss of crestal lamina dura D. Attachment and bone loss E. Suppuration

D, The initial, early, and established lesions of gingivitis do not have attachment loss associated with them

Torsades de pointes, or polymorphic ventricular tachycardia, is linked most closely to what characteristic of the electrocardiogram? A. Inverted T wave B. Shorter P-R interval C. Shorter P-P interval D. Longer Q-T interval E. Normal electrocardiogram

D, The long Q-T interval observed as a result of certain drugs or as a hereditary condition makes the patient more susceptible to this condition

Motor adverse effects from phenothiazine antipsychotic drugs are due to drug effects in what region of the brain? A. Chemoreceptor trigger zone B. Cerebrum C. Cerebellum D. Nigro-striatal pathway E. Mesolimbic pathway

D, The nigro-striatal pathway contains dopaminergic neurons—important in muscle control. Many antipsychotic drugs block these, leading to the motor adverse effects

The following teeth are erupted in a 4-year-old patient. What is the space maintenance of choice? A. Band-loop space maintainer. B. Lower lingual holding arch. C. Nance holding arch. D. Distal shoe space maintainer

D, The only possibility within these choices is the distal shoe space maintainer. Some clinicians find that a removable "kiddie" acrylic partial can also be successful. These kiddie partials extend distally to the point where the mesial of the first permanent molar would be. Some advocate placing a 1-mm-deep labial-lingual groove in the cast on the alveolar ridge on the mesial of the first permanent molar. This results in extra acrylic at the tissue-acrylic interface that causes pressure. This may aid in keeping the unerupted first permanent molar in position.

The purpose of applying a layer of opaque porcelain in a metal-ceramic restoration is to _____. A. Create a bond between the metal and porcelain B. Mask the metal oxide layer as well as provide a porcelain-metal bond C. Create the main color for the restoration D. A and B are correct E. All of the above

D, The opaque porcelain is used for masking the oxide layer of the metal and provides the porcelain- metal bond. The minimum thickness of the opaque is about 0.1 mm.

The mother of a 5-year-old patient is concerned about the child's thumb-sucking habit. Six months ago, the patient had 5-mm overjet and a 3-mm anterior open bite. Today, the patient has 10% overbite and 3.5-mm overjet. The mother says that the child only sucks his thumb every night when falling to sleep. Of the following, which is the best advice? A. Refer to a speech pathologist. B. Recommend tongue thrust therapy. C. Recommend a thumb-sucking appliance. D. Counsel the parent regarding thumb-sucking, and recall the patient in 3 months

D, The patient's overbite/overjet improved from the previous examination and therefore it is likely that the patient's digit-sucking habit had decreased significantly. The mother did state that the patient only sucks his thumb while falling asleep. When digit-sucking occurs for a limited time per day, not only is tooth movement normally associated with digit-sucking unlikely, it is possible for teeth to return to a more normalized position. Remember that the risk of malocclusion as related to habitual activity is a function of amount of time per day the habit is practiced, the duration of the habit in terms of weeks and months, and the intensity of the habit. Because the occlusion seems to be improving and because the habit has significantly decreased, the best treatment is to counsel the parent regarding thumb-sucking, and recall the patient in 3 months.

Which of the following is the most common method of sterilization? A. Dry heat B. Ethylene oxide C. Glutaraldehyde at 2% D. Autoclave E. Chemi-clave

D, The proper time and temperature for autoclaving is 250˚ F (121˚ C) for 15 to 20 minutes, which yields 15 pounds pressure of steam, or 270˚ F (134˚ C) for a minimum of 3 minutes, which yields 30 pounds pressure of steam. Moist heat destroys bacteria—denaturation of the highprotein- containing bacteria

Which of the following does not represent a fascial space for the spread of infection? A. Superficial temporal space B. Pterygomandibular space C. Masseteric space D. Rhinosoteric space E. Submental space

D, The superficial temporal, pterygomandibular, masseteric, and submental spaces are potentially involved in the of odontogenic infection. There is no rhinosoteric space.

Which of the following is(are) uses for the surveyor? A. To aid in the placement of an intracoronal retainer. B. To block out a master cast. C. To measure a specific depth of an undercut. D. All of the above. E. Only A and B are correct

D, The surveyor is used for surveying a diagnostic cast and to measure a specific depth of undercut. It also helps to determine the most desirable path of placement for a removable partial denture. It identifies bony areas that may need to be surgically removed because they interfere during insertion of the RDP. It is also used to survey crowns, place intracoronal retainers, machine or mill cast restorations, and survey and block out a master cast before constructing an RDP

The most common site for oral cancers in the oral cavity is _____. A. Lip B. Soft palate C. Hard palate D. Tongue E. Tonsils

D, The tongue is the most common place for incident cancers in the oral cavity.

A radiolucency near the apex of tooth #28 is seen radiographically. The tooth is asymptomatic and does not have caries or periodontal problems. Which is most likely the cause of the radiolucency? A. Submandibular fossa. B. Periapical granuloma. C. Complex compound odontoma. D. Mental foramen

D, The tooth does not exhibit any pathology to indicate that the radiolucency is derived from the tooth. The mental foramen can appear on the apex, depending on the direction of the x-ray beam

The setting reaction of dental amalgam proceeds primarily by _____. A. Dissolution of the entire alloy particle into mercury. B. Dissolution of the Cu from the particles into mercury. C. Precipitation of Sn-Hg crystals. D. Mercury reaction with Ag on or in the alloy particle.

D, The trituration process mixes the amalgam components and the reaction results in the alloy particle being coated by mercury and a product being formed.

The setting reaction of dental amalgam proceeds primarily by _____. A. Dissolution of the entire alloy particle into mercury B. Dissolution of the Cu from the particles into mercury C. Precipitation of Sn-Hg crystals D. Mercury reaction with Ag on or in the alloy particle

D, The trituration process mixes the amalgam components and the reaction results in the alloy particle being coated by mercury and a product formed.

Two equal and opposite forces that are not collinear applied to a tooth are called which of the following? A. The center of resistance B. The center of rotation C. Root movement D. A couple

D, This is the definition of a couple. A couple results in a rotational tendency or pure moment.

Centric relation is the maxillomandibular relationship in which the condyles are in their most _____. A. Posterior position with the disc interposed at its thickest avascular location B. Posterior position with the disc interposed at its thinnest locale C. Superior position with the disc in its most anterior position D. Superior-anterior position with the disc interposed at its thinnest location

D, This meets the definition of centric relation and the normal anatomic relationships of the temporomandibular discs to the condyles. Centric relation is a clinically repeatable mandibular position primarily defined by the temporomandibular joints, not the teeth

An adolescent patient presents to your office with a skeletal and dental Class II malocclusion and a deep bite. Which of the following would be a proper treatment plan for this patient? A. Reverse-pull headgear, extrusion arch, and full fixed appliances B. Reverse-pull headgear, intrusion arch, and full fixed appliances C. Extraction of maxillary first premolars, extrusion arch, and full fixed appliances D. Extraction of maxillary first premolars, intrusion arch, and full fixed appliances

D, This patient, if still growing, may be treated with a growth modification approach using headgear (either cervical or high-pull, not reverse-pull) to correct the Class II malocclusion. Since deep overbite is present, a cervical headgear should be used because this type of headgear will extrude the molars which, in turn, will aid in reducing overbite; however, this was not one of the choices. If the patient is a nongrowing patient, the second approach to treat Class II malocclusion is Class II camouflage, which includes extraction of maxillary first premolars to correct the malocclusion. An intrusion arch along with full fixed appliances should be used to correct the deep bite

In what situation is the postganglionic nerve of the sympathetic system a cholinergic nerve? A. The nerves to the eye B. The nerves to the heart C. Most nerves to blood vessels D. Most nerves to sweat glands E. Most nerves to salivary glands

D, This situation for sweat glands is atypical for the sympathetic nervous system

A biopsy of the lower lip salivary glands showed replacement of parenchymal tissue by lymphocytes. The patient also had xerostomia and keratoconjunctivitis sicca. These findings are indicative of which of the following? A. Lymphoma B. Crohn's disease C. Mumps D. Sjögren's syndrome E. Mucous extravasation phenomenon

D, This triad of signs defines primary Sjögren's syndrome. The patient has secondary Sjögren's syndrome if rheumatoid arthritis or other autoimmune disease is present.

Which of the following is least likely to be successfully treated with a bone graft procedure? A. One-walled defect B. Two-walled defect C. Three-walled defect D. Class III furcation defect

D, Through-and-through (Class III) furcation defects are least likely to be treated with bone graft procedures

Lack of reciprocation of a removable partial denture (RPD) clasp is likely to cause _____. A. Tissue recession due to displacement of the RPD B. Insufficient resistance to displacement C. Fracture of the retentive clasp D. Abutment tooth displacement during removal and insertion

D, Tooth mobility is prevented or diminished during function by the reciprocating clasp. The reciprocating clasp should contact the tooth on or above the height of contour of the tooth, allowing for insertion and removal with passive force. Displacement of the RPD toward the tissue, causing tissue recession, is a function of the lack of occlusal rests.

After completing your postoperative instructions for dental implant placement for replacement of tooth #14, your patient asks you how long it will be before she can get her new tooth. Which of the following is most correct to allow complete osseointegration? A. 3 weeks B. 6 weeks C. 3 months D. 6 months

D, Traditionally 6 months has been the recommended period for integration and subsequent loading of posterior maxillary implants. Today, because of technological advancements in specified cases, earlier loading may be possible

The indications for periradicular surgery include all of the following except which one? A. Procedural accidents during previous nonsurgical endodontic treatment. B. Irretrievable separated files in the canals. C. Failed nonsurgical endodontic treatment and persisting radiolucency. D. Treatment for a nonrestorable tooth.

D, Treatment for a nonrestorable tooth. Perpetuation of apical inflammation or infection after nonsurgical root canal therapy is often due to poorly obturated canals, tissues left in the canal, broken instruments, procedural accidents during treatment, or remnants of necrotic tissue in accessory canals. The removal of the apical segment of the tooth via root-end surgery usually removes the nidus of infection. Nonrestorable teeth should be extracted.

Your patient is 4 years old. Tooth E was traumatically intruded and approximately 50% of the crown is visible clinically. What is your treatment of choice? A. Reposition and splint B. Reposition, splint, and primary endodontics C. Reposition, splint, and formocresol pulpotomy D. None of the above

D, Unless it can be determined that the primary tooth is impinging on the permanent successor intruded primary teeth are left alone in the hopes that they will spontaneously re-erupt. On the other hand, intruded permanent teeth have a poorer prognosis. If there is an open apex, an intruded permanent tooth should be closely monitored for spontaneous eruption. An intruded permanent tooth with a closed apex should be repositioned orthodontically, and a calcium hydroxide pulpectomy should be performed 2 weeks following the injury

If an unwrapped, nonprocessed x-ray film is exposed to normal light for just a second and then processed, it _____. A. May still be used but will be a little dark B. May still be used but will be a little light C. May still be used but will be brown D. Will be completely black E. Will be completely clear

D, Visible light will expose all the silver bromide crystals and the film will be black after processing.

Occlusal loading resulting in tooth flexure, mechanical microfractures, and loss of tooth substance in the cervical area is _____. A. Abrasion B. Attrition C. Erosion D. Abfraction

D, Wasting diseases of the teeth include erosion (corrosion; may be caused by acidic beverages), abrasion (caused by mechanical wear as with toothbrushing with abrasive dentifrice), attrition (due to functional contact with opposing teeth), and abfraction (flexure due to occlusal loading

Which of the following statements is(are) true concerning the evaluation of the occlusion on a cast restoration? A. The restoration is in proper occlusion if it holds shim stock. B. The restoration is in proper occlusion if the adjacent teeth hold shim stock. C. The restoration is in proper occlusion when articulating paper marks multiple points of contact on the restoration. D. A, B, and C. E. None of the above.

D, When checking the occlusion of a cast restoration, mylar paper or shim stock is a very accurate method for testing occlusal contacts. The procedure is to check with the mylar paper before placing the restoration in the teeth adjacent to the tooth to be restored and the opposing side. Place the restoration and check whether the same occlusal contacts are maintained on the tested teeth. When all teeth, including the one being restored, hold the mylar paper upon occluding and even, articulating markings are present, then occlusion contacts are correct.

Which of the following statements about indirect pulp caps is false? A. Some leathery caries may be left in the preparation. B. A liner is generally recommended in the excavation. C. The operator should wait at least 6 to 8 weeks before re-entry (if then). D. The prognosis of indirect pulp cap treatment is poorer than that of direct pulp caps.

D, When doing an indirect pulp cap some caries may be left, a liner [probably Ca(OH)2] is usually placed over the excavated area, and the area may be assessed 6 to 8 weeks later. Regardless, the indirect pulp cap prognosis is better than the prognosis for direct pulp caps.

Which of the following statements about indirect pulp caps is false? A. Some leathery caries may be left in the preparation. B. A liner is generally recommended in the excavation. C. The operator should wait at least 6 to 8 weeks before re-entry (if then). D. The prognosis of indirect pulp cap treatment is poorer than that of direct pulp caps

D, When doing an indirect pulp cap, some caries may be left, a liner (probably Ca[OH]2) is usually placed over the excavated area, and the area may be assessed 6 to 8 weeks later. Regardless, the indirect pulp cap prognosis is better than the prognosis for direct pulp caps

The function of the filament is to _____. A. Convert electrons into photons B. Convert photons into electrons C. Release photons D. Release electrons E. None of the above

D, When heated, the filament releases electrons (thermionic emission).

For a gold casting alloy, which of the following is added primarily to act as a scavenger for oxygen during the casting process? A. Copper B. Palladium C. Silver D. Zinc

D, Zinc is added to act as a scavenger for oxygen during the casting process. Copper and palladium increase the hardness and affect the color. Silver has an affect on the color as well.

The administration of which compound will give "epinephrine reversal" (drop in blood pressure from epinephrine) if given prior to administration of epinephrine? A. Atropine B. Guanethidine C. Propranolol D. Phenoxybenzamine E. Tyramine

D, α-Adrenoceptor blockers such as phenoxybenzamine will inhibit the vasoconstrictor effect of epinephrine but not the vasodilator effect of epinephrine. Therefore, the administration of α-blockers will result in epinephrine reversal. Atropine would have little effect since it does not act at adrenergic receptors. Propranolol would only block the vasodilator effect of epinephrine and the effect of epinephrine on the heart. Guanethidine and tyramine act largely at prejunctional sites and don't block adrenergic receptors

Which of the following local anesthetics has the highest pKa? A. Lidocaine B. Prilocaine C. Mepivacaine D. Bupivacaine

D,The pKa for lidocaine or prilocaine is 7.8, mepivacaine is 7.7, and bupivacaine is 8.1.

In the radiolysis of water, _____. A. Free radicals are formed which are nonreactive B. The presence of dissolved O2 reduces the number of free radicals C. The formation of free radicals is the "direct effect" D. The resultant free radicals may alter biological molecules E. Two of the above F. None of the above

E, "Direct effect" refers to production of free radicals from the ionization of water (C). These free radicals formed in the radiolysis of water are highly reactive and may alter biological molecules (D). The presence of oxygen increases the number of free radicals

Which of the following are guidelines for disinfectants used in dental practice? A. Have an EPA registration number. B. Kill the Mycobacterium tuberculosis. C. Have an ADA seal of approval. D. Must be used according to guidelines. E. All of the above

E, A disinfectant should be able to kill the Mycobacterium tuberculosis. This is the benchmark organism for disinfectants. It is much harder to kill than most bacteria, viruses, fungi, and protozoa. This resistance is partially due to the waxy cell wall of Mycobacterium

In a Kennedy Class I arch in which all molars and the first premolar are missing and the rest of the teeth have good periodontal support, the preferred choice of treatment is _____. A. A removable partial denture replacing all missing teeth B. A fixed dental prosthesis replacing the missing premolar and a removable partial denture replacing the molars C. Implant supported crowns replacing the first premolars and a removable partial denture replacing the molars D. A and B are preferred choice of treatment over C. E. B and C are preferred choice of treatment over A.

E, A fixed dental prosthesis replacing the first bicuspids improves the prognosis of the second bicuspids when placing a removable dental prosthesis. Implants would also improve the prognosis by not leaving the second bicuspid standing alone and acting as a cantilever when in function with the removable prosthesis

Which of the following does not represent a possible finding of severe infection? A. Trismus B. Drooling C. Difficult or painful swallowing D. Swelling and induration with elevation of the tongue E. A temperature of 99˚ F

E, A patient with severe infection and systemic involvement unless immunocompromised are expected to present in a febrile state, or a temperature of greater than 100˚F. All the other items refer to symptoms that indicate potential airway emergency

Severity of the course of a periradicular infection depends upon the _____. A. Resistance of the host B. Virulence of the organisms C. Number of organisms present D. Both A and B only E. All of the choices are true

E, A patient's immune response to a periradicular infection varies according to the person. The size and volume of the pulp, the number and quality of the nerves, and the pulpal vascularity and cellularity are all unique to the person. The different virulence of organisms causing the infection may cause differences in pain experienced, differences in the amount of orthoclastic activity, etc Sheer numbers of organisms can influence their virulence.

A set of precautions designed to prevent transmission of HIV, HBV, and other bloodborne pathogens when providing first aid or health care is known as _____. A. Asepsis B. Infection control C. Sterilization D. Disinfection E. Standard infection control procedures

E, A thorough medical history, physical examination, and laboratory tests will not always detect patients who are carriers of infectious diseases. Therefore, you must assume that all patients are infected with HIV, HBV, or other bloodborne pathogens. Similar infection control procedures must be used for all patients, regardless of their medical history or the type of treatment to be performed

Potential problems in connecting implants to natural teeth include all of the following except _____. A. Stress is concentrated at the superior portion of the implant B. Breakdown of osseointegration C. Cement failure on the natural abutment D. Screw or abutment loosening E. Fracture in the connector area of the prosthesis

E, A tooth moves within the limits of its periodontal ligament during function. The relative immobility of the osseointegrated implant compared to the functional mobility of a natural tooth can create stresses at the neck of the implant up to two times the implied load on the prosthesis. Potential problems when connecting an implant with a tooth include (1) breakdown of the osseointegration; (2) cement failure on the natural abutment; (3) screw or abutment loosening; and (4) failure of the implant prosthetic component. Fracture in the connector area is rarely seen in this situation.

Which of the following statements is true concerning vertical dimension of rest (VDR)? A. VDR = physiologic rest position. B. VDR = position of the mandible when opening and closing muscles are at rest. C. VDR is a postural relationship of the mandible to maxilla. D. VDR = the amount of jaw separation controlled by jaw muscles when they are in a relaxed state. E. All of the above

E, All of the above statements are correct. Vertical dimension of rest (VDR) is a physiologic rest position; it is the position of the mandible when the muscles are in their minimum state of tonicity, which occurs when a patient is relaxed with the trunk upright and the head unsupported. In this position, the interocclusal distance is usually 2 to 4 mm when observed at the first premolar area

Which of the following statement(s) is(are) true regarding treatment of a tooth presenting with a sinus tract? A. Treat with conventional root canal therapy. B. Antibiotics are not needed. C. The sinus tract should heal in 2 to 4 weeks after conventional root canal therapy. D. If the tract persists post-root canal therapy, do root-end surgery with root-end filling. E. All of the above choices are true.

E, All of the choices are true. Suppurative apical periodontitis: continuously or intermittently draining sinus tract, usually drains into the oral mucosa. The exudate can also drain through the gingival sulcus of the involved tooth, mimicking a periodontal lesion with a "pocket." However, this is not a true periodontal pocket because there is not a complete detachment of connective tissue from the root surface. It should be treated with conventional root canal therapy. Antibiotics are not needed, since the infection is localized and draining. If the tract does not heal within a few weeks, root-end surgery may be required. If left untreated, however, it may become covered with an epithelial lining and become a true periodontal pocket.

Which of the following is an example of a cognitive strategy that may be useful in pain management? A. Address expectations by providing information and addressing any questions and/or concerns. B. Suggest to patients that they learn to identify, evaluate, and eliminate maladaptive thinking. C. Encourage patient efforts to address their anxiety and pain management. D. Suggest to patients that they learn to generate, evaluate, and apply more realistic thinking. E. All of the above

E, All of the strategies listed may be considered appropriate cognitive interventions in pain management.

Which of the following statement(s) about the hepatitis B vaccination is(are) true? A. HBV vaccine must be offered to all potentially exposed dental workers. B. The HBV vaccine must be free to all potentially exposed dental workers. C. At the time of employment, each person should be asked to provide documentation of previous immunizations. D. Three doses are given to confer immunity. E. All of the above.

E, All of these measures help ensure the safety of dental personnel

Which of the following effects is a typical effect of an antimuscarinic drug? A. Bronchoconstriction B. Lacrimation C. Miosis D. Sweating E. Urinary retention

E, All other choices are typical of muscarinic cholinergic receptor agonists

Which drug has an antibacterial spectrum that is limited to anaerobes? A. Amoxicillin B. Clarithromycin C. Clindamycin D. Gentamicin E. Metronidazole

E, Amoxicillin, clarithromycin, and clindamycin are effective against some anaerobes but their spectrum is not limited to anaerobic bacteria. Aminoglycosides are effective only against aerobes. Metronidazole's action requires a reduced environment. Its antibacterial spectrum is limited to anaerobes. Metronidazole is also effective against many parasites.

The use of H2 histamine receptor blockers is most clinically useful at what cell type? A. Beta cells of the pancreas B. Basophils C. Mast cells D. Juxtaglomerular cells E. Parietal cells

E, Basophils and mast cells release histamine. However, the cell that responds to histamine stimulation at the H2 receptor is the parietal cell of the stomach. Stimulation of this receptor leads to proton release and a decrease in the pH of the stomach lumen. H2 histamine receptor blockers are used to reduce stomach acid.

Which of the following are characteristics of proper documentation in a dental record? A. Specific B. Objective C. Complete D. Timely E. All of the above

E, Being specific helps to avoid misinterpretation of reports. Being objective provides the basis for accuracy in describing events. Being complete provides the basis for a thorough review of the facts when reviewing the report. Being timely ensures the best opportunity to recall all relevant events

Which of the following are factors associated with bone loss? A. Initial implant instability. B. Excessive occlusal force. C. Inadequate hygiene. D. Inadequate prosthesis fit. E. All of the above

E, Bone resorption around dental implants can be caused by inadequate oral hygiene, premature loading, and repeated overloading. If an implantsupported framework does not fit passively, the implant is placed under constant force. If significant compressive forces are placed on the interfacial bone, these can lead to implant failure

Which of the following is not a factor in the appraisal of stress? A. Familiarity—how familiar the situation is; the less familiar, the more stressful it may seem. B. Predictability—how predictable the situation is; the less predictable, the more stressful it may seem. C. Controllability—how controllable the situation seems to be; the less controllable, the more stressful it may seem. D. Imminence—the more imminent the situation is, the more stressful it may seem. E. Positive or negative valence—whether the situation is positive or negative; positive situations (e.g., a wedding) are typically experienced as less stressful than are negative situations (e.g., a divorce).

E, Both positive and negative events or situations are experienced as stress.

What behavior can you typically expect from an anxious patient in the dental chair? A. He or she is more likely to sit still, hands clasped together. B. He or she is more likely to sit casually, legs crossed, reading a magazine. C. He or she is more likely to keep to himself or herself and not speak unless spoken to. D. He or she is more likely to fidget in the chair, moving his or her hands and feet. E. Both A and C.

E, Contrary to their behavior in the waiting room, anxious patients are typically more likely to sit very still, often holding onto the arms of the dental chair, and engage in minimal verbal communication unless encouraged by the clinician.

In clinical practice, you frequently see young patients who are nervous about seeing the dentist. Knowing which factors are important influences on young patients' comfort, you consider which of the following to help your patients to feel more comfortable? A. Inviting a parent into the operatory for support. B. Placing toys and children's books in the waiting room. C. Hanging child-friendly décor in the operatory. D. Talking to the child about his or her interests before beginning your work. E. All of the above.

E, Creating a child-oriented environment (e.g., having toys and books in the waiting room, hanging pictures on the wall and/or ceiling that a child would find interesting), conveying interest in the child by asking about their interests, and having the parent present are all variables that may put child patients more at ease.

Oral antacids are most likely to reduce the absorption of which drug when it is given orally? A. Clarithromycin B. Clindamycin C. Metronidazole D. Penicillin V E. Tetracycline

E, Di- and trivalent cations, such as those found in oral antacids, chelate tetracyclines and prevent their absorption.

With no other intervention or instruction, which is most likely to trigger a physiological relaxation response? A. Observing one's own physiological responses (e.g., heart rate, blood pressure) B. Muscle tensing C. Reassurance D. Thought stopping E. Diaphragmatic breathing

E, Diaphragmatic breathing naturally activates the parasympathetic nervous system, producing a relaxation response

Proper radiographic infection control includes all of the following except _____. A. Wearing gloves while making radiographs B. Disinfecting x-ray machine surface C. Covering working surfaces with barriers D. Sterilizing nondisposable instruments E. Sterilizing film packets

E, Film packets need not be sterilized because the goal is to prevent crosscontamination, not ensure that everything that goes into a patient's mouth is sterile

Which of the following is the treatment of choice for a 7-year-old child with a nonvital tooth 30 with buccal sinus tract? A. Gutta-percha filling B. Gutta-percha filling followed by root-end surgery C. Extraction D. Apexogenesis E. Apexification

E, If an immature tooth is nonvital, the diseased tissue must be removed via pulpectomy. Apexification is the treatment of choice.

It is acceptable for the operator to hold the film in a patient's mouth _____. A. If the patient is a child B. If the patient or parent grants permission C. If the patient has a handicap D. If no film holder is available E. Never

E, If someone must hold a film and the patient cannot, then it should be a family member or friend of the patient, not an x-ray operator in the dental office.

How do people typically respond to stress? A. Physiologically (fight-or-flight response; i.e., autonomic arousal) B. Cognitively (beliefs of self-efficacy, stress appraisal) C. Behaviorally (e.g., disturbed sleep/appetite, impaired attention, acting out) D. Emotionally (e.g., anxiety, anger, fear) E. All of the above

E, Individuals respond to stress physiologically, behaviorally, cognitively, and emotionally

Which of the following is/are recommendations for the use of masks? A. Use whenever aerosols or spatter may be generated. B. A new mask should be worn for each patient. C. Masks should be changed at least once every hour. D. Masks should be changed more frequently in the presence of heavy aerosol contamination. E. All of the above

E, Masks that cover the mouth and nose reduce inhalation of potentially infectious aerosol particles. They also protect the mucous membranes of the mouth and nose from direct contamination. Masks should be worn whenever aerosols or spatter may be generated. If a mask is worn longer than 20 minutes in an aerosol environment, the outside surface of the mask becomes a nidus of pathogenic bacteria rather than a barrier. It is recommended that a new mask be worn for each patient and that masks be changed routinely at least once every hour and more often in the presence of heavy aerosol contamination.

Which of the following recommendations must be followed when handling mercury? A. Train personnel involved in the handling of mercury B. Work in properly ventilated areas C. Use high-volume evacuation systems when finishing or removing amalgams D. Avoid direct skin contact with the metal E. All of the above

E, Mercury can be absorbed through the skin as well as absorbed by inhalation. Safe handling, resulting in part from proper training, helps reduce the risk of exposure

A 15-year-old patient has a numb lower lip and pain in her right posterior mandible. A radiogram shows uniform thickening of the periodontal membrane space of tooth #30. The tooth shows abnormally increased mobility. Which one of the following should be seriously considered? A. Periapical cyst B. Periapical granuloma C. Traumatic bone cyst D. Ameloblastoma E. Malignancy

E, Numb lip is malignancy of the jaw until proven otherwise. About half of the patients with numb lip have associated malignancies. The other half of the patients have acute bone infections or neurologic problems

The odontogenic neoplasm, which is composed of loose, primitive-appearing connective tissue that resembles dental pulp, microscopically is known as _____. A. Odontoma B. Ameloblastoma C. Ameloblastic fibroma D. Ameloblastic fibro-odontoma E. Odontogenic myxoma

E, Odontogenic myxomas are connective tissue neoplasms that contain little collagen. This gives them an embryonic look microscopically.

In a tooth-supported RPD with a circumferential cast clasp assembly, there is _____. A. More than 180 degrees of encirclement in the greatest circumference of the tooth B. A distal rest on the tooth anterior to the edentulous area C. A mesial rest on the tooth posterior to the edentulous area D. Only B and C E. All of the above

E, On a tooth-supported RPD with a circumferential cast clasp assembly, there should be more than 180 degrees of encirclement by the clasp in the greatest circumference of the tooth (that passes from diverging axial surfaces to converging axial surfaces). Mesial and distal rests anterior and posterior to the edentulous areas, respectively, are generally used.

A prescription for which of the following drugs requires a valid DEA number on the prescription? A. Amoxicillin B. Carbamazepine C. Dexamethasone D. Diphenhydramine E. Oxycodone

E, Only oxycodone is a scheduled drug, requiring DEA registration on the part of the prescriber

Oral squamous cell carcinomas present typically in which of the following ways? A. Vesicular eruption B. Pigmented patch C. Inflamed pustule D. Submucosal swelling E. Indurated nonhealing ulcer

E, Oral cancers (squamous cell carcinomas) present typically as indurated nonhealing ulcers. They can also present as white patches, red patches, or irregular masses

A generalized red, atrophic tongue would suggest all of the following except _____. A. Vitamin B deficiency B. Pernicious anemia C. Chronic candidiasis D. Iron deficiency anemia E. Peripheral giant cell granuloma

E, Peripheral giant cell granuloma is the exception here. Although it is red, it occurs only in the gingiva. Answers A through D are the differential diagnoses for red atrophic tongue

A 12-year-old patient presents with premature loss of primary teeth. On radiographic exam, a sharply marginated lucency is seen in the area of tooth loss. Biopsy shows a round cell infiltrate with numerous eosinophils. This would suggest which of the following? A. Cherubism B. Gardner's syndrome C. Paget's disease D. Fibrous dysplasia E. Langerhans cell disease

E, Premature tooth loss is seen in several conditions, especially malignancies and Langerhans cell disease because of cellular invasion of the periodontal ligament. Sharply marginated bone lesions are characteristic of Langerhans cell disease (and Paget's disease of the elderly). The eosinophils in a round cell infiltrate suggest Langerhans cell disease (the round cells would be Langerhans cells).

If an exposed radiograph is too dark after proper development, one should _____. A. Place it back in the fixer B. Place it back in the developer C. Decrease development time D. Increase milliamperage E. Decrease exposure time F. Decrease development temperature

E, Reduce exposure time. Do not change development parameters if they are correct

Scaling and root planing are used in which phases of periodontal therapy? 1. Initial (hygienic) 2. Surgical (corrective) 3. Supportive (maintenance) A. 1 only B. 1 and 2 only C. 2 and 3 only D. 1 and 3 only E. 1, 2, and 3

E, Scaling and root planing are used in all phases of periodontal therapy where there has been loss of attachment through periodontitis

Which of the following signs or symptoms suggest a chronic benign process? A. Paresthesia B. Pain C. Vertical tooth mobility D Uniformly widened periodontal membrane space E. Sclerotic bony margins

E, Sclerotic bone margins indicate a long-term, lowgrade process, as it takes a considerable amount of time for bone to become radiodense. The signs and symptoms listed in A through D are associated with malignancies

Which of the following statements about material safety data sheets (MSDSs) is/are correct? A. Employees have the right to know about onthe- job hazards. B. The MSDSs help to protect employees. C. An MSDS contains information on hazardous materials, substance, and wastes. D. The MSDS describes chemical hazards and how to work with the chemical safely. E. All of the above

E, The Material Safety Data Sheet (MSDS) is an easy reference for information on hazardous substances. The MSDS must be "readily accessible" to workers exposed to hazardous substances. The MSDS provides information on hazardous materials, substances, and wastes. Chemical manufacturers develop and provide an MSDS for each hazardous product. The distributor is responsible for getting MSDSs to employers. At least one copy of the MSDS should be maintained with the chemical

The following part of a scientific article summarizes the background and focus of the study, the population sampled, and the experimental design, findings, and conclusion. A. Introduction B. Background C. Literature review D. Methods E. Abstract

E, The abstract allows the reader to determine whether the study is of interest. The abstract usually appears at the head of the article and is reproduced in the literature database.

The use of the rubber dam is best indicated for _____. A. Adhesive procedures B. Quadrant dentistry C. Teeth with challenging preparations D. Difficult patients E. All of the above

E, The advantages and benefits of rubber dam usage are reflected in all of the items stated. The rubber dam isolation increases access and visibility

The use of the rubber dam is best indicated for _____. A. Adhesive procedures. B. Quadrant dentistry. C. Teeth with challenging preparations. D. Difficult patients. E. All of the above.

E, The advantages and benefits of rubber dam usage are reflected in all of the items stated. The rubber dam isolation increases access and visibility.

A clinical differential diagnosis of an asymptomatic submucosal lump or nodule in the tongue would include all the following except _____. A. Traumatic fibroma B. Neurofibroma C. Granular cell tumor D. Salivary gland tumor E. Dermoid cyst

E, The dermoid cyst occurs in the midline floor of mouth when above the mylohyoid and geniohyoid muscles, and in the neck when below the mylohyoid and geniohyoid muscles

Which of the following disinfectants can be used with alginate impressions? A. Alcohol B. Iodophor C. Glutaraldehyde D. All of the above E. B and C only

E, The impression should be rinsed and disinfected with glutaraldehyde or iodophor and should be poured within 15 minutes from the time the impression was removed from the mouth

Where do lesions commonly occur in the primary form of acute herpetic gingivostomatitis? A. Buccal mucosa B. Tonsils, hard and soft palate C. Tongue D. Gingiva E. All of the above

E, The location of lesions of primary herpetic gingivostomatitis is on mucous membrane, including tonsils, hard and soft palates, buccal mucosa, tongue, palate, and gingiva. Children with this disease can be very sick and require close supervision and support. They typically have a very significant fever, can become dehydrated, and the process lasts up to 2 weeks. Treatment may consist of: a. Topical anesthetics such as 0.5% dyclonine hydrochloride and viscous lidocaine b. Coating solutions such as diphenhydramine elixir and kaolin-pectin compound c. Antivirals such as acyclovir d. Analgesics such as acetaminophen and ibuprofen

Dihydrofolate reductase is an enzyme inhibited by which anticancer drug? A. Bleomycin B. Cisplatin C. Doxorubicin D. 5-fluorouracil E. Methotrexate

E, The mammalian enzyme form of dihydrofolate reductase is the target for methotrexate. Bleomycin produces strand breaks in DNA. Cisplatin is an alkylating agent. Doxorubicin intercalates with DNA. 5-Fluorouracil, after undergoing activation, inhibits thymidylate synthase

All of the following characteristically present under the age of 20 except _____. A. Traumatic bone cyst B. Adenomatoid odontogenic tumor C. Ameloblastic fibroma D. Compound odontoma E. Ameloblastoma

E, The mean age for ameloblastoma is 40 years. All other lesions listed occur in children and teenagers

The following were the scores for six dental students in their Restorative Dentistry exam: 56, 64, 68, 46, 82, 86. Therefore, the median is _____. A. 68 B. 64 C. 67 D. 40 E. 66

E, The median is the middle of a distribution: half the scores are above the median and half are below the median. The median is less sensitive to extreme scores than the mean, making it a better measure than the mean for highly skewed distributions. For instance, the median income of a population is usually more informative than the mean income. When there is an even number of numbers, the median is the mean of the two middle numbers. Thus, in this case the median is (64 + 68)/2 = 66.

The design of a restored occlusal surface is dependent upon the _____. 1. Contour of the articular eminence. 2. Position of the tooth in the arch. 3. Amount of lateral shift in the rotating condyle. 4. Amount of vertical overlap of anterior teeth. A. 1 and 3 B. 2, 3, and 4 C. 2 and 4 only D. 3 and 4 only E. All of the above

E, The posterior and anterior factors, position in the mouth, and side shift have influence on the occlusal anatomy of a restoration.

The analgesic effects of dextromethorphan are due to what receptor effect? A. Gamma aminobutyric acid (GABA) receptor antagonism. B. Dopamine receptor antagonism. C. Nicotinic cholinergic receptor antagonism. D. Mu (μ) opioid receptor antagonism. E. N-methyl-D-aspartate (NMDA) receptor antagonism.

E, The recently described mild analgesic effect of dextromethorphan has been linked to N-methyl- D-aspartate (NMDA) receptor antagonism in the CNS.

The recommended level of fluoride for community water supply systems in the United States ranges from _____. A. 0.2-0.5 ppm B. 0.7-1.2 mL C. 1.2-1.5 ppm D. 0.2-0.5 mL E. 0.7-1.2 ppm

E, The recommended level of fluoride for a community water supply in the United States ranges from 0.7 to 1.2 ppm of fluoride, depending on the mean maximum daily air temperature over a 5-year period. Thus, in a warm climate the fluoride level would be lower and in a cold climate it would be higher. In the United States, most communities are fluoridated at approximately 1 ppm, which is equivalent to 1.0 mg of fluoride per liter of water.

Empathic understanding reflects which of the following characteristics? A. It accurately reflects others' feelings. B. It connects the feelings to concrete circumstances that are likely to be causing the feelings. C. It accepts the feelings as real and important. D. It is nonjudgmental: it does not compromise the listener's objectivity. E. All of the above

E, The use of empathy serves a number of purposes, including all of the choices given.

Which statement is true about the use of silence as an interviewing technique? A. It permits and encourages patient participation. B. It is a nonverbal technique for showing interest in the patient. C. It is a nonverbal technique for encouraging the patient to speak. D. It is done by silently attending to the patient, while maintaining eye contact. E. All of the above

E, The use of silence can be a useful technique to encourage patient comment following a statement or question posed to the patient.

Identify the enzyme whose inhibition is most responsible for the cell wall synthesis inhibitory effect of penicillin G. A. β-lactamase B. DNA gyrase C. Nitro reductase D. Transglycosylase E. Transpeptidase

E, Transpeptidase is the enzyme that catalyzes the peptide crosslinking of peptidoglycan. Transpeptidase, is inhibited by penicillins and cephalosporins

On a routine radiographic exam, a well-defined radiolucent lesion was seen in the body of the mandible of a 17-year-old boy. At the time of operation, it proved to be an empty cavity. This is a(an) _____. A. Osteoporotic bone marrow B. Aneurysmal bone cyst C. Odontogenic keratocyst D. Static bone cyst E. Traumatic bone cyst

E, Traumatic bone cysts characteristically occur in the body of the mandible of teenagers. They are pseudocysts in that they have no epithelial lining. They are empty cavities

Which drug is often combined with sulfamethoxazole for the treatment of respiratory tract and urinary tract infections? A. Amoxicillin B. Ciprofloxacin C. Clindamycin D. Metronidazole E. Trimethoprim

E, Trimethoprim, by virtue of its inhibition of bacterial dihydrofolate reductase, acts synergistically with the sulfonamides

Which is a main function of a guide plane surface contacted by a minor connector of an RPD? A. Provides a positive path of placement and removal for an RPD B. Can provide additional retention C. Aids in preventing cervical movement D. All of the above E. Only A and B

E. The contact of the framework with parallel tooth surfaces acting as guide planes provides a positive path of placement and removal for a removable partial denture. In addition, guide planes can provide retention by limiting the movement of the framework. The rest on a removable partial denture prevents vertical or cervical movement.

The mean energy of an x-ray beam is influenced by the _____. A. Kilovoltage B. Milliamperage C. Voltage in the filament circuit D. Quantity of electrons in the tube current E. Amount of filtration F. Two of the above G. None of the above

F, The mean energy (wavelength) of an x-ray beam is influenced by the kilovoltage setting on the machine and the amount of built-in filtration that preferentially absorbs low-energy photons.

Which of the following are probably not clinically significant in terms of influencing the retention of a cemented restoration? 1. Tooth preparation 2. Surface textur 3. Casting alloy 4. Tooth taper 5. Luting agent A. 1, 3, and 4 B. 1, 2, 3 C. 1, 2, 3, 5 D. 3 and 5

The casting and luting agent have been shown to have a minimal effect in the retention of a crown. The geometry of the preparation, parallelism between the walls (taper), and surface texture of the preparation have an effect on the retention of a crown.


Conjuntos de estudio relacionados

Life 2 (Chapters 5 & 6) practice test

View Set

cosa 30 domain 6.0 research fluency

View Set

ACCT 284 LS: Stockholder's Equity

View Set

Comm 219 Midterm #1 Chapters 1, 2, 3, 7, 8, 10, & 12

View Set